Sie sind auf Seite 1von 151

EXERCISE l(A)

x 2 +2,x ~ 1 '
Ex.1 lff(x) = , then LHV- f(x) equals ­
{ 2x+l,x<1 x-»

(A) 1 (B) 2
(C) 3 (D) Does not exist
Sol. lim fix)
x-->1-0
\
lim = [2(I-h)+1]= 3
h-->O

lim
x-->I+0 \
lim [(1+h)Z + 2] = 3
fix) = h-->O
. LHL = RHL, so lim f(x)
., \ = 3. x~l Ans.[C]

, l+e- I1x
Ex.2 lim is equal to ­
x-->o 1- e -II x
(A) 1 (B)-1
(C) 0 (D) Does not exist
IIh
I
Sol. LHL= lim ~ lf h h-->O 1- e
-IIh I

= h-->O
lim = e +-1

e-1/h-l

, l+e- II h 1+0
RHL = 11m = =-= 1 Ans.[D]
h-->O l_e-1/ h 1-0
LHL ;;/:. RHL, so given limit does not exist.

!
X- l, X < O

Ex.3 lff(x)= lI;,X: O then ~~o f{x)equals­


x ,x> o
(A) 0 (B) 1
(C) -1 (D) Does not exist
Sol. Here x---+o+
lim f(x)
\
= Jim
x-+o+
X2 = 0

lim_ f(x) = lim_ (x-I) =-1


and x-->O \ x-->o

.. lim f(x);;/:. lim_ f(x)


x---+o+ \ x-s-O \

!~ f(x) does not exist. Ans.[D]

Ex.4 lim~'
I x -31' IS equal to­
x-->3

(A) 1 (B)-1
(C) 0 (D) Does not exist
LHL - lim- (3~h)-3
Sol. - h-->O 1(3-h)-31

-h
= lim -=-1
h-->O I-hi

RHL - liim ----'---'-­


(3+h)-3
- h-s-O 1(3 + h) - 31

1
ee
· -h: =
I im 1
h-tO Ih I
LHL *- RHL, so limit does not exist. Ans.[D]

. 2x 2 +3x
Ex.S lim equals-
x-too 3x2+4
(A) 1/2 (B) 2/3
(C) 3/4 (D) 0
-lim 2+(3/x) 2
Sol. - x-too 3+(4/x 2 ) 3 Ans.[B]

Ex.6 ~~~ (~x2 + I-X) equals-


(A)-1 (B) 0
(C) (D) None of these

Sol. Limit> lim


X~OO
x[(I+~)ll2 ;'1]
x~'

:= lim X [1+- 2
1- -_1_+ ...-1]
x-too 2x 8x~

:= lim [_1 __1_+...]:= 0


x-eec2x 8x3 •
Ans.[B]

x-sin x .
Ex.7 Iff(x) :=-1----::-2 - ,then lim f(x) equals -
x + cos X x-too

(A) 0 (B) 00
(C) i (D) None of these
{I-(sin xix)}
Sol. lim f( x)
x-too \
= x-too
lim
{I + (cos 2 X I x)}

Ans.[C]
(
Ex.8
. [x -(a+I)x+a]
lim
x..... x3 _a3
2
is equal to -

a-I
(A)- (B) a- I
3a 2
(C) a (D) 0

(~oform)
2
Sol. lim [x -Ca3 + 1)x
3
+a]
x-e-a x _a

:= lim
2x-a-I - a-I
x.... a 3x 2 - 3a2
(D.L.Hospital rule) Ans.[A]

2
lim x
Ex.9 tt:": r;----- is equal to ­
x-sO v1+x-v1-x
(A) 1/2 (B)2
(C) 1 (D) 0

Sol.
"
LImit l'
= 1m
x(.Jl+x +~)
---'--------'­
x-eo (1+x)-(1-x)

= lim .J];"; +~ = 1
Ans.[C]
x-->o 2 .

. tan 2x - x
Ex.10 lim
x-e-O 3x
.
-Sill X
equals-

(A) 2/3 (B) 1/3


(C) 1/2 (D) 0

Sol.
The given limit is in the form , therefore applying L 'Hospital's rule, we get
" lim 2 sec 2 2x - 1 2-1 1
L mut = -=­ Ans.[C]
x-->o 3 - cos x 3-1 2

, (tanxJI/X
Ex.11 hm
x-->o
-X- is equal to ­

(A) e3 (B) ell 3 (C) 1 (D) e


2
Sol. · ' = I"im (x+x 3/3+ .... JI/X
L imit
x-->o X

= lim (1+~JI/X2
x-->o 3

['.' X ~ 0, so neglecting higher powers of x]

Ans.[B]

Ii sin XO •
Ex.12 1m - ­
x~o x IS equal to ­
(
(A) 1 (B) n (C) x (D) n/180
Sol. Limit = lim sin(7t/180)x
x-e-O X

= lim (7t/180)cos(7t/180)x
x-e-O 1

7t
180 Ans.[D]

Ex.13 lim
x~co
(~-
2
tan -1 XJI/X equals­

(A) 0 (B) 1 (C) co (D)-l


I/X
Sol. Let y = lim ( ~-tan-I x
X-->OO 2 )

3
= lim (corx)!"
X-''''

I -1
:. log y = lim ogcot x
x~c.o X

(Ox 00 form)

=_ lim (l+x
2
r
l
x-.+'" cor" x

-2x

. (I+X 2 ) 2
. x
hm ...0..-----'_
x-.+'" -I =-2 II m - ­ 2

x-'<XlI+x
l+x 2

=-2 JimJ...-=O :.y = eO = 1. Ans.[B]


X-.+<Xl 2x

Ex.14 IfG (x) = -J25-x 2 ,


/.1
then Jim G(x)..!-G(1) equals _
x-e l x-I
(A) 1/24 .(B) 1/5
(C) -.J24' (D) None of these
Sol. Here G(1) = -~25-X2 =-.J24
:. Given limit

= lim -~+.J24

x-e l x-I

= ~i~ ~25-X2 (By L Hospital ruel)

I
=.J24 Ans.[D]

. Jf(x)-3
Ex.15 Iff(9) = 9 and f'(9) = 4, then h~ r is equal to-
x-.+ '\Ix-3 (
(A) 1 (B)3 (C) 4 (D) 9
Sol. Given limit is in % form, so using Hospital rule, we get
I
~.f(x)
2v f (x)

Limit = x-.+9
Jim I

2..h

Ans.[C]

- 1' whenx < -1


Ex.16 Functionf(x)= -x,when-1:;;xs1 IS
{
1, whenx >1

continuous ­
(A) Only at x = 1
(B) Onlyatx=-1
(C) At both x = 1 and x = - 1
(D) Neither at x = 1 nor at x = - 1
Sol. f(-1-O) =-1, f(-1) =-(-1)= 1
=> f( -1-0) 7:- fe-I)
=> f(x) is not continuous at x = - I
Further , f( 1) = -I
f(1+0) = I => f(1)7:-f(1+0)
=> f(x) is not continuous at x = 1. ADS. [D]

Ex.17 Iff(x) = { 0,
k
x cos(l/x), x 7:- 0
xe O
~.... ""-?..\t~ £d e{..
~"f....!"
\., L< ::0'
. contmuous
IS
. L" d~ \:. s eO 4-~ tue ~ ~
at x = 0, then ~€. ~ ~ ;6 ~ I
t!
(A) k < 0 (B) k > 0 (C) k = 0 (D) k z 0
'" Sol. Since f{x) is continuous at x = 0
:. !~ ftx) = f(O)
but f(0)= 0 ( given)
: • x-e-O
lim fr x) = x-e-O
lim xk cos (I/x)
L\.

= 0, ifk > O. ADs.[B]

1 I
--x O<x<­
2 ' 2
0, x=O
1 1
Ex.18 Iff(x) = X=­
2' 2
3 I
--x -<x <1
2 ' 2
1, x =1

( then wrong statement is ­


(A) fix) is discontinuous at x = 0
(B) ftx) is continuous at x = 1/2
(C) f(x) is discontinuous at x = 1
(D) f(x) is continuous at x = 1/4
Sol. Obviously function f{x) is discontinuous at x = 0 and x= I because the function is not defined, when
x< 0 and x> 1, therefore f(O-O) and f{I+0) do not exist. Again

f(.!.-+O)
2
= lim (~-x)
2
=I
x--+1/2

f (.!.--O)
2
lim (~-x)=o
=
2x--+112

function f(x) is discontinuous at x =~ ADs.[B]


2
5
3
X +X2 -16x+20 2
2 ,X ""
Ex.19 Iff(x)= { (x-2) is-
k, X =2
continuous for all values of x, then the value ofk is ­
(A) 5 (B) 6 (C) 7 (D) 8
Sol. .: f(x) is continuous at x =2
:. f(2-o) = f(2+O) = ft2) = k
But ft2+0)
= lim (2+h)3 +(2+h)2 -16(2+h)+20
h-->O (2 + h - 2)2

Ans. [C]

I, x::;2
Ex.20 If~pe function fix) = ax + b, 2<x<4
{ 7,
, x24 (
is continuous at x = 2 and 4, then the values of a and b are­
. (A) 3, 5 (B) 3, -5
(C) 0, 3 (D) 0, 5
Sol. Since f(x) is continuous at x = 2
:. f(2) = x-->
Iim 2
+ f(x)

=> 1 = !~~+ (ax+ b)


.. 1=2a+b ...(1)
Again f(x) is continuous at x = 4,
.. f(4) = !~~_ f(x)
=> 7 = lim (ax+ b)
x-->4

:.7=4a+b ...(2)
Solving (l) and (2), we get a= 3, b = -5. Ans.[B]
x, whenx E Q
Ex.21 Iff(x)= { h Q,thenf(x)
-x, W en x s
is continuous at ­
(A) All rational numbers
(B) Zero only
(C) Zero and 1 only
(D) No where
Sol. Let us first examine continuity at x = O.
f(O)=O (":OEQ)
= f (0-0) = ~~ f( 0- h) = ~~ ft-h)

= ~~ { -h or h according as - h E Q or -h ~ Q)
=0
f( 0+0) = ~~ ftO+h) = ~~ f(h)

6
= lim { h or -h} = 0
h~O

f(O) = f(O - 0) = f(O + 0)

=> f(x) is continuous at x = o.

Now let a E R, a -:t 0, then

f(a--Q) =!~ f( a-h)

= !~ {(a-h) or - (a-h)}
= a or -a, which is not unique.
=> f(a-O) does not exist
=> f(x) is not continuous at aERo.
Hence flx) is continuous only at x = o. Ans.[B]

Ex.22 f(x) = x -[x] is continuous at ­


(A) x=O (B)x=-1
(C) x= 1 (D) x = 1/2
Sol. We know that [x] is discontinuous at every integer. Therefore it is continuous only at
x = 1/2, while the function x is continuous at all points x= 0, -1, 1, 1/2. Thus the given function is
continuous only at x = 1/2. Ans.[D]
/1
x<nl2
Ex.23 Iff(x) = a, x = nl2 is continuous ar x = n12, then value ofa and b are-
b(1-sin x) x > nl2
(n-2x)2 '

(A) 1/2, 1/4 (B) 2,4


(C) 1/2,4 (D) 1/4,2

3(%-h)
l-sin
Sol. f ("::-0)
2
= lim
h~O

3COS
2(%-h
J
· l-cOS3h
= I1m _-::-_

h->O 3sin2h

° (1- cosh)(1 + cosh--cos" h)


= 1~ =In
h->O 3(I-cosh)(1 + cosh)

= lim b(l- cosh)


h->O 4h 2

= lim 2b~n2h/2 b
h-+O 4h 2 8

Now f(x) is continuous at x = 2:


2

=>f(~-OJ=f(~+OJ =f(~J =>~=%=a

.. a = 1/2, b = 4 Ans.[C]
7
Ex.24 If the function

1+sin2: xfor-co < x ~ 1


2
f(x) = ax + b for I < x < 3
X1t
6 tan - for 3 ~ x < 6
12

is continuous in the interval (- co, 6), then the value of a and b are respectively ­
(A) 0, 2 (B) 1, 1 (C) 2, 0 (D) 2, 1
Sol. Obviously the function f(x) is continuous at x = 1 and 3. Therefore !~+ f(x) = f(1)
~ a+ b=2 ...(1)
and !~~_ f(x) = f(3)
~3a+b=6 ...(2)
Solving (1) and (2), we get a = 2, b = O. Ans.[C)

l-cos4x
2
0
,x < r' {
x
Ex.25 Iff(x) = a, x=o
s: .x c- O
~16+~ -4
then at x = 0­
(A) f(x) is continuous, when a = 0
(B) f(x) is continuous, when a = 8
(C) f{x) is discontinuous for every value of a

(D) None of these

22x
Sol. f(O-O) = lim l-cos4x = 2sin =8
x-->o x2 x2

, £ ~+4
f(O +0) = Iim x
x-e-O (~16+£ -4) ~16+£ +4

= lim £(~ +4)=8 (


x-e-O 16+£-16
f (0 + 0) = f{0 - 0)

f(x) can be continuous at x = 0, if

f(O) = a= 8. Ans.[B]

sin[ x] 0
--x>
[x]+I'

cos-­
Ex26 Iff(x)= lim 2[x] ,x <0
• \ x---+o [x]

k, x = 0

8
(Where [x] = greatest integer s x) is continuous at x = 0, then k is equal to ­
(A) 0 (B) 1
(C) -1 (D) Indeterminate
Sol. As given f(O - 0) = f(O + 0) = k
cos (-h)
Now f(O - 0) = h....O
lim 2[-h]
[-h]

= lim cosC~:I))
h....O --'--:........;.~

=_ 1
-I

f(O + 0) = lim sin[h] = lim ~ =0


h....O [h]+1 h....O 0+1

.: f(O - 0) :t- f(O + 0), so k is indeterminate. Ans.[D]

(l + Isinx l)aljsin xl,_n/6 < x < 0

Ex.27 Iff(x) = b , x=0


{ etan2x/tan3x, 0 < x < n/ 6

is continuous at x = 0, then value of a, bare ­


(
(A) 2/3, e2/3 (B) 1/3, e ll 3
(C) 2/3, 1/3 (D) None of these
Sol. f (0-0) == ~~ (1+ I sin (_h)l)aJ1sinC-hll
= (1+ sin h)alsinh =e a

f( o+ 0) = h~
r tan2h
e tan3h =
r
eh~
(tan2h)
tan3h

lim 2sec' 2h _ 213

e".... 3sec' 3h - e

Now f(x) is continuous at x == 0


=> f(O-O) = f(0+0) = f(O)
=> ea = e2!3 =b
.. a = 2/3, b= e2l3 Ans.[A]

Ex.28 f(x) == [x] is not differentiable at ­


(A) x=-l (B)x=O
(C) x= 1 (D) None of these
Sol. at x = 0:

f' (0-0)= limIO-hl-O =-1


h....O -h

f' (0+0)= limIO+h\-O = 1


h....O h
N ow, since f' (0-0) :t- f' (0+0)
=> f(x) is not differentiable at x = O. Ans.[B]

- x if x <0
2
Ex.29 Function f(x) = lim x : if 0 s x s 1, is
h~O{
x 3 -x+ 1, if x> 1
differentiable at ­
(A) x = 0 but not at x = 1
(B) x = 1 but not at x = 0

9
(C) x = 0 and x = 1 both
(D) neither x = 0 nor x = 1

Sol. Differentiability at x = 0

R [f' (0)] = lim f(O+h)-f(O)


h-.O h

0 _ limh-O
-- 1°1m (O+h)2_ - ­
h.....O h h..... o

L [f' (0)] = lim f(O-h)-f(O)

h.....O -h

= lim -(O-h)-O =-1

h.....O -h

R [f' (0)] =1= L [f '(0)]

f(x) is not differentiable at x = 0

Differentiability at x = 1

R[f'(l)]= lim f (1+ h)3 - f (l)


h.....O h

- 10 (1+h)3-(l+h)+I-1
- Im~~~-~-
h.....O h

-1°
- 1m
2h+3h 2+h 3_
- 2

h.....O h

L [f'(1)] = lim f(l-h)-f(I)

h.....O -h

= lim -,-(l_-_h~)-_I

h.....O -h

2
- 1°1m -2h+h -­ 2
-
h.....O -h

ThusR [f' (1)] =Lf'(l)]

:. function fix) is differentiable at x = 1


Ans.[B]

3X,- 1S; X S; 1
Ex.30 If f(x) = { 4- x.l < x < 4

then at x = 1, f(x) is ­
(A) Continuous but not differentiable
(B) Neither continuous nor differentiable
(C) Continuous and differentiable
(D) Differentiable but not continuous
Sol. Since fi(l..c..Q) = x.....
lim1 3x = 3

f(1+0) = ~~ (4-x)=3
and f(l) = 3 1 = 3

f(l..c..Q) = f(1+0) = f(l)

:. f(x) is continuous at x = 1

=> Again f' (1 + 0) = lim f(x) - f(I)

x .....l+ x-I

x-3
_ I. 3
- I m - ­
x .....l x-I

10
. 3J+h_3
=hm-­
h->O h

. 3h-I
=3hm-­
h->O h
= 31og3
and f '(1+ 0) lim f(x) -f(I)
x-I
x ....l "

. 4-x-3
= hm =-1
x-s-l x-I
.. f' (1+0)*f'(1-0)
(}/I- f(x) is not differentiable at x = 1. Ans.[A]
x
x.31 Function ftx) = 1+ I x I is differentiable in the set­
(A) (-ex:>,ex:» (B)(-<lO,O)
(C) (-<lO, 0) U (0, co (D)(O, co
x
Sol. When x <0, f(x) ~r~-x
( 1
f'(x) = (I-x)2 ... (1)

which exists finitely for all x < 0


1
Also when x > 0, f(x) = -1­
+x
1
~f'(x)=(l+X)2 ...(2)
which exists finitely for all x > O. Also from (1) and (2) we have

e:
f '(O- O) = 1
{ f'(O+O)=I ~f'(O)=l ~
Hence f(x) is differentiabl~ R Ans.[A]

2
x sin ~,x * 0
Ex.32 Iff(x) = x , then
{
0, x=0
(
(A) f and f' are continuous at x = 0
(B) fis derivable at x = 0
(C) f and f' are derivable at x = 0
(D) f is derivable at x = 0 and f' is continuous at x = 0
Sol. When x *0
f' (x)=2xsin -+x2cos -. - 2 1
x
1( x1)
x

. 1 (1)
= 2x sm -;- cos -;
which exists finitely for all x*-O
2sinllx
and f' (0) = lim f(x)-f(O) lim x = 0
x->O X- 0 x->O X

:. fis also derivable at x = O. Thus

11
f' (x) =
! 2x sin l- - cos l- , x * 0

0,
x x
x =0

Also xuirn
....o
).rm (2xsm--cos­
f' (x) = x....o
. 1
X
1)
X

)1m . 1
= 2- x....o
COS -
X

But ;~ cos'; does not exist, so !~ f' (x) does not exist. Hence f' is not continuous (so not

derivable) at x = O. Ans.[B]

Ex.33 If x = a (8 + sin 8), y = a (1- cos 8), ~~ then dy/dx equals ­


(A) tan 8 (B) cot 8
1 I
(C) tan 28 (D) cot 2 8

dx dx.
Sol. de = a (1 + cos), de = a sin 8
(
dy dy I de a sin e I

:. dx = dx/de = a(l+cose) = tan 2 e Ans.[C]

Ex.34 If y = log (e:: J' then dy/dx equals ­

(A)

(C) (D) None of these


Sol. Y= log e" -log (e" + 1)
=x-log(ex + 1)

. dy = 1- ~=_1_
X+l
Ans.[A]

., dx e eX+l

Ex.35 If y = -Z--Z
)
, then
dZy
-Z equals-
(
x -a dx

3x +a
Z z 3x 2 +a z

(A) (x Z_a Z ) 3 (B) (x Z_a 2 ) 4

2(3x 2 +a 2 ) 2(3x 2 +a 2 )
(C) (x 2 _a 2 ) 3 (D) (x Z_ aZ) 4

Sol.

(x 2 _ a 2 ) 2 .2- 2x.2(x 2 - a 2 ).2x


-- (x 2_a 2 ) 4

2
2(3x 2 +a )

= (x 2_a 2 ) 3 Ans.[C]

12
sec x - tan x dy
Ex.36 If Y= ,then -d equals­
secx+ tan x x
(A) 2 sec x (sec x - tan X)2
(B) - 2 sec x (sec x - tan x)'
(C) 2 sec x (sec x + tan X)2
(D) - 2 sec x( sec x + tan x)'
sec x - tan x sec x - tan x
Sol.
y= sec x i tan x ' secx-tanx
= (sec x - tan x)21l
dy

:. dx = 2(secx - tanx) (sec x tan x-sec- x)

=-2 sec x (sec x- tan x? Ans.[B]

dy
Ex.37 If x J1+Y + y~ = 0, then dx equals­

1 1
(A) (l+x)2 (B) - (l+X)2

.F' 1
(C) 1+x 2 (D) None of these
Sol. Let us first express y in terms of x because all alternatives are in terms of x. So

x v1+; = -yJ!D.

=> x" (l + y) = y2 (l + x)

=> x2 -y2 + x2y - y2 X = 0


=> (x-y) (x +y + xy) = 0
=> x + Y + xy = 0 (': x 7: y)
x
=>y=-­
I-x

dy = (l+x)l-x.l = __1_

.. dx (l+x)2 (l+X)2 Ans.[B]

dy
Ex.38 If x> yX = 1, then dx equals-

x(y+xlogy)
(A)
y(x+ylogx)

x(x+ylogy)
(B)
y(y+xlogx)

y(y+x)ogy)
(C)
x(x+ylogx)

y(y+xlogy)
(D) x(x+ ylogx)
Sol. Taking log on both sides, we have
y log x + x log Y = 0
Now using partial derivatives, we have
dy =_ y / x + log y y(y+xlogy)

dx logx+x/y x(x+ ylogx) Ans [D]

13
Ex.39 Ify = ~Sinx+~Sinx+.JSinx+ ......a: , then dy/dx equals­

sinx COS X
(A) 2y+l (B) 2y-l

cosx
(C) 2y+l (D) None of these

Sol. Herey= ~sinx+y => y2= sin x + y


dy dy dy cos x
:.2y dx =cosx+ dx => dx = 2y-l Ans.[B]

Ex.40 If ex+e"<"~ , then dy/dx is ­


y
(A) (B) ----'L
l+y y-l

y
(C) l-y (D) None of these
Sol. y = eX+Y

logy=x +y => J.. dy =1+ dy


y dx dx

dy y
dx - l-y Ans.[C]

14

EXERCISE 1(B)
More than one options may be correct

x . en (cos x) x '* 0
If f(x) = en (I +x2 ) then:

o x=O
(A *) fis continuous at x = 0
(B) fis continuous at x = 0 but not differentiable at x = 0

(C*)fis differentiable at x = 0

(D) fis not continuous at x == O.


2
· hln(cosh) . In(cosh)l/h 1 1 1
[Hint: f' (0+)- L1m 2 - LIm = Lim -(cosh-I) =- _. lilly f' (0-) =-­
- h~O hln(I+h ) - h~O In(l+h 2 ) h~O h 2 2' 2
h2
hence f is continuous and derivable at x = °]
2 Which of the following limits vanish?

lim x t sin _1_


( A *) X--Jooo (B*) lim (1 - sin x) ° tan x
J;. x--,>,,12

2
( C) lim 2x + 3 ° s (x) (D*) lim [xf - 9

x-eeo x2 + X - 5 gn x--,>3+ x 2 - 9

where [ ] denotes greatest integer function


Li1m (I-sinx)sinx l-i-sin x
[Sol. 0 _ _­

x~o cosx l-i-sin x


. sinxcosx

L im =0]

x~1C12 1 + sin x

3 Let [xl denote the greatest integer less than or equal to x ° If f(x) = [x Sin 1t x], then f(x) is:
(A*) continuous at x = 0 (B*) continuous in (-1,0)
(C) differentiable at x = 1 (D*) differentiable in (-1, 1)
0 O<x<1
[Hint: f(x)= [ 0 x=Oorior-I ~ f(x)=Oforallin[-I,l]
o -I<x<O
(
4 The function, f(x) == [ x I I] - I I
[xl where [x] denotes greatest integer function
(A*) is continuous for all positive integers
(B*) is discontinuous for all non positive integers
(C*) has finite number of elements in its range
(D*) is such that its graph does not lie above the x - axis °
o

l
x=-I
-1 -I<x<O

[Sol.
[ I x I] -I [xl I == ~ ~ ~::i- . Y

=> :;e~~;s-I} =r:::::q~r"


x
5 Let f (x + y) == f(x) + fey) for all x, y E R. Then:
(A) f(x) must be continuous 'if x E R (B*) f(x) may be continuous 'if x E R
(C) f(x) must be discontinuous 'if x E R (D*) f(x) may be discontinuous 'if x E R
15
[Hint: Limit f (x + h) = Limit f(x) + f'(h) = f(x) + Limit f (h)
h~O h~O h~O

Hence if h ~ 0 f (h) = 0 => I f' is continuous otherwise discontinuous]

6 The function f(x) = Jl-"h-x 2


(A *) has its domain -1 :::: x:::: 1.

(B*) has finite one sided derivates at the point x = o.

(C) is continuous and differentiable at x = O.

(D*) is continuous but not differentiable at x = O.

1
[Hint: f'(O+) = - . f' (0-) =- -
1
. f(x) =
~ =
Ixl ]
.J2' .J2 ' J1+~1-x2 J1+~1-x2
7 Consider thefunction f(x) = I x3 + 1 I then
(A*) Domain of f x E R
(B) Range of f is R+

(C*)f has no inverse.

(D)f is continuous and differentiable for every xER.

[Sol. Range is R+ v {OJ => B is not correct


,-'
f is not differentiable at x = - 1 (
x 3+1 if x z >I 3x 2 ifx>-l

as f(x) = [ => fl(X)=[

-(x 3+1) ifx<-l -3x 2 ifx<-l

f' (-1 +) = 3; f' (-1-) = - 3 => f is not differentiable at x = - 1

also since fis not bijective hence it has no inverse => (C)]

. . 8 2 +8-2 f(8) 8 2 +28-1 . . . .


8 Assume that LIm f(8) exists and s -82 s holds for certam mterval contaming
e~-1 8+3 8+3
the point 8 = - 1 then Lim f(8)
e~-l

(A*) is equal tof(-l) (B) is equal to 1


(C) is non existent (D*) is equal to - 1
1-1-2 1-2-1
[Sol. put 8=-1; 2 ::;f(-l)::; 2
-1::;f(-l)::;-l => f(-l)=-l (
. 82+8-2 . 8 2+29-1
L l lfl =-1 = Llm---­
e~-1 8 +3 e~-1 8 + 3

using sequeeze play theorem

. f(8)

L1m-2- =-1; Limf(8) =-1]

~-1 8 e~-1

9 f is a continous function in [a, b]; g is a continuous functin in [b, c]


A function h (x) is defined as
h(xj=f(x) for x E [a, b)
=g(x) for x e tb;c]
if feb) = g (b), then
(A *) hex) has a removable discontinuity at x=b.
(B) hex) mayor may not be continuous in [a, c]

(C*) h(b-) = g(b+) and lub") = f(b-)

16
(D) htb") = g(b-) and h(b-) = f(b+)
[Sol. Given [ is continuous in [a, b] (1)
g is continuous in [b, c] (2)
feb) = g(b) ....(3)
h (x) =f(x) fOrXE[a,b)}
=f(b)=g(b) forx=b ....(4)
= g (x) for x E (b, c]
h (x) is continuous in [a, b) u (b, c] [using (1), (2)]
also f(b-) == feb); g (b") = g(b) ....(5) [using (1), (2)]
.. h (b) = f(b-) == feb) == g(b) = g(b+) = h(b+) [using (4), (5)
now, verify each alternative. Of course! g(b-) and f (b") are undefined.
h (b) = f (b") = feb) = g (b) = g(b+)

and h (b") = gtb") = g (b) = feb) = f (b")

hence h (b") = h (b") = feb) == g (b)

and h (b) is not defined => (A) ]

(A *) continuous at x = 1 (B*) diff. at x = 1


(C*) continuous at x = 3 (D) differentiable at x == 3

x -3 if x ~3

[Sol. f (x) = 3-x if 1:s; x < 3

x 2 3x 13.
---+-lfx<1
4 2 4

· . 3 - (1 + h) - 2
f Tl") == Limit f(1+h)-f(1) = L Imrt 1
==­
h--70 h h--70 h

(l-h)2 3 13
--'---'-- - -(1- h)+-- 2 (1- h)2 - 6(1- h)+ 5
f ' (1-) = Limit 4 2 4 = Limit --'-----~----'-------'--
h--70 - h h--70 - 4h

2
. . h - 2h + 6h _1
= LI mlt---­
( h--70 -4h

f is continuous at x = 1 ]

11 Which of the following statements are true?

(A *) If xe XY == y + sin-x, then at y I (0) = 1.

(B) If f(x) == a o x2m+ 1+ a l x2m + a3 X2m- 1 + ...... + azm+1= 0 (aa 7:- 0) is a polynomial equation with
rational co-efficients then the equation f' (x) = 0 must have a real root. ( mEN ).

aa
(C*) If (x - r) is a factor of the polynomial f(x) = an x" + an_ 1 xn- I + an- 2 x n- 2 + ..... + repeated m

times where 1 :s; m s n then r is a root ofthe equation f' (x) = 0 repeated (m - 1) times.

(D*) Ify = sin-I(cos sirr' x) + COS-I (sin COS-IX) then :~ is independent on x.

1t
[Hint: (D) Let sin!x = t => COS-IX = - - t
2

17
y = sin-I( cos t) + cos' (sin( - t ~ JJ= sin-I( cos t) + cosl(cos t)
1t dy
y= - => -=0 True]
2 dx

12 Let y = ~x + ~x + Jx + ...... C1J then ~~ =

(A*) _1_ (B) _x_ (C*) 1 (D*) Y


2y -1 x +2y J1 + 4x 2x + Y

dy 1 x dy y
[Hint: y2 = X + Y => - = - - also y = - + 1 => _ = - ­
dx 2y - 1 Y dx 2x + y

make a quadratic in y to get explicit function => C]

13 If.Jy + x +.Jy - x = c (where c:t: 0), then dy has the value equal to

dx

2x x c2
(A*) 2 (B*) y +J y 2-
x 2 (D)­
2y

(
[Hint: square both sides, differentiate and rationalise ]

14 Iff(x) = cos[ ~]cos(%(x - 1)J ; where [x] is the greatest integerr function of x, then f(x) is continuous

at

(A) x = 0 (B*) x = 1 (C*) x = 2 (D) none of these

[Hint: (A) = Not defined at x = 0; B = f (l) = cos 3 ; f(2) = 0 and both the limits exist

15 Select the correct statements.


2
3
(A) The function f defined by fix) = [2X + for x:O:1 is neither differentiable nor continuous at x=1.
3x+2 for x >I

I I
(B*) The function f(x) = x2 x is twice differentiable at x = o.

(C*) IfJis continuous at x = 5 andJ(5) = 2 then Lim J(4x 2 -11) exists.

x--*2

(
(D) If Lim (f(x) + g(x») = 2 and
x--*a
~~ (f(x) - g(x»)= 1 then Lim J(x) . g (x) need not exist.
x--*a

16 Which of the following functions has/have removable discontinuity at x = 1.


1 x 2 -1
(A) f(x) = -II
in x (B*) f(x) = x 3 - 1

(C) f(x)=2-2('~X) (D*) f(x) = JX+i2 --J2;.


x
x
Sol (A) lim
x-e-I
f (x) does not exist

(B) limf(x)=~
x-e-l 3 f(x} has removable discontinuity at x = 1

18
(C) limf(x)
x-->I
does not exist

-1
(D) limf(x) = Tn f(x) has removable discontinuity at x = 1
HI 2'\12

17 Let f(x) = -VIX-2~ . x then:


~-l
(A*) f' (10) = 1 (B*) f' (3/2) = - 1
(C) domain of f (x) is x ~ 1 (D) none

Sol. f(x) =V
1(..Fi)2 + 1- 2~
~ _ 1
.x =
1..Fi - 11 .x = .if
~_1
[-J x E [1 ,2)
@ If x E (2 , 00)
]
(
18 Given that the derivative f' (a) exists, Indicate which of the following statement(s) is/are always True.
. f(h) - f(a) '( ) I' f(a)-f(a-h)
(A*) f'(a) = hm----'-'---'---'- (B*) f a = Im---'--'---'--­
h
h-->a h-a h-->O

(C) f '(a ) = liirn ---'----'-----'---'­


f(a+ 2t)-f(a) f'() li f(a+2t)-f(a+t)
HO t (D) a = t~ 2t

Sol. ° 1
(C) is false and is True only iff' (a) = limit is 2f' (a). In (D) same logic limit is "2f'(a)]
\

X -cos 2 X . IS hdx
19 Let I J = Lim . and 12 = LIIl}. h 2 2' Then

x~ro x+smx h~O -I +x

(A *) both II and 12 are less than 722


(B*) one ofthe two limits is rational and other irrational.
(C*) 12 > II
(D*) 12 is greater than 3 times of I I'

2
1- cos X
[Sol. II = Lim x = 1
( X~ 1+ sinx
x

12 = Lim 2
h~O+
J ~dX+ 2=
0 h x
Lim
h~O+
[2~tan-1
h
~]I
h 0+
= 1t Ans.

22
Note: ->1t [2 = 3.1428571 and 1t::::: 3.1415929]]
7 72

20 f(x) is an even function, x = 1 is a point of minima and x = 2 is a point of maxima for y = f(x).

Further X400
lim f(x) = 0, and lim
X40
f(x) = 00. f(x) is increasing in (l,2) & decreasing everywhere

in (0,1) u (2, (0). Also f(l) = 3 & f(2) = 5. Then


(* A) f(x) = 0 has no real roots

19
(*B) y = f(x) and y = If(x)1 are identical functions

(*C) f''(x) =0 has exactly four real roots whose sum is zero

(D) f''(x) = 0 has exactly four real roots whose sum is 6

Sol. lim f(x) = 00, lim f(x) = 0


X-70 x~'"

f(x) is increasing in (1, 2) and decreasing in (O,1)1j(2,00)

from the graph Hence (A), (B), (C)

-2 -1 2

PASSAGE 1

A curve is represented parametrically by the equations x = f(t) = alnCbt) and y = get) = b-lnCat)

°
a, b > and a "j; I, b "j; I where t E R.

21 Which of the following is not a correct expression for ~~ ?


-1 (C) - get) - f(t)
(B) - (g(t»)Z (D*) - ­
(A) f(t)z f(t) get)

dZy
22 The value of -Z at the point where f(t) = get) is
dx

(A) ° (B) ­
2
1

(C) 1 (D*) 2

f(t) f"(-t) f(-t) fIlet)


23 The value of f'(t)' f'(-t) + f'(-t)' f'(t) Vt E R, is equal to
(A) - 2 (B*) 2 (C) - 4 (D) 4

[Sol. x = f(t) = alnCbt) = at In b ....(1)

Y = get) = b-lncat~ = (b1na)-t = (a1nb)-t = a-tin b

.. Y= get) = alnCb-t) = f (- t) ....(2)

From equation (1) and (2)


{
xy = I

I dy I

(i) .. y=- :. dx = -~= - f\t) (A) is correct


x

dy I yZ

Also xy =I => dx = -~=--I =- gZ(t) => (B) is correct

dy Y get)
Again xy = I => -=--=-­ (C) is correct (D) is incorrect
dx x f(t)

(ii) f(t) = get) => f(t) = f(-t) => t = 0


{ .: f(t) is one-one function}
At t = 0, x = y = I

20
At x = 1,

(iii) " xy = 1 :. fg = 1 ., fg'+gf'=O


f g" + g' f' + g' f" + g f " = 0

fg"+gf"+2g'f'=O

f g'' gf"
--+-=-2 ....(3)
f' g' g'f"
from equation (2) g(t) = f(-t)

" g'(t)=-f'(-t)

and g"(t)=f"(-t)
substituting in equation (3)
f(t) . f"(-t) + f(-t) .f"(t) =-2
f'(t) -f'(-t) -f'(-O f'(t)

f(t) J"(-t) + f(-t) J"(t) = 2 Ans

f'(t) f'(-t) f'(-t) f'(t)

PASSAGE 2

1
[x] ,-2~x~--
Let a function be defined as f(x) = 2, where [.] denotes greatest integer
2 1
{2x -1 , --<x< 2
2 ­
function.

Answer the following question by using the above information.

24 The number of points of discontinuity of f(x) is


(A) 1 (B*) 2 (C) 3
(0) °
Sol Two points of discon. -1, -1 / 2 .
(
-2 -1 -1/2 1/../2

, ,
_ _ ____e
r
25 The function f(x-l) is discontinuous at the points
1 1. 1
(A) -1'-2 (B) -2'1 (C*) 0'2 (0) 0,1

21
Sol Discontinuous at 1,1/2

-1 112
1+1/../2

26 number of points where If(x) I is not differentiable is


(A) 1 (B) 2 (C*) 3 (D) 4

Sol

-1 -1/2 (
1
.J2
at -1,-l/2,1/J2 the functionis not differentiable.

PASSAGE 3
Two students, A & B are asked to solve two different problem. A is asked to evaluate

lim
x~o
1- cos(ln(1 + x))
x2
& B is asked to evaluate lim ~ + ~
n~oo \ln 3 +1 \ln 3 +2
+ +~ J,n EN. A provides
(.In.Jn.Jn
\ln 3 +2n
the following solution

Let
1- cos(ln(1+X) .
x
x) 1-cosx
(A 5
u1m [n(1 + x)
--=
1) =:)
1
11 = ­
[1 = lim 2 lim - - 2- X~O x 2
x~o x x~o X ~

B provides the following solution


(
.fri)
2n __n_ = lim {2n
{~ ~n3 +r
.fri ] 1
n~oo ~ R
Let 12 = lim n
n-e-co n

= n~oo
lim n [!{R1+R2+·······+A2!J
n+-
n 2 n+-
n 2 n+---.D.
n 2

lim
= n~OO
[![~ + 1+; ....+1]] = lim 2n = 2
n 2n times n~oo
n

27 Identify the correct statement

(*A) both of them get the correct answer

(B) both of them get the incorrect answer


(C) A gets the correct answer while B gets the incorrect answer.

22

(D) B gets the correct answer while A gets the incorrect answer.

28 Who has solved the problem correctly


(A) A (B) B (C) both of them (D*) no one

4l, (tanx ~3 sinx) x< a


29

..
f(x) Z2
(eX1 -- cosx
x - 1 )
x = a where l1 and l2 are correct values of the corresponding limits, if

x> a

is continuous at x = 0 the K is equal to­

(A) 1 (B) 2 (C) 3 (*D) no value ofK

Z1 = lim 1- cos(Zn (1 + x)).(zn (1 + X}J2 =!

Sol.
x~o Zn 2 (1 + x) x 2

A & B have made the same mistake, they used the notion of limit partly in the problem, where
( as once the limiting notion has been used the resulting expression must be free from the
variable on which the limit has been imposed
"
Z1m 2n.m Z Z" 2n.m
~<2< 1m ~
n~lXl "n 3 +1 n~lXl 3 + 1
"n
Hence 12 = 2 (sandwich theorem)

S01.1 Hence (A)

So1.2 Hence (D)

So1.3 lim 4.!(tanx - sinxJ = 4.!.! = 1


x~o 2 x3 2 2
2
lim
x~o
Z2[e
X
-:
x
-1. x
1-cosx
J=2(2.2} =8
for no value of K Hence (D)

PASSAGE 4

( Let fix) is a function continuous for all x E R except at x = O. Such that f" (x) < 0 \;j XE (- 00, 0) and
f I (x) > ° \;j X E (0, 00). Let Lim f(x) = 2,
X --70+
Lim f(x) = 3 and f (0) = 4.
X--70

3_X2 4-X 5
30 The value of 'A for which 2 (Lim f(X )) = 'A (Lim f(2X ) \) is
X--70 X--70

r
(A) ­ (B) 2 (C*) 3 (D) 5
3
2
31 The values of Lim· -x) X } where [ . ] denote greatest integer function and { . } denote fraction
X--70+ l-cosx
[f(x)]
part function.
(A) 6 (B*) 12 (C) 18 (D) 24

23
[[ ( )] [[ ]\]J
3
X -sin 3 x sin x"
32 3f x4 -f -x- where [ . ] denote greatest integer function.

(AP (B*) 5 (C)7 (D) 9

Sol.
(i) x -e O , x3 - x2 = x2(x - 1) ~ 0­
x -o O , 2x 4 - x5 = x4(2 - x) ~ 0+ y
2 (3) = A (2) => A= 3 Ans.

(ii)
f(-x)x 2 ~,
(\~~:;; )_[ 1[~~~;; ]
l-cosx 0
2

= 6 x 2 = 12 Ans.

2 . 2 .) 1
(iii) x +sm x+xsmx x= - (3)x~ 0- => f(O-) = 3
( 2 I 6 "
X
/
3] (
Sin x
=> [ x =0 f(O) = 4
.
3
x -sin3 x)
3f ( 4 >9
x
[9+] - f(O) = 9 - 4 = 5 Ans.]

Assertion I reasoning type

33 Let h (x) = f l (x) + fix) + fix) + + fn(x) where flex), f2(x), fix), , fn(x) are real valued functions
ofx.
I II
Statement-It I(x) = cos I x +cos-\sgn x) + lin x \ is not differentiable at 3 points in (0, 2n)
because

Statement-2: Exactly one function flx), i = I, 2, ..... , n not differentiable and the rest ofthe function

differentiable at x = a makes h (x) not differentiable at x = a. (

(A *) Statement-I is true, statement-2 is true and statement-2 is correct explanation for statement-I.

(B) Statement-I is true, statement-2 is true and statement-2 is NOT the correct explanation for statement- I.
(C) Statement-I is true, statement-2 is false.
(D) Statement-I is false, statement-2 is true.
[Sol. y = lin x I not differentiable at x = 1
n 3n
y = Icos I x II is not differentiable at x = "2' 2:
y = cosJ(sgn x) = cos:' (1) = 0 differentiable v x E (0, 2n) ]

34 Statement-I: f (x) = I x I sin x is differentiable at x = 0


because
Statement-2 : If g (x) is not differentiable at x= a and h (x) is differentiable at x = a then g (x) . h (x) can
not be differentiable at x = a.
(A) Statement-I is true, statement-2 is true and statement-2 is correct explanation for statement-I.

24
(B) Statement-1 is true, statement-2 is true and statement-2 is NOTthe correct explanation for statement-I.
(C*) Statement-I is true, statement-2 is false.
(D) Statement-1 is false, statement-2 is true.

hsinh-O .

[Sol. f" (0+) = = 0


h
hsin(-h)-O
f" (0-) = =0
-h

f(x) is diff. at x = 0

e.g. x I x I is derivable at x = 0]

7t
35 Statement-It f(x) = I cos x I is not deviable at x = "2 .
because

Statement-2: If g (x) is differentiable at x = a and g (a) = 0 then I g(x) I is non-derivable at x = a.

(A) Statement-1 is true, statement-2 is true and statement-2 is correct explanation for statement-I.
(B) Statement-I is true, statement-2 is true and statement-2 is NOTthe correct explanation for statement-I.
(C*) Statement-l is true, statement-2 is false.
(D) Statement-l is false, statement-2 is true.
[Hint: Consider g (x) = x 3 at x = 0; g (0) = 0
I g(x) I is derivable as x = 0
actually nothing definite can be ~a!d/lso for g (x) = x - 1 with g (1) = 0
then I g(x) I not derivable at x = ~ .

36 Letf(x) =x - x2 and g (x) = {x} \:j X E R. Where { . } denotes fractional part function.
Statement-1: f(g(x)) will be continuous \:j x E R.
because
Statement-2: f(O) = f(l) and g (x) is periodic with period 1.

(A*) Statement-1 is true, statement-2 is true; statement-2 is correct explanation for statement-I.

(B) Statement-1 is true, statement-2 is true; statement-2 is NOT the correct explanation for statement-I.
(C) Statement-l is true, statement-2 is false.
(D) Statement-l is false, statement-2 is true.
[Hint:

~
-1 0 1 2 3

37 Statement 1: f(x) = sinx + [x] is discontinuous at x =0


because
Statement 2: If g(x) is continuous and hex) is discontinuous at x = a, then g(x) + hex) will
necessarily be discontinuous at x = a
(A *) Statement-l is true, statement-2 is true and statement-2 is correct explanation for statement-I.
(B) Statement-1 is true, statement-2 is true and statement-2 is NOT the correct explanation for statement-I.
(C) Statement-1 is true, statement-2 is false.
(D) Statement-I is false, statement-2 is true.

Sol !~.(sinx+[x])=O

25
lim (sinx + [x]) =-1
x....o
Limit doesn't exist

~~(f(x) + h(x)) = ~~f(x)+ ~~h(x)

;t:f(a) + h(a)
f(x) + h(x) is discontinuous function

Matrix match type

38 Column-I Column-II
l-cos2x
(A) Lim
x~o x2 x equa 1s (P)
e -e +x

, ((3/X)+ 1)I/X

(B) If the value of ;~~ (3/x)-1 can be expressed in the


(Q) 2

form of ep/q, where p and q are relative prime thenl(p + q) is equal to

(
, tan 3x-tanx3
(C) LIm equals (R) 4
x~o x5

' x+2sinx
(D) L1m -r======-----r==== (S) 5
x~o -J x 2 + 2 sin x + 1- -Jsin 2 x - x + 1
[Ans, (A) R; (B) S; (C) P; (D) Q]
[Sol.

(A)

1
1=4 Ans,

. 3+x I/X Lim


. >1(3+X)
--I . 2x
LIm-­
(B) I = Lim (- -) = e".... o x 3-x =e x....o x(3-x) = e2/3 ~ 2 + 3 = 5'Ans,
x~o 3-x (

(C)
v
zero (by expansion)

2x+xtanx+x 2)
= Lim (tanx-x).(tan =.!. x3=1 Ans.
x-e-O x3 x2 3
(D) rationalising gives

. (x+2sinx>[ ~(x2 +2sinx+l) +-Jsin 2 x-x+l ]

LIm 2

x-e-D (x +2sinx+l)-(sin 2x-x+l)

26

sin2x
2. Lim x+sin2~ =2' Lim 1+-~ = 2(1+2) =2
x~O x 2-sin2x+2smx+x x-e-O sin 2x 3

x---+2+1

39 Column-I Column-If
x+ 1 if x < 0
(A) I(x) = [ at x = 0 is (P) continuous
cos x if x ~ 0
(B) For every x E R the function (Q) differentiability

sin(1t[x -1t])

g(x)= l+[xf (R) discontinuous

where [x] denotes the greatest integer function is (S) non derivable
(C) h (x) =~{x}2 where {x} denotes fractional part function

for all x E I, is

f -­
- I r
X [nx if x :t: 1
(D) k(x)= [ at x = 1 is
e if x = 1
[Ans. (A) P, S; (B) P, Q; (C) R, S; (D) P, Q]
. cosh-O
[Sol. (A) f' (0) = LIm does not exist. Obviously f (0) = f (0-) = f (0+) = 1
h-tO h
Hence continuous and not derivable
(B) g (x) = 0 for all x, hence continuous and derivable
(C) as O:$; {f(x) } < 1, hence h (x) = ~{x}2 = {x} which is discontinuous hence non derivable all
xEI
I
·
L im [nx . log e
(D) x = LIm x x = e = f (1)
x-tl x~l
Hence k (x) is constant for all x > 0 hence continuous and differentiable at x = 1. ]

40 Column - I Column - II
2
(A) Number of points of discontinuity of f( x) = tan" x - sec x (p)

in (O,21t) is

(B) Number of points at which f(x) = sin" x-t tan" x +coC1 x (q) 2

is non-differentiable in (-1,1) is

(C) Number of points ofdiscountinuity of y=[sinx],xE[O,21t) (r) o


where [.] represents greatest integer function

(D) number of points where y =1(x-1)31+I(x-2tl+!x-3! is (s) 3


non-differentiable

27

7t 37t
(A) tan" x is discontinuous at x = 2'2

7t 37t
sec" x si discontinuous at x = 2'2

=> Number of discontinuities = 2

(B) Since f(x)=Sin-1x+tan-1x+cot-1x=Sin-1x+%

:. f(x) is differentiable in (-1,1)


=> number of points of non-differentiable = 0

7t

a O:5;x<­
2

7t

1 X=­
2 7t
(C) Y = [sinx] = .. Points of discontinuity are 2,7t
7t
a -<X:5;7t
2

-1 7t < x < 27t

a x = 27t (

(D)

28

EXERCISE l(C)

3x 2 +ax+a+3
If the function f (x) == 2 is continuous at x == - 2. Find the value of a + f (-2).
x +x-2
[Ans. 14]
Sol. Since the function is conti then
Y.F.l x = _ 2 == RHLl x = _ 2 == LHLl x = _ 2

f(-2) == !~~ f(x)

. 3x2+ax+a+3 (I5-a).c:
= 11m ; - - lorm
x--> -2 x 2 + X - 2 0

this limit will exist when

15 -a= 0

a= 15 ...(1)

( , 3x2+15x+I8
II m - - - : - - - ­

x-->-2 x 2 + x-2

, 3(x+2)(x+3)
_ Ilffl ----'-_-C..-'-------'­
- x-->-2 (x+2)(x-I)

= lim 3(x+3)

x--> -2 (x -1)

3
=- =-1
-3

hence f(-2) == - 1

f(x)

( 2 Suppose that f(x) == x 3


- 3x -4x + 12 and hex) = [ X~3
2 then
,x=3

find the value ofK that makes h continuous at x = 3

[Ans.5]

· x3-3x2-4x+I2
1tm
u1m (x-3)(x 2-4)
== n1m (2 3)
x - =5
Sol.
x-->3+ X - 3 x-->3+ (x - 3) x-->3+

hence Ik == 51
n
3 Determine the value of product of values of a & b so that f is continuous at x == "2' f(x)

29
13-c:~: if x<~

= a if
[ b(l-sinx)
{n_2x)2
if

[Ans. 2]

Sol. Y.F,\ n =a
x=­
2

, (1-Sin3 XJ '-
Iim 0 c.
rorm

x-{~r 3cos x '0

1t
put x> --h
2

,
= lim [1-Sin3(~-h)J = lim 1-C~S3hJ =I'('
1m
(l-cosh) (l+cos
2h+cosh)
=..!..
h-+O
3cos
2 (1t)
--h

h-+O 3sin 2 h h-+O 3(1- cosh) (1 + cosh) 2


2

RHLI n = lim b(l-sinx)


x~2 ,,->~ ( 1t - 2x )2

1t
put x= -+h
2

= lim b (1- cosh) =~


h->O 4 h2 8

since function is conti

V.FL2: =LIfLL=2: =RHLL_ n


2 2 2

1 b
a= - = ­
2 8

sin3x+Asin2x +Bsinx
4 If f(x) = x5 (x -::f:- 0) is cont. at x = O. Find the value of A + B + f(O)

3
[Ans.2]

Sol. Since the function is conti

VFlx=o = RHLlx=o = LHLlx=o

frO)
\
= limf(x)

x---+o

-I' (sin3x+Asin2x+Bsinx)
- 1m 5
x~o X

. (3COS3X + 2Acos2x + BCOSX).(3 + 2A + B)


= I1m 4 ' . form
x~o 5x 0

3 + 2A + B = 0 ...(1)

-I'
- 1m
(-9sin3x-4Asin2x-Bsinx).(Q~
3 ' lorm
)

x-->O 20x 0

' (-27 cos3x - 8Acos 2x - Bcos x)' (-27 - 8A - B)


=Itrn
X40 60x
2 ' 0 form

27+8A +B=O ...(2)

= lim(81sin3x + 16Asin2x + Bsin x);(Q form)

x-->o 120x 0

= lim (243COS3X+ 32A cos 2x + Bcos x)


X40 120

243+32A+B
f(O) = 120
...(3)

using (1) & (2)

!A=-4,B=S!

(
then

f(O) = 1

2(n;·2 X
xnf(x)+ h(x)+ 1 ' sin )
5 Let g (x) = Lim n ' x:;t 1 and g (l) = LImI I ~ ( 2 x )) be a continuous function at
n-e-eo 2x +3x+3 X~ n sec n;'
x = 1, find the value of 4 g (1) + 2 f(l) - h (1). Assume that f(x) and h (x) are continuous at x = I,
[Ans. 5]

. xnf(x)+h(x)+1
Sol. g(x)= I1m---"---C----"---C-­ since g is conti
X400 2x n+3x+3

& g(l) = 2

31
h(1) = 1

f(1)=4
arg = 1 so arg - 1 therefore
=> 4g(1) + 2f(1) - h(1)

\ = 4.2 + 2.4 - 11

\
\
\ = 8 + 8 - 11 = 5 ADS

_ lim (l-cosn2
X
)(l+cosn2
X
) cos(n2'\)
\\
- HI 1-cos(n2X ) \

g(l) = 2 ...( 1 ) . r 1 \

" xnf(x)+h(x)+l 1
(
x = I1m
g() n+3x+3 ;x =t­
X--><XJ 2x

h(x)+l
O<x<l
3(x + 1)
g(x) = 2 x =1 using (1)
f(x)
x> 1
2

o ; O<x<l
" t X--><XJ
H In lim x" = 1 ; x =1
{00; xc- l
(
whenx< 1 thenxn=O

g(x) = h(x)+l

3(x + 1)

f(x)
g(x) = -2­

32
.(2 + cosx 3)
6 The function f(x) = 3' - -4 is not defined at x = O.
x SInX x
Let L be the value of the function at x = °so that it is continuous at x = 0, then find the value of L:'
[Ans.60]
Sol. Since the function is conti. at x = °
then
Y.F.l x= 0 = LHLlx=o = RHLlx=o

f(x)= (
2 + COSX

­
3)
\ x 3sinx x''

L = f(O) =limf(x)
x-->o

_ lim[2X+XCOSX-3Sinx]

- x-->o sin
x4 ( - x)
-.X

,r'

= lim("2X+XCOsx-3SinX]
S
x-->o X )

(x x" J ( x J]
2 3 S

= hm-
. 1s [ 2x+x 1 - - + -.... -3 x - - +X- ....
HO x 2! 4! 3! 5!

5 3 s
x' x -3x+~--
. 1 [ 2x+x--+-...
=hm- 3x 3x ....]
s
HO x 2! 4! 3! 5!

= lim
HO
x: [(~_2J+
x 4! 5!
..]
(
1 3 1
=--~=- :. L-I = 60
4 120 60

7 Let f (x) = x3 - x2 - 3x - 1 and h (x) = ~~:~ where h is a rational function such that

(a) it is continuous every where except when x = - 1, (b) Lim h(x) = 00 and (c) Lim h( x) =~ .
x--'>oo x--,>-l 2
If Lim (3h(x)+f(x) - 2g(x)) = _£. where p and q are coprimes then find the value of p+q.
x--'>o q
[Ans. 43]

33
Sol. hex) = f(x)
g(x)
(.;};n g (I)

h(x) =
(x + l)a ...(2yh x->O
3
) = lim (3. [X _ X- 3x
(x+1)4
2
-1) +(x 3 - x2-3x-1)-2(4(x+1» :
I
'.' x = - 1, function will = 3(-1) -1-8
4

_ 39 _ P
disconti; and for --­ --­ .. p+q =43 Ans
4 q

x = - 1 hex) = 00

Now lim hex) =..!..


X-->-) 2

2-2x-1)(x+1)
lim (x =..!..
x-->-l (x + l)a 2

2 1
-=­
a 2

put in (2)

'x3 _x 2 -3x-1
h(x)=---­
(x+1)4
(
g(x) = 4(x + 1)

8 Evaluate:
x 6000 -(sin ooo
Lim --~----'-::::-::-::­
xt
x-s-O 100'x 2 '(sinx)600o .
[Ans. 10]

[Sol. Put m = 6000

m (si )ffi . x
1001= Lim x -,-. SlOX = Lim as L1m ( - -) m =1:
x--+O x 2 '(sin x)'" x--+o r'C:=::::===s:-:;----=' [ x--+O sin x

Using L'Hospital's Rule

34
. )ill

-D smx
(

1001= Lim x

x~o 2x

now let y = ( Si: x ) ill ; dy = m(SinX)ill-l[xcoSX-Sinx]


dx x x2

. 6000·cosx '(x -tanx)

1001 = - L im 3 = 1000

x~o 2x

using expansion of tan x re get ~


. . L' . f(3+h 2 ) - f((3-h
2
)
9 Iff(x) IS derivable at x = 3 & f '(3) = 2, then find the value of imit 2
h~O 2h

[Ans.2]

2)+2hf'(3-h 2
· f(3+h2)-f(3h2) L' 2hf'(3+h )
Sol. L 1m 2
= 1m-----'-----'---....:....-----'­
h~O 2h x~o 4h

. f'(3+h 2)+f'(3-h2)
= lim by L'hospital
x~o 2

= f'(3)+f'(3) =~=2
2 2

10 Let f (x) [3 + 4 sin x] (where [ ] denotes the greatest integer function). If sum of all the values of
=
kn
in [n, 2n] where f(x) fails to be differentiable, is 2' then find the value ofk.
[Ans.24]
Sol.
y

rex) = [3 + 4 sin x]
= + [4 sin x]
sum of all x = 1t + (n - a) + (1t + ~) + (1t + y) + (2n - y) + (2n - ~) + (2n - a)
= 12n
:. k = 24
II The number of points at which the function f(x) = max. {a - x, a + x, b}, - co < x < co, 0 < a < b cannot
be differentiable is
[Ans.2]
Sol. /'"
'bo
I;
A

Ans. Two points

(
12 Let I, g and h are differentiable functions. If 1(0) = 1 ; g (0) = 2 ; h (0) = 3 and the derivatives oftheir
pair wise products at x = 0 are
(f g)'(O) = 6; (g h)'(O) = 4 and (h f)'(O) = 5
then compute the value of(fgh)'(O). [Ans. 16]
(fi )'h+( h)'f +(hf)'
[Sol. (fgh)'(O) = f g h' + g h f' + h f g' = g g g
2
(fg)'(O) 'h(O) + (gh)'(O)'f(O) + (hf)' (0) .g(O) 6·3 + 4·1 + 5 ·2
.. (fgh)' (0) = 2 2 = 16 Ans.

13 Let f(x) = x + _1 I I_ ......... co . Compute the value of 1(10)'f' (10).


2x+ 2x+ 2x+

1
Sol. f(x)=x+ - - - - - - , - - - - =x+ - - ­
x + x +--------:--­ x i-f'(x)
2x+---­
2x +......... co
(
hence f(x) ­ x = x +f(x)
.. f2(x) - x2 = 1
differentiating w.r.t. x
2f(x)'f'(x)-2x=0 or f(x)'f'(x)=x
hence f(10)' f' (10) = 10 ]

2
.­ 3 'd y . . p .
14 If the value ofthe expression Y - 2 for the ellipse 3x 2 + 4y2 = 12, IS -­ where p and q are copnmes
dx q
then
the value of p + q [Ans. 13]
[Sol. Differentiating implicity we have

36

3x
6x + 8yy' = 0 and hence y' = - 4y; 4[yy" + (y')2] = - 3

differentiating again and substitute for y' we have

9x 2
3 + 4(y')2 + 4yy" = 0 and hence 3 + 4y2 + 4yy" = 0

2
9x 2 d y
multiplying by y2, 3y2+ - +4 y3- =0
4 dx 2

but 3x2 + 4 y2 = 12 and hence y3 ylll = ­ 2.4 = - Pq at every point on the ellipse

15 If f: R -+ R is a function such that f(x) = x3 + x2f'(1) + xf"(2) + f"'(3) for all x E R, then find the
value of f(2) - f(1) + f(O). [Ans.O]
Sol Given that I{x) = x3 +x21'(1) = 1+ 1'(1)+xI"{2)+ 1"'(3) ...(1)
Putting x = 0 and .e = 1 in (1), we get
I{O) =1"'(3) and 1(1) =1+ 1'{I)+ 1"(2)+ 1"'(3)
.. 1(1)- 1(0)=1+ 1'(1) + 1"(2) ...(2)
Differentiating both sides of (1) w.r.t. x, we get
I'{x) = 3x2 + 2xl'(1) + 1"(2) ...(3)

and 1"{x)=6x+21'{1) ...(4)

also I"'{x) = 6 ...(5)

Putting x = 1,2,3 in (3),( 4),{5) respectively then

1'(1)=3+21'{1)+1"(2)
or 1'{I) = 1"(2) =-3 ...(6)
(
=> 1"(2) = 12+21'(1) or 21'(1)-1"(2) = -12 ...(7)

and 1"'(3)=6 ...(8)

Solving (6) and (7), we get /'(1) = -5 and 1"(2) = 2


Hence 1(1)- 1(0)=1-5+2=-2 ...(9)

Also from (1), 1(2)=8+4/,{1)+2/"{2)+ /'''(3)


..
=8-20+4+6 = -2 ...(10)
Hence from (9) and (10), we get 1(2) = 1(1) - I{O).
.. f(2) - f(1) + f(O) = 0

37
2
d
16 Let y = x sin kx. Find the sum of possible values ofk for which the differential equation - { + y = 2k
dx
cos kx holds true for all x E R. [Ans. 0]
[Sol. y = x sin kx ....(1)
Y1 = kx cos kx + sin kx
Y2 = k[cos kx - kx sin kx] + k cos kx
2
d y_ 2 .
2 - 2k cos kx- k x SIn kx....(2)
dx

d2y d2 y

now given -2 + Y = 2k cos kx (substituting the values ofy and - 2 )

~ ~
hence, 2k cos kx - k 2x sin kx + x sin kx = 2k cos kx

x sin kx[l - k2] = 0, Hence k = 1, - 1 or 0

17 The functionf: R ~ R satisfies j'{x/) . f"(x) = f'(x) . f'(x 2 ) for all real x. Given thatf(l) = 1 and
f"'(1) = 8, compute the value of 1'(1) +1"(1). [Ans.6]
[Sol. Given,j(x2) . I"(x) = I'(x) . f'(x 2)
Put x = 1 in the given relation
f(1)· f " (1) = (f'(I)f (
let f'(1) = a and f"(1) = b
.. bc=a2 .•.• (I) (asf(1)=I)
to find (a + b) = ?
differentiating the given relation
we get f(x 2) • f"'(x) + f"(x) . 2x f' (x2 ) = f' (x) . f"(x 2) 2x + f' (x2 ) • f"(x)
put x= 1
8 + 2ba = 2ab + ab => ab = 8 ....(2)
from (1) and (2)
a3 = 8 => a = 2 and b = 4
=> a + b = 2 + 4 = 6 Ans. ]

18 A polynomial function f(x) is such that f(2x) = f' (x)f" (x), then find the value off(3)

[Ans. 12]

[Sol. Suppose degree of f (x) = n.


Then deg f' = n - 1 and deg f" = n - 2, so n := n - 1 + n - 2.
Hencen= 3.
So put f (x) = ax3 + bx 2 + ex + d. (a *- 0). Now using f (2x) := f' (x) . f" (x) (
Then we have 8ax 3 + 4bx 2 + 2cx + d = (3ax 2 + 2bx + c)(6ax + 2b)
= 18a2x3 + l Sabx-' + (6ac + 4b2)x + 2bc.

3 2
Comparing x , 18a = 8a (1) => a > 4/9

Comparing x2, l Sab > 4b (2) => b > 0

Comparing x, 2c:= 6ac + 4b 2 (3) => c:= 0

Comparing constant term, a = 2bc => d = 0

3
f'(x)> 94x :. f(3) = 12

19 Supposefand g are two functions such that/, g: R ~ R,j(x) = In (1 +.Jl +x 2 ) and

g (x) ~ In( x+.J1 + x' ) then find the value of x eg(x{f[~))' + g'(x) at x ~ 1. [Ans. 0]

38
[Sol. g (x) = In( x +~1 + x 2
)

1
g'(x) = --== ....(1)
~1+x2

x 1
now f'(x)= 1+~1+x2 . ~1+x2

f' (~1) = x+~1+x2


x 1 x
. ~. ~1+x2

also eg(x) = eln(X+~1+x2) = x + ~1 + x 2


x eg(x) = x(x +~1 + x 2 )

hence, #')·x· (f(±)} ~- b ....(2)

from (1) and (2), g'(x)+x#<) [{±)} ~O


x+ln( ~x2 +l-X) P
= - where p and q are coprimes then find the value of Ip 2 1.
2
20 If Lim 3 _ q
x~o x q
[Ans.35]
x
· ~X2 +1 - 1 ~x2 +1 x ._1 1 p
[Sol. Using L'Hospital's Rule, L 1m . - ~ 2
~'
x~o 3x 2 ' 6x - vx +1 6x 6 q
'.'
~,
/

39
(

EXERCISE 2(A)

. 1{fX-7JX
1. The value of LIm is
HI ~_~

9 5 9 45
(A) 13 (B) 91 (C) 91 (0) 91

I I
. x 13-x 7
Sol. hm
x~l
I
_
I
_
x 5 -x 3

Apply L'Hospital rule


I I
1 -1+­ 1 -1+­
-x 13 --x 7
lim 13 7
I I
x~1 1 --I 1 --I
5
-x --x 3
5 3
l

1 1
r
lim 13 -7 = 45
ADS.
HI 1 1 91
5 3

2. The value of LIm K=I


.. [IX
J - I 00
x-> I =:.....::...--=---­
is
k

x-I
(A) 4950 (B) 5050 (C) 5150 (0) 5151

100
~>k -100
Sol. Lim-,,-k=--,-I--­
x~1 x-I

. x +1 . x 2 -1 . x 3 -1 . x 100 -1
=:> Ltm-- + Llm--+ Llm-- .....L l m - ­
x~1 x -1 x~1 x -1 x~1 x -1 x-e-l x -1
(
Applying L'Hospital rule
2 3
=:> Lim 1+ Lim 2x + Lim 3x + Lim 4x + Lim 100x 9
x~1 x~1 x~l x-e-l x~1

=1+2+3+4 100

= 100(100+1) =50 x 101(1+2+3 n=n(n+1)1


2 2 )
= 5050

3. The value of Lim


x~l
(-p­--q-) p, q N
1-xP 1-x Q
E
,
is

(A) p-q (B) P + q (C) q - p (0) None of these


2 2 2

J..-::- tim
~-:3 'L
I-'?
0=){.P t.)
\-)('t­ 1
_, _ \ " .... l , 1\
(00 - 00 form)

Lim p-px 2 -q +qx P )

x-e-l [ l-xq-xP+x
p+q

Apply L'Hopital rule

q-I p-I
Lim[ - pqx + qpx )
x~1 qx'' l_pXP I +(p+q)X(p+q-l)

q-2+qp(p
Lim[ -pq(q-l)x l)x P 2 )
x~1 q(q q
l)x 2 p(p l)XP 2+(p+q)(p+q_l)x(p+q-2)

pq(p-q) p-q
=> =~-
2pq 2

4. The value of Lim 1 + ~


X-->¥ 1- 2 cos'' X is
1 1
(C) - ­ (0) - ­
2 3
I

Sol. Lim 1+ (tanx)3


x~311/41- 2cos 2
x

I
Lim 1+(tanx)3

x~311/4 -cos2x

Applying L'Hospital rule

1 -2

-(tan x) 3 x sec2 x

Lim~3_----
X~ 311 + 2sin 2x

4 2
=> 6-1 ( -sec"41tJ2

-1 1
-x2=-­
6 3

5. The value of Limo [In (1


X~
+ sin-x). cot(ln 2 (1 + x))] is
(A) 0 (B) 1 (C)-1 (0) 1/2

Sol.

=> LimRn(l -r sirr' x) ;


HO tan [Rn (l + x)]

" (sinx/x)2 1
L
=> im tan(Rn 2 (l + x)) (
/1 HO .~)
Rn 2(1 + x) .

2(1
Rn + x)
x

L" 1

=> x~~ Rn(1 + x) Rn(1 + x) = 1

x x

" 2.,[;. + 3x1/ 3 + 5x ll 5


6 The value of Lim is
X~ .J3x-2+(2x-3)113

1 2
(A)l (B) J3 (C) J3 (D)J3
1 1 I

2x 2+3x 3 +5x 5
Sol. Lim----:-----,­
X~OO l l
(3x - 2)2 + (3x - 3)3
divide numerator & denominator by x l /2

7 The value of Lim sec4x-sec2x is


x~ sec3x -secx
(A) 1 (B) 2/3 (C) 3/2 (D2
" sec4x-sec2x
Sol. Ll m - - - - ­
x~o sec3x-secx

3
1
Lim cos4x cos2x
x~o 1 1
-----
cos3x cosx

. cos2x-cos4x cosx.cos3x
L1m x----­
x~o cosx-cos3x cos4x.cos2x

. +2sin3x.sinx
LI m - - - - ­
x~o 2sin 2x.sin x

. sin3x 2x 3x
LIm--X-X­
x~o sin 2x 3x 2x

Lim 3x =i
x~o 2x 2

tan x - sinx
8 The sum of an infinite geometric series whose first term is the limit of the function f(x) = ~--­
sin' x (
1-~
as x -)0 °and whose common ratio is the limit of the function g(x) = ( -1)2 as x -)0 1, is
cos x
(A) 1/3 (B) 1/4 (C) 1/2 (D) 2/3
Sol. Let first term of on infinite G.P. is a & common ratio of infinite G.P. is r
. tanx-sinx (0 )
given a = LIm
x~o
. 3
sm x
-form
°
Apply L'Hospital rule
. sec2x-cosx
L 1m -----=---­

x~o 3sin 2 xcosx

" l-cos3 x
L 1m~~~~~

x~o 3sin 2 xcos3x

·
L rm 1-cos3x x L"Im-­1
x~o 3sin 2 x cos 2x x~o cos x
3x (
· 4 l-cos 1
L1m-x x
x~o 4 3sin 2 x cos2 x

. 4(l-cos3x)
L 1m --"-~~-----;;:-'­

x~o 3(sin2x)2

Apply L'Hospital rule


. 4 (+3cos2xsinx)
L 1m-X~-------"':"­
x~o 3 2(sin 2x) x 2cos 2x

" cos/ xsinx 1


a= L1m
x~o (2sinxcosx)cos2x
=2

. I-JX
r = LHfl ~-----,,---::­
x~l (cos" X)2

1_1_
Lim 2..h
x---71 2( cos - Ix) x ­ r:-?"
1
2
"\I1-x

2
" 1x.J1- x
L 1m ------,~-- 1
x---71 4..h(cos- x)

put x = cos t

a
so sum of infinite G.P. is J00-­
1-r

1
=--=-x-=­
"2 1 4 2
1'-! 2 3 3
4

9 The value of Lim (x - I n cosh x) is


X---7CX)

(A)O (B) In 2 (C) 1 (D) 21n 2

eX +e- x
Sol. Lim(x ­ fn(cos nx)) coshx = 2
X---700
\1\IV)
~ 'J'\.-rl fX)

\ \ Vv)

~~oO

lim lx - [Rn(e
2x
+ 1)- fn2e X
]J
X---7CX)

--- \ \r-- L

lim [x -.€ (e 2x + 1)- .€n2- x]


X---7CX)

5
(e2X 1) 2x
-£02 - lim £0 .e
X~OO x

1+­
° e2x
-£02- lim . 0 - 1I­:
x~C()

e2 x

2 2 2 2
°
The value of LIm 8 [ x x x x
-8 1- cos­ - cos ­ + cos­ cos-
]
is
10 x~ x 2 4 2 4
(A) 1/16 (B) 1/32 (C) 1/64 (D) 1/8

Sol.
x~o x
8 [( x
2
x (
4
x
Lim-8 l1-cos-'j -COS-l1-cOS-j
2
2
1 2 2
1]
8 [( x 1( x
Lim=-8 l1-COS-jl1-COS-j
2 2
'J ]
x~o x 2 4

Lim = -
x~o
8 [(
x
2 x
8 1-l2cos - 4
2
-1j 1( x
l1- COS-)
4
2
1]
8 [ 2-2cos 2x- l 1( - C O S
Lim=-
x~o x 8 4
x -j
4
2 2
1]
2
8X2[(
°
Llm=-
2 x x
l1-cos -jl1-COS-)
1( 2
1]
x~o8 x 4 4

X2]
. 16[.2X2
L Im=g SIn - o2
2sIn -
x~o x 4 8

r.t1m = -32[ SIn 2x2


-SIn 2X2]
°
-,. °
(
x~o 8x 4 8

, ( 1
j
l
SiO X2
Lim=_1 __4_
x~o 32 (x 2 1
l4j

Lim J2 - cos e- sin e


11 The value- of ,e~.rr4 (4-1t
e)2 is

1 1 1 1
(A) 2J2 (B) 4J2 (C) sJ2 (D) 16J2

6
.J2 -cos8-sin8
Sol. lim (%fonn )
e--7~ (48 _1t)2
4

Apply L'Hospital rule

sin8-cos8
lim
e--7~ 2(48 -1t).4 (%fonn )
4

Again apply L'Hospital rule

1 1
11/
lim cos8+sin8 = .J2+ .J2 = .J2 =_1_

~
e--7~ 8.4 32 32 16.J2
4
-J\ ~y / ~
Lim TC~sx -I
)vol .~~
r',{
12 The value of X~
2 x ( x- 21t) is ?-- .'1)1
: ~

Rn2 2Rn2 3Rn2 4Rn2


(A)- (B)- (C)- (D)­
1t • 1t 1t 1t

cOSX

Sol. pm 2- -)1

e ~2 X x-­1t
2

lim ---4---~

tv
~~2cOSX

_.
2 ..

(
. 1_2cOSX
hm--­ 'J-t1\

_.
1t 2 1t
X--7
2x L'

( ~

lim ------"''-­
1t
X--7­
2

. (In(1+x)-ln2)(3.4 x ­ 1-3x)
13 The value of LIm 1 1 is
X--71 [(7+x)3 -(1+3x)2].sin(x-l)

3 4 3 4 9 4 9 4
(A) -Rn- (B) --Rn- (C) -Rn­ (D) --Rn­
4 e 4 e "4 e 4 e

here (I)CO

7
=> 2 .!..3·l
Rn4 1
- ]
1-9 =>~
~

12

/l'7'\/
L The value of Lim
n400
t ((r+1)Sin~-rSin~)
r=2 r+1 r
is

(A) n - 2--- i - ~ t 2n
(B) (C) - '2. (D) ~4 -1
. 2­

(
Sol. If R = lim t((r+ l)Sin~-rSin~)
n400 r=2 r +1 r
c-;::."
o
-t. = l'im (3'sm--
n 2' n 4'
Slll-+ n 3'
Slll-- n 4~i+nS1ll
n Sr[)'
Slll-+ . -n - (~s'
n- AI
~AA
n400 3 2 4 3 5
0
1\
n- .r

4 .;
. r::'

+(TI+l&1-TI~)
R= lim((n+1)~2Sin~)
H<O CY 2 1\(V\

e = liml(TI+l ~)< n-i-l

n4<o
~~~-2J
n+1

(
R = lim(n-2)= 3.14 - 2
n400

= 1.14
so {R} = R- [.e]
.14-1=.14orn-3

4
(3x + 2x ) sin
2
t+ 1
3
x 1 +5 .

15 The value of x-+--co


Lim
.-
I x 13 + 1 X 1
2 + I x I +1 IS
(A) 2 (B) -2 (C) 1 (D) -1

Sol.

. 3X3+2x _(X)3+ 5

I1m + ~----=---'--'--=---
x-t-oo - (xi - (xi - (x) + 1 - (X)3 + (xi - (x) + 1

2 5
3+- 2
-1+­ 3
lim x + x

X-7-00 _1_~ __1 __ ~ _1_~ __


1 +~

3
x x2 x X x 2 x
3

-3

lim -+1=-2
X-7-00 1

3
Ii (x + 27}en(x - 2)
16 The value of x~ (x 2 -9) is
(A) 1 (B) 3 (C) 6 (D) 9
" (x3+27)fn(x-2)
Sol. I 1m -"---~----'---"":""
2
x-73 (x -9)
" (x-3)(x2+9-6x)fn(1+(x-2))
I 1m .
x-73 (x + 3)(x - 3)
Iimx' +9-6x = 18 - 9 = 9
X-73

. 27x _9 x _3 x +1
17 The value of ~~ ..fi - ";1 + cos x is

(D) J2(fn3)2

Sol.

" 212(27 X _9 x _3 x +1)


hm ----'--------'-­
X-70 2-1-cosx

412(27 X - 9 x-3 x + 1)
. ~---'--~--"":""
hm
( X-70 2 x
X(3 X
lim412[9 -l)-l(3x -l)J
X-70 x2

X X X x-1
Iim412[(9 -1) (3 -l)J 9 -IJ 3
X-70 x2
=> lim 412[ - ­ x lim-
X-70 x
­
X-70 X
=> 812(fn3)2

:::;> 412(fn9) (fn3)

9
(1+eX)IIX ]IIX I'S
18 The value of Lim
x-tO [
1 1
(A) Je (B)­
Je
(C) -
e
(D) e

I/X
. (1 + x)1!x
Sol. LIm (1 )00
x---+o [ e ]

Lim (l+XY/x- e 1
L=e H o .­
e X

In(l+x) -I
Lim e[e x-I] (fn(l+X)-x)
Act.: HO e( f n(1+ x) - x) . X2

Lim
HO
t-et+l/e
(e t - li / e
=
1
2 c
(taking commone)

. rr
-vn2+n-l
:': J2~-1
19 The value of LIm is
n---+oo ( n

1 1
(A) Je (B)­
Je
(C) -
e
(D) e

L im[,J;;2;;.+n]( 2,J;;2;;.-I)
n..­ n
Sol. e

L" [(n 2+n)-(l+n)2] (2,J;;2;;.-1)


e x~ n{,J;;2;;.+(l+n)j
(

(-n-l)(2~-I) [ki 2
en
Ll~ { rt:
n vn-+n+(l+n)
j ta ng n as common]

T
20 The value of Lim (tan
x---+I
)tan
7tX
4
is

(A) e (C) Je (D) e- 1/ 2

10

]
tan~
° 1tX 2
Sol. LIm tan-
[
X-41 4
xx ) Ian­
Lim ( Ian--l xx
eHI4 2

. (xx x)
Limsm 4-4" .tan~
e x-+I cos~. cos~ 2

4 4

sin~(x-l)

Lim 4 .tan~.~(x-l)

e x~1 ~(x-I)cos~cos~x 2 4

4 4 4

e Lim~(X-l)cost(~~)
HI 2 2 2x
_
- e-1 -- ( e -1)

eX -cos2x -x
21 Given I(x) = x2 for x E R- {O}

1
f({x}) for n<x<n+­
2
where {x}denotes
1 E I fractional part
g(x) = f(l- {x} ) for n + - < x < n + 1 , n { function
2 -
5
otherwise
2
then g (x) is
(A) discontinuous at all integral values ofx only
(B) continuous everywhere except for x = 0
1
(C) discontinuous at x = n +"2 ; n E I and at some x E I

(D*) continuous everywhere

Li . e h -cos2h - h

[Sol. Limit g(n + h) = irmt ------:::--­


h--+O h--+O h2

o ° el-{n-h} -cos2(1-{n-h})-(1-{n-h})
\l~cit g(n - h) = (1- {n _ h})2

LO e h -cos2h-h 5
= h~ h2 ({n-h}={-h}=l-h)= 2

5
g (n) = "2' Hence g(x) is continuous at \f x E I.

Hence g (x) is continuous \f x E R]

11
22 Let I(x) = ~g~, where g and hare cotinuous functions on the open interval (a, b). Which of the
following statements is true for a < x < b?
(A)I is continuous at all x for which x is not zero.
(B) I is continuous at all x for which g (x) = 0
(c) I is continuous at all x for which g (x) is not equal to zero.
(D*) I is continuous at all x for which h (x) is not equal to zero.
[Hint: By theorem, if g and h are continuous functions on the open interval (a, b), then g/h is also continuous
at all x in the open interval (a, b) where h (x) is not equal to zero. ]

2cosx-sin2x e- coSX_ l
23 I(x) = (n _ 2x)2 g (x) = -8-x---4-n-
h (x) = f(x) for x < nl2
= g (x) for x > nl2
then which of the following holds?
(A) h is continuous at x = nl2 (B*) h has an irremovable discontinuity at x = nl2

(c) h has a removable discontinuity at x = nl2 (


"

2cos x -sin 2x n
x<-

(x)~ [
(n-2x)2 2
[Sol. h
e-cosx -1 n
x>-
8x-4n 2
LHL at x = nl2
Li 2sinh-sin2h LO 2sinh(1-cosh)
rm = un = 0
h~O 4h 2 h~O 4h 2

° esinh -1 ° e sinh -1 sin h 1


RHL: Lim = Lim ._- = -
h~O ((nI2) + h)- 4n h~O 8h sin h 8
=> h (x) is discontinuous at x = n12.
Irremovable discontinuity at x = n12.

f( ~l 0 and g ( r) ~ ~ =>
(

x-eX +cos2x
24 Iff(x) = x2 ' x*-O is continuous at x = 0, then

5
(A) f(O) = ­ (B) [f(O)] = - 2 (C) {f(O)} =-0.5 (D*) [f(O)] . {f(O)} = -1.5
2
where [x] and {x} denotes greatest integer and fractional part function

[Hint: Lim x-ex +1-~1-COS2X) =_..!.- -2=-~. hence for continuity f (0) = - ­
5
x~o x 2 2' 2

[f(O)] = - 3; {f(O)} = {-25} = 2;1 hence [f(O)] {f(O)} = - 23= - 1.5 ]


12
x+b, x <0
25 The function g (x) = [ can be made differentiable at x = o.

cosx, x 2::0

(A) ifb is equal to zero (B) ifb is not equal to zero


(C) ifb takes any real value (D*) for no value ofb
. cosh-1
[Hint: g' (0+) = LIm =0
h-tO h
-h+b-1
g , (0-) = Lim h for existence of limit b = 1 thus g , (0-) = 1
h-tO ­
Hence g can not be made differentiable for any value of b.]

26 If f(x) = sin-I(sinx) ; XER then fis


(A) continuous and differentiable for all x
n
(B *) continuous for all x but not differentiable for all x = (2k + 1)"2 ' k E I

(C) neither continuous nor differentiable for x = (2k - 1) ~ ; k E I

, I (D) neither continuous nor differentiable for x E R - [-1,1]


f,

't_ n n
x-2kn for 2kn-- :':;;x:':;;2kn+­
2 2
[Hint:
[ n 3n
(2k+1)n-x for 2kn+-:':;; x<2kn+­
2 2

x{x}+l 0:.:;; x <1


27 Consider the function f(x) = [ { where {x} denotes the fractional part function.
2- x} 1:':;;x:.:;;2
Which one ofthe following statements is NOT correct?
(A) Limf(x) exists
x-e-l

----*
(B) f(O) f(2)

(C*) f (x) is continuous in [0, 2] (D) f(x) is differentiable in [0, 1)

[Sol. f (1+) = f(1-) = f(1) = 2

f(O)=l, f(2)=2

f(2-) = 1; f(2)=2 => f is not continuous at x = 2

(
28 Which one of the following functions is continuous everywhere in its domain but has atleast one point
where it is not differentiable?
[x ]
(A*)f(x)=xI/3 (B)f(x)=- (C) f (x) = e- X (D) f(x) = tan x
x

[Hint: x 1/3 is not differentiable at x = 0 ]

29 Let f(x) = then

jf x<2
x-'\hx-2
(A) f (2) = 8 => f is continuous at x = 2 (B) f (2) = 16 => f is continuous at x = 2
13
(C*) f (2-) *- f (2+) => f is discontinuous (D) f has a removable discontinuity at x = 2
[Hint: f(2+) = 8 ; f(2-) = 16

30 The graph of function f contains the point P (1,2) and Q(s, r). The equation ofthe secant line through

P and Q is y ~ [s': :sl- 3 Jx-I - s. The value of f' (1), is


(A) 2 (B) 3 (C*) 4 (D) non existent

[Sol. I By definitionJ'(l) is the limit ofthe slope of the secant line when s --)- 1. [29-01-2006,12&13]

. S2 +2s-3
Thus J'(l) = L l m - - ­
s-l
~Q(S")

s---tl

= Lim (s-1)(s+3)
s---tl s-l
P(1,2)
= Lim(s+3) =4
s-ol
=> G!)
~
II By substituting x = s into the equation ofthe secant line, and cancelling by s - 1 again, we get
y = s2 + 2s -1. This is f(s), and its derivative isf'(s) = 2s +}' so!' (1) = 4.]
(
Let [x] denote the integral part of x E R . g(x) = x - [x] . Let f(x) be any continuous function with f(O)
= f(1) then the function hex) = f(g(x)) :
(A) has finitely many discontinuities (B) is discontinuous at some x = c
(C*) is continuous on R (D) is a constant function.
[Sol. g(x) = x- [x] = {x}
f is continuous with f(O) = f(1) f(x)
hex) = f(g(x)) = f({x})
Let the graph of f is as shown in the figure
satisfying x
f(O) = f(1)
now h(O) = f({O}) = f(O) = f(1)
h(0.2) = f ({Oo2}) = f(0.2)
h(1.S) = f ({ 1.S}) = f(O.S) etc.
Hence the graph ofh(x) will be periodic graph as shown
=> h is continuous in R => C

~
X)
(
I I . ]
1-1-­
o 1 2 3 x

32 If f(x + y) = f(x) + feY) + I x I y + xy2, 'if x, Y E Rand!, (0) = 0, then


(A)f need not be differentiable at every non zero x
(B*)f is differentiable for all x E R
(C)f is twice differentiable at x = 0
(D) none
2
[Hint: f'(x) = Lim f(x+h)-f(x) = Lim f(h)+lxlh+xh wherex=handy=x
h---tO h h---tO h

. (f(h)-f(O) )

'.: f(O) = 0; hence f' (x) = ~~1IJ h + I x I+xh

f'(x)=f'(O)+lxl=lxl ]

14

33 Let j be a differentiable function on the open interval (a, b). Which of the following statements must
be true?
I. j is continuous on the closed interval [a, b]
II. j is bounded on the open interval (a, b)
III. If a<al<bl<b, andj(al)<O<j(b l), then there is a number c such that al<c< b, andj(c)=O
(A) I and II only (B) I and III only (C) II and III only (D*) only III
[Sol. I and II are false. The function f(x) = l!x, 0 < x <1, is a counter example.
Statement III is true. Apply the intermediate value theorem to f on the closed interval [a., b J ]

-rt7h
34 Let f(x) = max. {I x 2
-21 x 11,1 x I} and g (x) = min. {! 2
x -21 x 1\,1 x I} then
(A) both f(x) and g (x) are non differentiable at 5 points.

(B*) f (x) is not differentiable at 5 points whether g (x) is non differentiable at 7 points.

(C) number of points of non differentibility for f(x) and g (x) are 7 and 5 respectively.
(D) both f (x) and g (x) are non differentiable at 3 and 5 points respectively.
[(x) y
[Hint: j(x) is non differentiable at x = a, ~, 0, y, 8

and g (x) is non differentialbe at x = a, ~, 0, - 2,2 => (B)]

3x2_4--h+l forx<1

35 Let g(x) = [

ax+ b for x z 1
If g (x) is the continuous and differentiable for all numbers in its domain then
(A) a = b = 4 (B) a = b =-4

(C*) a = 4 and b = - 4 (D) a = - 4 and b = 4

3x2_4--h+l for x c l

[Sol. g(x)= [

ax+ b for x z 1

for differentiability at x = 1, g'(1+) = g'(1-I)


4
,- =>
a= 6x- a=6-2=4
( 2-vx
for continuity at x = 1, g(1+) = g(1-)
a+b=3-4+1 a+b=O b=-4
a = 4, and b = - 4 Ans. ]

sin!n[x]
36 Let [x] be the greatest integer function and f(x) = 4 . Then which one of the following does not
[x]
hold good?

(A*) not continuous at any point (B) continuous at 3/2

(C) discontinuous at 2 (D) differentiable at 4/3


. n[x]
S1O-­
[Hint: g(x) = 4
[x]
obv. cont. at x = 3/2
15
n I
at x = 2 f(2-) = sin"4 = .fi }
. n
sm­
f(2) = _ 2 =!

2 2

Hence discontinuous at x = 2 ]

37 Number of points where the function f (x) = (x2 - I) I x2 - X- 2 I + sine I x I) is not differentiable, is
(A) 0 (B) I (C*) 2 (D) 3
[Hint: not derivable at x = 0 and 2

x
38 Consider function f: R - {-I, I} -+ R. f(x) = 1-1 x I. Then the incorrect statement is

(A) it is continuous at the origin. (B*) it is not derivable at the origin.


(C) the range of the function is R. (D) fis continuous and derivable in its domain

l
- x- if x 2':0, x:;t:I y
[Hint: f(x) = I-x
_x_ ifx<O,x:;t:-I
I+x
~I
_.- ••-•••_ ••
_.~.~

i
,/1 .
•• •••••_ ••-•• _ •••_.... -r--vt-•••••••_ •••_ •••••
~

!
(
! !
I 2 if x > 0, x :;t: I ~
and f' (x) =
(I-X) ············--·--r --·-··..=-i ----------.------.l"(
I 2 ifx<O,x:;t:-I
[l-i x]

39 Given f (x) = b ([xF + [xj) + 1 for x 2': -1


= Sin (n (x+a)) for x <-1
where [x] denotes the integral part of x, then for what values of a, b the function is continuous at
x=-I?
(A *) a = 2n + (3/2); b E R; n E I (B) a = 4n + 2; b E R; n E I
(C) a = 4n + (3/2); b E R+ ; n E I (D) a = 4n + 1; b E R+ ; n E I
[Hint: f(-I)=b(I-I)+I=I
and Lim f (-1 + h) = 1
h~O
(
Limf (-I-h) = sin ((-1 + h + a)n) = - sin na
h~O

3n 3
for continuity sin na = - 1 = sin ( 2nn + 3;) => na = 2nn +- => a= 2n+­

2 2

3
hence a = 2n + -, n E I and b E R]

I!n (e x2
+ 2£)
40 If f(x) = ~ is continuous at x = 0, then f(O) must be equal to:
tan x
(A) 0 (B) I (D*) 2

en (ex2 +2f;.)
[Hint: f (0) = ~i~~t
~
16

x2

= Limit _1_ (e x2 + 2£ _ 1) = Limit (e - 1 + 2J = 2

x~O .f;. x~O .f;.

41 If f( x + y) =:: f(x) + fey) + c, for all real x and y and f(x) is continuous at x = 0 and f' (0) = 1 then f' (x)
equals to
(A) c (8) -1 (C) 0 (D*) 1

. , . f(O+h)-f(O)

[Hint : f (0) = hm 1 also f(0) = -c f '


h~O h
· f(x+h)-f(x) L' f(x)+f(h)+c-f(x)
(x) = L im = im --'.......:....----0.--:....-_ _'----'­
. h~O h h~O h

. f(h)-f(O)
= LIm = f'(O) = 1
h~O h

f' (x) = 1 ]

cosx sinx cosx (


42 Let f(x) = cos2x sin2x 2cos2x then f' ~) =
cos3x sin3x 3cos3x
(A) 0 (B) - 12 (C*) 4 (D) 12
[Hint: Differentiate column wise, where 8 1 = - 4; 8 2 = 0 and 8 3 = 8

43 People living at Mars, instead ofthe usual definition ofderivative D f(x), define a new kind ofderivative,
D*f(x) by the formula

. . f 2 (x + h) - f 2 (x)
D*f(x) = Lurut where f2 (x) means [f(x)]Z. Iff(x) = x lnx then
h-e-O h

n*f(x)!x=e has the value


(A) e (B) 2e (C*) 4e (D) 8e
[Hint: D*f(x) = 2f(x).f' (x)
D*(x lnx) = 2x lnx (1 + lnx)]

44
.
Consider f(x) =
[2 (sinx - sin
3x) 3x
+ Isinx - sin 1
2 (sinx - sin 3x) -Isinx - sin3xl
, x
j * -n2 for x E (0, n)

f(nI2) = 3 where [] denotes the greatest integer function then,

(A*) f is continuous & differentiable at x = n/2

(B) fis continuous but not differentiable at x = nl2


(C) f is neither continuous nor differentiable at x =:: nl2
(D) none of these
[Sol. In the immediate neighborhood of x = nl2 , sinx > sirr'x => lsinx - sirr'x] = sinx - sirr'x
3 3
_ [2(SinX -sin x)+sin x -sin x] _ 3sinx-3sin 3 x

Hence for x * nl2 , f (x) - 2(' . 3).


smx-sm x -smx+sm x
. 3 - 3
sinx-sin x
3

Hence f is continuous and diff. at x = nl2 ]

17
d 2y dy .
45 If Y =(A + Bx) enx + (m _1)-2 eX then -2 - 2m - + m-y IS equal to:
dx dx
(A *) e" (B) emx (C) e- mx (D) e(l - m) X
[Hint: y = (A + Bxje?" + (m - 1)--2 . eX
y . e- mx = (A + Bx) + (m _1)-2. e(l-x)x
e- mx. YI _ my + e-mx = B - (m _1)-1 . e--{m-l)x

e- mx. Y2 - YI e-mx. m - m[e-mx. YI


- Ye- mx. m] = e--{m-I)x

e- mx. Yl - m2Yl e-mx + my' e- mx = e--{m-l)x

Y2-2mYI -r my s e-Ans.]

46 L et f (x) be a polynomial function of second degree. If f (1) = f (-1) and a, b, c are in A.P., then f '(a),
f'(b) and f(c) are in
(A) G.P. (B) H.P. (C) AG.P. (D*) AP.
[Sol. Let f(x) = px? + qx + r
f (1) = f (-1) gives p + q + r = p - q + r

hence q = 0

Hence f(x) = px? + r

f'(x)=2px ....(1)
Given a, b, c are in A.P. ./
hence 2pa, 2pb, 2pc will also be in AP. (
or f" (a), f' (b), f' (c) will also be inAP. => (D)]

3-x 2 for x::;-1


47 The function f(x) = { 2 for -1 < x < 1 is
I-x for x ~1

(A) decreasing in (- 00, -1)


(B) differentiable at x = - 1
(C) continuous at x = 1 but discontinuous at x = - 1

(D*) continuous at x = - 1 but discontinuous at x = 1

[Hint: l-x ]

(
48 If x2 + y2 = R2 (R> 0) then k = y" where k in terms ofR alone is equal to
~(1 + y,2r
1 1 2 2
(A) ­ R2 (B*)- ­ (C) ­ (D)- R 2
R R
[Sol. 2x + 2yy' = 0
X
x + yy' = 0 => y'=- ­
y
....(1)
1 +Oyy" + (y')2 = 0
1 + (y,)2
y"=- _..:.:........:.­
y

18

y" 1+(y,)2 1 1
now k= ( 2\312
1+(y') ) - - y(1 + (y')2)12 = - y~1 + (y,)2 R

On the interval 1= [-2, 2], the function f(x) = (x+l) e


-[0+*1 (x e 0)
49
. {0 (x = 0)
then which one of the following does not hold good?

(A *) is continuous for all values of x E I

(B) is continuous for x E 1-(0)


(C) assumes all intermediate values from f(- 2) & f(2)
(D) has a maximum value equal to 3/e.
(x+l)e-2/x if x c- O 3/e

[Sol. f(x)= x-i I ifx<O


[
o if x=o 2
the graph of f (x) is L-----l-l
hence f can assume all values for f(- 2) to f(2)

50 The derivative of the function,

f(X)=cos-I{.Jh (2cosx - 3SinX)} + sin" { .Jh. (2cosx + 3SinX)} w.r.t. p atx= ~ is


3 5 10
(A) - (B) - (C*) - (D) 0
2 2 3
2 3 3

[Sol. Put cos ~ = .J13 ; sin ~ = .J13 ; tan ~ = '2

y = cos' { cos(x + ~) } + sirr' {cos(x - ~) }


7t
= cos" {cos(x + ~) + '2 - cos:' {cos( ~ - x)} (think !)

7t
=x+~+--~+x
2
7t
( y=2x+ - . z= ~1+x2
2 '
dy

now compute dz

2
x - x d (-I )
51 If f(x) = 2
x +4x
' then
x
d
f (x) at x = 2 is

(A) 2 (B) 3 (C) 4 (0*) 5


2
x -x
[Sol.-We have f(x) = 2 4
x + x

f (x) is not defined at x = 0, - 4

.. domain off= R - {O, - 4}

F or all x E domain of f, we have

19
x 2 -x x-I 5
f(x) = 2+4x
= - = 1- ­
x x+4 x+4

f(el(x)) = x
5 5

1 - e l(x)+4 =x or l-x= e l(x)+4

5
rl(x) = - - 4
I-x

d
dx f
(-I) 5
(x) = (1-x)2 ;
d (
dx f
-I)
(x) at x=2 =
5
(1-2)2 = 5 Ans.]

52 If f(x) = ~x+2.J2x-4 + ~x-2.J2x-4, then the value of 10 f' (102+)


(A) is - 1 (B) is 0 (C*) is 1 (D) does not exist
[Sol. f(x)= ~x+2-hx-4 + ~x-2,J2x-4

for ,Jx - 2 to exist x ~ 2 (


Also, ,Jx - 2 + -fi > 0 (always true, think! why?)
but .Jx - 2 - -fi ~ 0 only if x ~ 4
< 0 only if x < 4
now f (x) becomes
f(x) = ,Jx-2 + -fi - ,Jx-2 + -fi for 2 s x < 4
=,Jx-2 + -fi + .Jx-2 - -fi for x~4
f (x) = 2-fi, for 2 s x < 4
= 2.Jx-2, for a s x « co
fis continuous [2, 4) u [4, oo)(verifY)
f'(x)=O, 2~x<4

1 1 (
f' (102+) = .JI02-2 = 10
10 f' (102+) = 1

53 Suppose A = :~ for the curve x2 + y2 = 4 at ( -fi, -fi), B = :~ for the curve sin y + sin x = sin x . sin

dy
y at (n, n) and C = dx for the curve 2e XY+ eX eY- eX ­ e Y= e XY+ I at (1, 1), then (A + B + C) has the value

equal-to
(A) - 1 (B) e (C*) - 3 (D) 0
x
[Sol. A: 2x + 2yy' = 0 => y'=- ­
y
y'( -fi ) = - 1 = A

20

B: cos y , y' + cos x = sin x ' cos y , y' + sin y , cos x


when x = y = n
-y'-l=O+O => y'(n)=-l
C: 2e xy(xy' Y Y Y
+ y) + eXe y' + e e" - eX - e y' = e ' eXY(xy' + y)
at x = 1, y = 1
2e(y' + 1) + e2 y' + e 2 - e - ey' = e2(y' + 1)
ey' + e = 0 => y' = - 1
hence A + B + C = - 3

cos x
x 1 L' . fl(X)
54 Let f(x) = 2sinx x2 2x . Then x~Jt -- =
x
tanx x 1
(A) 2 (B*) -2 (C) -1 (D) 1
[Hint: C 2 ~ C2 - xC3 => fix) = x2 (tan x - cos x)] => f (x) = (tan x - cos x) 2x - x2 (sec-x + sin x) ]

x+a
55 If -2- = b corI(b In y), b > 0 then, value ofyy" + yy' In yequa1s
. I
2
r
(A) y' (B*) y' (C) 0 (D) 1

x+a x + a )

[Sol. -2- = b cot-I(b In y); cot ( Th =bIny [Apex 2007: Test-V: Paper-I]

·. - cosec-
(x+a) 1 b,
Th 2b = yy ; ·.
1 ( 2(x+a)J
- 2b2 l+cot 2b =
y'
Y
2

_ ~((l+(bIny)f)= L. _ - 1 (2(bIny)-y'
b ) = yy"-y'
· . 2b s : ·. 2b2 Y y2
2 2
· . -In y y' = y y" - y' ; ·. yy"= y' -y' y In y
2 2
· . Yy" + Yy' In y = y' - y' y In y + Yy' In y = y'

(A) equal to 0 (B) equal to 1 (C*) equal to - 1 (D) non existent


x"
[Hint: Lim x X= 1 ; let I = x hence as x ~ 0, XX ~ 1
x---*o
L = (0)' - 1 =- 1 => (C)

If y2 = P(x), is a polynomial of degree 3, then 2 (:J (l .::;) equals

(A) P I" (x) + P' (x) (B) P" (x) . P'" (x) (C*) P (x) . P'" (x) (D) a constant
d- .
[Sol: 2 dx (y3 Y2) = 2 (Y3'Y3 + 3 y2 YI Y2)' Now differentiate y2 = P(x) thrice)]

58 Let f (x) be the continuous function such thatf(x) = 1- eX for X:f=. 0 then
x

21

1 1 1 1
(A) f' (0+) = - and f' (0-) = - - (B) f' (0+) = - - and f' (0-) = ­
2 2 2 2
1 1
(C) f' (0+)= f' (0-) = - (D*) f' (0+) = f' (0-) = - ­
2 2

I-ex eX -1
[Sol. for continuity Lim-- = f(O); hence f(O)=- Lim-- =-1
x-tO X h-tO -x

h
_1-_e_ +1 h
I-h-[I+~+ h: + ] 1
f' (0+) = L'tm h L' 1- e + h
= im -
1. 2. = - ­
2
h-tO h h-s-O h2 - h2

l-e- h h h -......]

i-t.- 1-,+-, 1
~~+1 -h [ 1. 2. = __
f' (0-) = L'1m -h l-e -h
= L'Im~~-
h-tO - h h-tO h2 h2 2

l-e- x

if x:;t: 0 J

hencef(x) = [ X .r

-1 if x =0

59 Suppose f(x) = eax+ ebX, where a:;t: b, and that f" (x)-2 f' (x)-15 f(x) = 0 for all x. Then the product
ab is equal to
(A) 25 (B) 9 (C*) - 15 (D)-9
[Sol. (a2 - 2a - 15)eax + (b2 - 2b - 15)ebX= 0
=> (a2-2a-15)=0 and
b2-2b-15=0
=> (a-5)(a+3)=0 and (b-5)(b+3)=0

=> a = 5 or - 3 and b = 5 or - 3

a :;t: b hence a = 5 and b = - 3

or a = - 3 and b = 5

=> ab =-15 Ans.]

d2 y
60 If eY + xy = e, then the value of -2 for x = 0 is
dx
(A) lie (B*) lIe 2 (C) lIe 3 (D) none of these
[Sol: When x = 0, eY = e => y = 1
Differentiating w.r.t. x, we get {
dy dy
eY - +y+x- =0 .........(1)

dx dx
2y 2y
eY d l-~y(dYJ2 + dy
+ dy + X d = 0 .........(2)

dx 2 ~ dx dx dx dx 2

-, ~"Y dy 1

~X=O,y=1 :.From(l)-dx =-­ e

Putting the data in (2), we get


oV /I
d2y 1 2 d 2y ~
e : dx2 +e' ~-~ =0 :. dx 2 =~e2 ]

cos 6x+6 cos 4x + 15 cos 2x+1O dy


61 If Y= then - =
cos 5x + 5 cos 3x + 10 cos x ' dx

22

(A) 2 sinx +cosx (B*) -2sinx (C) cos2x (D) sin2x


[Sol. N' = cos6x + (1+5) cos4x + (5+10) cos2x +10
= cos6x + cos4x + 5(cos4x + cos2x) +10 (1 + cos2x)
= 2 cos5x cosx + 10 cos3x cosx + 20 cos-x
= 2cosx [cos5x + 5 cos 3x + 10 cosx]
------Denominator-----­
Nr
:. Y= - == 2cosx
Dr :. :~ == -2sinx => ~ fi
d2 y
62 The value of -2 if x2 - y4 == 6, is
dx
3 y-3x
(A)- 16y7 (B)L (C)2T
3x
[Sol. y4 = x2 - 6
dy dy X

4 y3 - ==2x => y3 dx. ="2. =>

dx

d2y X 1 d 2y x2 1 3 d2y + 3X2_~


y3_ +3 y2 - =- => y2
y3 dx 2 + 3 4y6 ="2 =>
dx 2 ( 2y3 J' 2 Y dx 2 4y4 - 2

y3d2y = ~ _ 3x 2 d2y 2 y4 -3x


2
d2y 2y4 -3x 2
=> => y3_== => 4y7 Ans.]
dx 2 2 4y4 dx 2 4y4 dx 2

63 If f" (x) = ~(x) and ~ , (x) = f(x) for all x. Also, f(3) = 5 and f '(3) = 4. Then the value of [ f (10)]2 ­
[~(1 0)]2 is
(A) 0 (B*) 9 (C) 41 (D) None of these
d
[Sol: dx ([f(x)]2 - [~(x)F} == 2 [f(x) ·f'(x) - ~(x) '~'(x)] = 2 [f'(x): ~(x) - ~(x)· f(x)] = 0
[.: f' (x) == ~(x) and ~ , (x) = f(x)]
=> [f(X)]2 -[~(x)F = constant
:. [f(10)]2- [~(10)F = [f(3)]2 - [~(3)F == [f(3)]2- [f' (3)]2 = 25 - 16 = 9
4
cos" a sin a 1 dy
64 I f - - + - - = - - then - =
x Y x+Y dx
(A) xy (B*) tarr'rx (C) 0
4
cos a sin" aJ
[Sol: (x + y) ( - x - + - y - == 1 = (cos?« + sin2a)4

or

23
65 The function f(x) = eX + x, being differentiable and one to one, has a differentiable inverse

f-I(x). The value of ~ (r') at the point ftc n2) is


dx

1 1 1

(A)­ (B*) ­ (C) ­ (D) none


.en2 3 4

[Hint: y =ff- + x; diff. w.r.t y, 1 = (e" + 1) :~ ; :~ = eX 1+ 1 => :x]y x=Rn2

X tan " x+sec-I(I/x), x E(-I,I)-{O}


66 If f (x) = [ , then 1'(0) is
n/2 if x = 0
(A *) equal to - 1 (B) equal to 0 (C) equal to 1 (D) non existent

1 (1 )
[Sol.
dy
dx ~ l+x'
x
J:, -0
+ tanlx + I~I -I

~ X' +lan~~: +kb (-:' J


(
x
l+
d'
as x ~ 0, d~ = - 1 Ans.
l(h)+sec- I(I/h)-n/2
Alternatively: 1'(0+) = Lim htan- = Lim cos-\h)-n/2
h~O h h~O h
-- L' 1m -sin-\h) -
- ­ 1
h~O h

lilly
1'(0-) = - 1

Hence1'(0) = - 1 Ans.]

sinx cosx sinx cosx


67 Ify= I+~l+~""""cotheny'(O) is
(A) equal to 0 (B*) equal to 1/2 (C) equal to 1 (D) non existent
Sill X sinx sinx(l+y) (
[Sol. y= cos x y = 1 cos x 1+ Y + cos x ' yeO) = 0
1+. +-­
1+ SIllX l+y

1+ ..
y(l + Y+ cos x) = sin x (l + y)

y' + 2yy' + cos x y' - y sin x = cos x(l + y) + sin xy'

y'(O) [1 + 2y + cos x] - 0 = 1 + 0

1
2y'(O) = 1 => y'(O) = ­
2

24

1
if x w O
68 Let y = flx) = [ :-;;>
if x ec 0

Then which ofthe following can best represent the graph ofy = f(x) ?

(A) ±x WX (B) (C*) ~x (D)·~x

Hencef is differenticble at x =O. Also !d~ e -"""7 ---t 1 ~ C

d2y
Alternatively: check concavity by finding -2 and eliminate D. ] I

dx /

2;
69 If y is a function of x then d + Y dd Y = O. If x is.a function of y then the equation becomes
dx x

d 2x dx d
2x
(dX J3
(A) - +x - =0 (B) d y2 +y dy) = 0
dyz dy

Z
(C*) -d X -y (dXJ2
- =0 (D) dZx -x (dXJ2 =0
dl dy dyz dy

d 2y dy
[Sol. Given -Z +y- = 0
. dx dx

now
(

(putting in (1) )

d 2x
-d2Y y
+yd =0 ~ ~C ]

(:;J dx

25
70 Let ef(x) = in x. If g(x) is the inverse function of f(x) then g ' (x) equals to
(A) eX (B) eX + x (C*) e(x + eX) (D) e(x +In X)

[Hint: Let f (x) = y => x = f-l(y) = g (y) => x= eeY

dx eY y eY+y
- = e 'e = e = g'(y)
dy

hence g' (x) = eex+x ]

26

EXERCISE 2(B)
In cos x
if x >0

Vl+x2_1

Let f(x) =
[ esin4x -1

if x c O

In(1 + tan 2x)

Is it possible to define f(O) to make the function continuous at x = O. If yes what is the value off(O), if
not then indicate the nature of discontinuity.

. esin4x -1
Sol. LHLI = hm---­
x= 0 x->o- Rn(1 + tan2x)

putx=O- h
. e-sin4x -1
= lIm----­
HO Rn(1- tan 2h)

sin4h
. e-
= 11m -1 (-sin4h) 4h [ 1 .
J.I ]
h--+O -sin4h 4h . Rn(1-tan2h) (-tan2h).2h

(-tan2h) 2h

If(O-) = 21

= lim [ cos x -1
x--+o· 1+-x
1 2 ­ 1

( = lim (l-COSX)(-4)
x->o+ x2

hence f(O) can not define.


and .: f(0-) & f(O+) are finite hence there non-removable type disconti.
2 2 2
x x x
2 Let y (x) = x2 + --2 + 2 2 + + 2 I and y (x) = Lim y (x)
n 1+ x (1+ x ) (1+ X )D- n-7DO n
Discuss the continuity ofYn(x) (n E N) and y(x) at x = 0

2 x2 x2 x2
Sol. Y ( x) = x +
n
--2 +
1+ x (1 + x )
2 2 + ... +
(1+ X
2
r- I

27
y (x) = X 2
(l-CL, JJ
n 1_ _
1_
1+x 2

~X2 {1-(6J}
1+x 2 -1
1+x 2

l-sin7tX X < L

1+cos2ltx' 2

3 ui f(x) =[ p, X = 1.Determine the value ofp, ifpossible, so that the function is continuous If
\ .

../2x=I x> l.
J4~-2' 2
at x = 1/2.

Sol. y.pL.!.=p ...(1)


2

LHL!,,=.!. = 1i~ f(x)


2 X-)­
2

. I-sin nx
_ Il I D - - - ­
- x-.!= 1+ cos(2nx)
2

1
put x= --h
2

. l-Sin(~-nh)
= hID----"---~
h-.+O 1+ cos(n - 2nh)

r ( 1- cos nh) [
=~..To (nh)2 1-cos(2nh)
(2nh)2

1 l(
4n
2h2

n 2h2
)

28

RHLI 1
X=­
= lim f(x)
[1)+
2 X4­
2

u [ ~ :
= x~Gr ~4+.J2x-I-2

= hrp
x~
.( ~
2
~
4+-v2x-I-4
J~
( 4+-v2x-l
r;;--; +2)

': LHLL=~ *RHLlx~


2 2

so the value of function cannot determine & the function is discontinuous.


I)
4 Give1thefuncti on 9 (x) = .J6-2x ana h (x) = 2x 2- 3x + a. Then

g(x), x:51

(a) evaluate h (g(2)) (b) If f(x) = [ h() l' find 'a' so that fis continuous.
. x, x>

Sol. (i) h(g(2)) =

g(2) =.J6-4 = Ji

hex) = 2x2 - 3x + a

h(Ji) =4-3Ji+a Ans

g(X) ; x:51
(ii) f(x) = {hex) ; x> 1

.J6-2x ; x:51

{
f(x) = 2x _ 3x + a ; x » 1

2
(

V·F.lx=l =2 ...(1)

R.H.L.lx=l = ~~ f(x)

= lim(2x 2 -3x+a)
x~l+

R.H.L.lx=l = a-I ...(2)

L.H.L.lx-- 1 = x41
lim f(x)

=2

29
since function is conti

L.H.L.lx=l = R.H·L.lx=l = VFlx=l

2 =a-l =2

a-I=2=>la=31

5 Let fex) = [1+X ,O:'S:x:'S:2. Determine the form ofg(x) = f [f(x)] & hence find the point of
3-x ,2<x:'S:3

discontinuity of g, if any.

. = {l+X O~x~2
Sol. f(x)
3-x 2<x~3

3 .

/
1" 1 . (

o 2 3

g(x) = f (f(x))

={l +f(x)
O~f(x)~2

3-f(x) 2<f(x) s 3

let f(x) = y

l + Y ; O~y~2

fey) = { 3- Y ; 2 < y ~ 3

3 ---.-.
.r« :
c
I; '"! :
... ~ ........•..........
! _ .

2 . .
··· ...
1 ·······i········ J..
: : ~
: : ..;
: : -,
! i +
2 3
1+ (1 + X) O~x ~l

= 1+(3-x) 2<x~3
{
3-(l+x) l<x~2

30

2+ X os x s 1
= 2-x 1<x:s;2
{
4-x 2:S;x:s;3

: ...~:~l-·-· ·s- - _· _·
!,
; :
:.
.
1 __.. · ····i··· · .
···· ..
. ...,,
··
,
I
.' .
'
'
I :

o 2 3

so the point of discontinuity

1,2 Ans

( Or
F.V.l x = LHL = RHL

6 Let [xl denote the greatest intege- function & f(x) be detlned in a neighbourhood of 2 by

(exp {(x+2).en4}t:l] -16 2


.x-c
f (x) = 4'-16 .

[ 1-cos(x-2)

A (x-2)tan(x-2) ,x>2
Find the values ofA & f(2) in order that f(x) may be continuous at x = 2.

Sol. RHLI x -_2 = lim f(x)


x~2+

( = lim A(l-cos(x-2» . 4- n / 2_1


= hm-:--­
x-4 (x-2).tan(x-2) h--.O 4- h_1

putx= 2+ h

. A(l- cosh)
- 11m ----'---'­
- h--.O h tan h

31
A
RHLl x=2 = 2 since the function is contin.

VFIx=2 = RHLl x=2 = LHLl x=2

A 1
LHLl x = 2 => !~~ f(x) Y.F.l x=2 = 2 ="2
[x+1]
(e(X+2)£n4) -16-4­
= lim
x->2­ 4 x -16 IV.F.I.=2= ~I Ans

(x+2) ([x)+I)
=
liI m 4 -4 - - -16

x->2­ 4 x ­ 16
!A=ll Ans

[ X+2)
. 4
2 -16
= h m x- - - ­
x.->T 4 -16 (
put x=2-h
4-h
. 4 2 -16
= lim ---'-2-'-h-­
x~ 4 - -16

(6~
~JlMI5X if 0<x<.1!.2
7 b+2 if x=1
-£ ~tanxl)
(l~cosxl"'-b- if -kx<n
2
Determine the values of 'a' & 'b' , if f is continuous at x = n12.

Sol. v.F.1 x=~ 2


= b +2 ...(1)

LHLI "= lim f(x) = lim


x~2 x-+~
2
(
(")­ 5
X-+ -
2
6 ) :: ~ :
-
(

n
put x= --h
2

tan6h

LHLlx=~ r ( 6 J tan(3n - 6h) . (6 J cotSh


2 =hl~ 5" tan(5nI2-5h) = ll~ 5" = 1

RHLj _" = lim f(x)


"-­
2 X-",­ [")+
2

a
--tan x
_ lim (l-cosx) b

- x-+(~r

32
7t
putx= -+h
2

~coth
= lim(1 + sinh)b ; 1<Xl form
h-->O

_ lim (sinh)~coth
- eh-->o b
Iim!cosh a
= eh->Ob =e b
since the function is conti so

LHL\x=-"-
2
= RHLlx=-"-2 =v.F·lx=-"­2

!a=O, b=-ll

J)
(
(~-Si~-l(1- {X}2)}in-1(1- {x})
8 Let f(x) = x:;tO
.J2({x}- {X}3)
7t
x=O
2

where {x} is the fractional part ofx. Consider another function g(x); such that
g(x) = f(x) ; x ~ 0

= 2.J2f(x); x < 0

Discuss the continuity of the function f(x) & g(x)at x = o.

Sol. RHLl x = 0 = !~~ f(x)

. (~-Sin-l(1-(x - [X]2) }in-I(1- x + [xl)

- lim .J2

- x-->o+ 2(x-[x]-(x-[x]i)

33
1 - X2 = cos 8
x2= 1- cos 8

x =.Jl-cos8
when x ~ 0+ then 8 ~ 0

- -1t],
- 1m -----r===8
- 2.fi 8-->0+ .Jl- cos8

= _1t_ lim 8 = ~lim 8


2.fi 9-->0+ .J2-sin z8/2 49-->0+ Isin8/21

RHL
Ix = 0 = ~lim2(
4 8-->0+
.8/2 J=~
8I 2
SIll 2

LHLI x= 0 = x-->O
lim f(x)
(
~ -sin-l(l-x -[X])Z)sin-1(l- x + [x])

( 2

nIlfl ....:>...:...----;=------<---------,----­
= x-->O- .fi(x-[x]-(x-[x]n

~ - sin-1(l- (x + 1)2) sin' (-x)

= lim -=2'----------,=- _

x-->O- .fi(x+l-(x+ln

=
lim 1t-COs-1(x 2+2x),sin-lx
x-->O- .fi(x + l)(x + 2) x

1t
LHLlx=o = 4.fi

for f(x) since LHLl x = 07:- RHLlx=o so the function is discontinuous at x = O.


for g(x) ~

RHLl x = 0 = !~r;;, g(x)

34

= lim f(x) = 2:
x-->O+ 2

LHLlx=o = !~~ 2J2f(x)

= 2J2lim f(x)
X40­

r: 1t 1t
= 2...;2.-=­
4.J2 2

1t
g(O) = f(O) = ­
2

x2
- - for x s O
9 If the ~9ction f(x) defined as f(x) = [
2 is continuous but not derivable at x = 0
( xnsin l for x c- O
then find the range of n. x

Sol.
.
f(x) =
[-~for
2

x ~O

xnsin~for x c- O

f(x) is continuous at x = 0

f(O) = 0

r r (_x 2 \ 0

L 1= x~~ f(x) = x~~l2) =

"
L = 1lmf(x)= l'rm x" sin ­ 1
2 x-->O+ x-->o+ X

( for continuous,

lim f(x) = lim f(x) = f(O)


X40+ X-.+O­

=> x-->O+
lim x" sin(l) = 0
X

limit is defined only when

:. n>O

since f(x) is non-differentiable at x = 0

h2
fh+O-fO. -~--o . -( h)
f '(0-) = lim ( ) () = lim 2 = lirn ­
h-->O- 2 h-->O- h h-->O- 2

35
h" sin~
f(h + 0) - f(O) I'
· h
f '(0+) = Iim == rm
h-->O' h h-->O' h

=> lim h n - 1 sin(~) ;t: 0


h-->O' h

only when n - 1 S 0

=> n s 1 .....(ii)

from equation (i) & (ii)

n E (0, 1]

10 Letf(O) = 0 andf' (0) = 1. For a positive integer k, show that

Lim~(f(X)+f(~)+
x~o x 2
.f(~))
k
= 1+~+.!.+
2 3
+~k
(

= lim f(x) 2
f(~) k
f(~)
x-->o - - + - - + ....+ - ­
X X X

f(~+Ol-f(O) f( ~+o) -f(O) 1


=
· f(x+O)-f(O) u
I1m + x-->o
1m
x)
.
~
+ +
lim
O"
k
x

k

x-->o X X 2···· x--> _

2 k

1 1
= f'(O) +2 f'(O)+ .....+ k f'(O)
(
1 1

= 1 +2+····+ k

11
__[a x2
- b if <1 Ixl
II is derivable at x = 1. Find the values of a & b.
If f(x)
-g if xlz l

ax2 _ b if'[x ]c l
Sol. f(x) = 1
if l x lz l
r-[x-]
f(x) is differentiable at x = 1, hence it is also continuous at x == 1

36

lim
x-»t
f(x) ee f(1)

~Ia-b==-II ...(i)

· f(h+I)-f(I) l' a(h+I)2-b+I


f '(1) = Irm == rm ---'--"'------­
h-->O­ h h-->O­ h

. ah 2 + 2ah + a - b + 1
== I1 m - - - - - ­
h-->O­ h

. ah? +2ah .
== hm == lim (ah + 2a) == 2a
h-->O­ h h-->O­

-1
--+1
· f(h+I)-f(l) I' Ih+l[
fl(1 +) = Itrn = im
h-->O+ h h-->O+ h
(

-1+I+h
lim 1+ h = lim-I-=I
h-->O+ h h-->O+ 1+ h

f '(1-) = f'(1 +)
~2a= 1
a= 1/2
b=3/2

aX(X- I) + b when x <1


12 Thefunction f(x) = x-I when l s x s J
[
px' +qx+2 when x >3
( Find the values of the constants a, b, p, q so that
(i) fix) is continuous for all x (ii) f' (1) does not exist (iii) f'(x) is continuous at x = 3

ax (x -1)+ b when x < 1


Sol. f(x) = x-I when l s x sJ
[ 2
px + qx + 2 when x > 3
f(x) is continous at x = 1

lim
x-e l
f(x) = f(1)

~ x-->l
lim ax (x - I) + b = 0

37
lim a(h+I)(h+l-l)+b
1 _ lim f (h + 1)- f (1) _ h.....O- h
f (1) - h.....O h - h + 1- 1
{ lim
h..... O+ h

_
lim
h.....O­
a(h+~h+O = {~~a(h+l)
{ lim h 1
h..... O+ h

': f'(1)=DNE~a*1

:,aER-{l}&b=O

f(x) is cant. at x = 3

lim
,,->3
f(x) = f(3)

~ lim (px 2 + 9x + 2) = 2
" .....3

~ 9p+ 3q +2=2

~ 9p + 3q = 0 ....(i)

': f'(x) is cant. at x = 3, hence f(x) is diff. at x = 3

, f(h + 3) _ f(3) {~~~ _3_+_h_~_I_-_2


f'(3)=hm =
h->O h , p(h+3/+q(h+3)+2-2
hm -'----'---~----'--'--~--
h.....O+ h

{I 'h Im-
h.....O- n {1 2+6ph+qh
(
2+6ph+qh+9p+3q
= I' ph = lim ph
im h->O+ h
h .....O+ h

[from equation (i) 9p + 3q = 0]

={:im (ph+6p+q) =
h~O+
{6P~q
:. fl(3+) = f'(3-) ~ 6p + q = 0 ...(ii)
solving equation (i) & (ii) p = 1/3, q = - 1

a E R - {I}, b = 0, p = 1/3 , q = - 1

13 Discuss the continuity on 0 ~ x ~ 1 & differentiability at x = 0 for the function.

38

f(x) = x.sin ~.sin . I where x*-O, x*-1I rn & f(0) = f (11 rn) = 0 ,
x x.sm X
r = 1,2,3, .

Sol. f(x) = x . sin ~ . sin ~ 1 x*- 0, 11m

x x.sm­
x

f(O) = 0 = f(~), r = 1,2,3....

' f(h + 0) - f(O)


f'(O) = Lh-->O
im
h

( 1
hsin(~).sinl hsm.1(1)J-0

­
. h
= LI m--------­
h-->O h

( 1
hSin(~) .sinl hsm. 1(­ l)J
. h
= LI m - - - - - - - ­
h-->O h

( 1
= ~~~sin(~).sinl hsm.1e)J
­
( h
'\

= L. sm. (1)
Hfl
h-->O h
. (-
- sm -1­ )
hsin(1/h)
'-v---J '~--v----'
v
1-'; st -I'; ,;1

=DNE
so f(x) is not differentiable at x = 0

Limf(x) =
x-->o
Limxsin(~) ,sin( x sm(11
x-e-O X
. 1
x)
)

39
= Lim x .sin(l/ x) . sin ( 1 \j
<-->0..':'0 ~ nsin(l/x)
-L,;], • '
-I'; ,;1

=0
= f(O)

( "
Lirpf(x) == LirpxSin(.!-).Sinl
X-->-
rn
X-->-
1tr
X
xsm ­
.
l( )j 1
x

( 1 "

.lrpx.sm. (1)- .sm. sin(~) ,r' (


== L
(~)
x-->- X

rn

( "
~~x,sin(~).sinl
rn '----v--'
-->0
X sm
. Ie )j ­
x

-I'; <:1

=0

(
== fC~)
Hence function is continuous V x E [0, 1]

l- X (Osxsl)
14 f(x) == x+2 : (1<x<2) Discuss the continuity & differentiability of y = f [f(x)] for 0 s x s 4.
[
4-x , (2sxs4)

1- X (Osxs1)
Sol. f(x) = x+2 : (1 < x < 2)
f4-x , (2sxs4)

40

1- f (X) 0:$ f(x):$1

f(f(x)) = f(x) + 2 1 < f(x) < 2

{
4-f(x) 2:$f(x)~4

l-l-x O:$x~1 (J l~l-x:$l~O~x~1

l-x-2 1< x < 2 (J 0:$ x + 2:$1 ~ -2:$ x:$-1


1-4+x 2:$x~4 (J 0:$4-x~I~3:$x:$4

l-x+2 O:$x~l(J 1<I-x<2~-I<X<O

x+2+2 1<x<2 (J 1<x+2<2~-1<x<O

4-x+2 2:$ x ~ 4 (J 1< 4x < 2 ~ 2 < x < 3


4-1+x Os x s l (J 2~1-x~4~-3~x:$-1
4-x-2 1<x<2 (J 2:$4~O~x:$2

4-4+x 2:$x:$4 (J 2:$4-x:$4~0:$x:$2

x O~x~l

x-3 3:$x~4
= -x+6 2<x<3
-x+2 1<x<2

x Os x s I

-x-2 1< x <2

f(f(x)) = x x=2

-x+6 2<x<3

x-3 3~x~4

( (2,4)~

Y
(3,3)
(2,2)
(4, 1)
1

2 3 4

:. f(x) is continuous at x = 1 & discunt.

at x = 2, 3 & non diff. at x = 1, 2, 3

15 Let f be a function that is differentiable every where and that has the following properties:
(i)f(x+h)=f(x)·f(h) (ii) f(x»Oforallrealx. (iii) f'(O)=-1
41
Use the definition of derivative to find II (x) in terms of I(x).

x=O
Sol. f(x + h) = f(x) . f(h) f(O) (f(O)- 1) = 0 => f(O) = 1
Ih=O

· f(x + h) - f(x)
f'(x) = 11m -----'----'------~
h-->O h

= lim (x).f(h) ­ f(x) = lim f(h) -1 f(x)


h-->O h h-->O h

. f(h)-f(O)
=> f'(x) =hm
h-->O h
f(x)

= f'(O) f(x)
=> f'(x) = ­ f(x)
:. f'(x) = ­ f(x)
/' (
16 Discuss the continuity & the derivability of 'f where f(x) = degree of (UX 2 + u2 + 2u - 3) at x = ~2.
x2 2 t:
Sol. f(x)=degreeof(u +u +2u-3)atx =,,2

. f(h+Ji)-f(Ji)
f' (Ji ) = Lh!..~ h

-lE~ f(h+Ji~-f(Ji)
. f(h+ Ji)-f(Ji)
LIlfl -----'------'---'---'­
h-->o­ h

2
Lim 2+ 2Jih + h - 2
= h-->o+ h
{
o

42

Hance f(x) is non differentiable at x = J2

Limf'(x) = Lim x 2

x--'>.J2 x--'>.J2

=2

=> f(J2) = Limf(x) x--'>.J2

Hance f(x) is confinous at x = J2

17 Letf(x) be a function defined on (-a, a) with a> O. Assume thatf(x) is continuous at x = 0 and
. f(x) - f(kx)
Lim = a , where k E (0, 1) then compute f' (0+) and f' (0-), and comment upon the
x-tO X

differentiability of fat x = o.

Sol.
": Lim f(x)-f(ka) =a
x--'>o X

" f(x)-f(O)+f(O)-(kx)
=> L 1m =a
x--,>o X

' f(x)-f(O)-f(kx)+f(O)
=> L 1 m . a
x~o x

" (f(X)-f(O) f(kX)-f(O»)

=> L 1m =a
X--'>o X X

=> (L ' IHl


x-e-O
f(X)-f(O»)
X
-
(L' 1m f(kX)-f(O») k =a
X--'>O kx

43
. f(x)-f(O) L· f(kx)-f(O) k
1m 1m . =0­
X-->O- X X-->O- kx
~
{L
Lim f(x)-f(O) -Lim f(kx)-f(O).k =0­
x-->o+ X x-->o+ kx

f '(O- ) - kf'(O-) = 0­
= { f'(O+)- kf'(O+) = 0­

]8 A derivable function f: R+ ~ R satisfies the condition f(x) - fey) ;::: In (xjy) + x - y for every

x, y E R+. If g denotes the derivative of f then compute the value of the sum L g(1)
100
- .

n=l n

Sol. f(x) - f(x);::: fn(x/y) + x - y


~ f(x)- f(y);::: fnx- fny + x - y

f(x)-f(y) .enx -my ]

~
x-y
>
- x-y
+ [for x:;t: y]

· f(x)-t(y) 1· fn x-fny ] (
~lm 1 ;:::Im +
X-4y X - Y X-4y X - Y

(Y+h)

~ lim f(h+y)-f(y) ;::: lim m -y- +1

h-->O h h-->O h

h)l/h
h-->O
(
=:> f'(y);::: lim fn ]+-
Y
+1

]9 If y= ~-+~x~x2+1+/n~x+~x2+]provethat2y=xy'+/ny'.where'denotesthederivative.

44
2+
=x+ 2.Jx 2+1 [2 (x 1)]

y' = x + .Jx 2 + 1

Also2y=x2+X.Jx 2+1 +In(x+ .Jx 2+1 )


=x(x+ .Jx2+1 )+In(x+ .Jx 2+1) =xy'+lny'Renceproved]

dy 16t(1-t 4 )
20 If Y= sec 4 x and x = tan-let), prove that
dt (1_6t 2+t 4)2'

1 1+tan 22x
[Sol. y= cos4x = 1-tan22x ....(1)

using tan x = t(given)


2t
tan2x= - ­ 2

1-t
substituting in (l)

4t 2
1+-----0--::­
(1-ei _ (1+t 2)2 _ (1+t 2)2

y= 4t 2 -(1_t 2)2_4t2-1-6t2+t 4

1- -----=-_=_

2)2
(1-t

dy = (1-6t 2 +t 4)'2(1+t 2)'2t-(1+t2)(4t 3 -12t)


dt (1-6t 2+t 4)2

4t(1+ t 2)[(1-6t2 + t 4 ) - (1 + t 2)(t2 - 3)] 4t(1+ t 2)(1- t 2) 4t(1- t 4 )


(1_(t 2+t 4)2) = (1-6t 2+t 4)2 = (1_6t 2+t 4)2

21
l+lnt
If X=-2- and y
3+2lnt dy
" Show that y - = 2x -
(d )2+ 1.
y
t t dx dx
dx t -(1 + lnt) 2t t(1- 2 -lnt) (l + 2lnt)
[Sol. 4 4 3
dt t t t

t(~t )-(3
+ 2lnt)
dy =----"---..:....-~_- (1+21nt)
dt t2 - - t2

3
dy = 1 + 2ln t t
dx t2 "1+ 2In t = t

3+2lnt
Now L.R.S. .t = 3 + 2Int
t

45
2(1 + Int)
2
R.H.S. == 2 . t + 1== 3 + 2In2
t
=> L.H.S. == R.H.S. ]
X, x 2·x x 3.x 2
22 If y == 1+-_-+ ( _ )( _ ) + ( )( )( ) +..... upto (n+ 1) terms then prove that
x x, x x, X X2 x-xl X-X 2 X-X 3

dy
dx
==J...[~+~+~+
X X,-X X X3-X
... +~]
Xn-X2-X
[Sol. adding term by term

X X x x
Iny== In + In - - - + In - - - + +In - - ­
(x-cx.) (x-x2) (x-s x.,')
(
_ x ) _ x-x n [(X-Xn)-X] _ ~ [~)
nowD [ ­ 2 ­
X- Xn X (x - x n) x x n - X

Hence y1 dxdy 1 [Xl X2


= ~ xl -x + X2 -x + .... + Xn -x
Xn ]

dy
dx
=y[~+~+ +~]
X xl - X X - X .... X - X ]
2 n
23 Suppose f (x) == tan (sin-, (2x))
(a) Find the domain and range of f
(b) Express f(x) as an algebaric function ofx.

1 1) 2x "(c) 16.fi ]
(c) Find f' (1/4). [Ans.(a) ( -2'2 ' (-00,00); (b) f(x) = ~1-4x2
9
[Sol. f (x) = tan (sin-\2x)) (
(a) for f to be well defined
1 1 1 1t
-1 < 2x < 1 => - 2 <x <"2 [... for x == ± "2 ' tan 2" is not defined]

Hence domain is (- ~, ~)
for x E (-~,~), sin- 12x E (-%' %) hence for (-%,~) can take all real values.
Hence range of f is x E R

(b) f (x) = tan e where sin- I(2x) = e


=> sine = 2x
2x

f(x) == ~ 2

1-4x

46

(c)

dy 1
24 If x== tanI-ln
2
Show that dx == "2 sin y(1 + sin y + cos y).

Y . 2t 1-t Z
Sol Put tan- ==t smy=--z ,cosy==--z
2 l+t l+t
. 2+2t
l+smy+cosy ==--2
l+t
and y==2tan-1t ...(1)
dy 2
-==-­
2 ...(2)
dt 1+t
Now X == t -21og(1 +t)+ logt

dx=l_~+!==~
dt 1+ t t t (t + 1)

dy dy dx 2 t 2 +t
dx == dt +dt == 1+t 2 ·1+t2' by (2) & (3)

dy
or - = -2t-2 . 1_2t+2
-­2
dx 1+t 2 1+ t

1. (
==-smy .
l+smy+cosy ) ,by(1)
2
(

~
OS 3X dy 6 . 0.
25 If Y== arc cos - - 3­ then show that dx == - ,smx>
cos x cos2x + cos4x
Sol We have,
cos3x
y=cos -I
cos' x

cos3x
cosy=
cos' x

cosy=
)4COS x-3cosx
--­
3

cos' X

=> cosy =.J4-3sec 2 x

47
:=:} COS
2
y=4-3(1+tan 2 x)
:=:} l-cos 2 y=3tan 2 x
:=:} sin 2 y = 3 tan 2 x

:=:} sin y = J3 tan x


Differentiating both side with respect to x, we get, cos y: = J3 sec'' x

:=:}
dy
=
J3
dx cos vcos" X

dy
=-­
J3 =
3
dx cos" X cosx cos3x

dy 6 .
Hence Proved dx = 1 - - - - - , smx> O.
cos2x + cos4x

26 Prove tha if I al sin x + ~sin 2x + + ansin nx I s I sin x Ifor x E R, (


then lal+2al+38:3+......+nan I s 1
[Sol. Let f'(xj v a.sin x e-a.sin Zx } +ansinnx
f' (x) = a l cos x + 2~cos 2x + + nan cos nx
f' (0) = a l + 2~ + ....... + nan
Hence TPT If' (0) I ~ 1

Given I f(x) I s I sin x I for x E R

f'(O)= Lim f(h)-f(O)


h~O h

f' (0) = Lim f(h) (as f(O) = 0)


h~O h

I f' (0) I = . If(h)\


LIm - . ISinhl
~ LIm - = 1 [as lf'(xj l s l sin x ] ]
h~O h h--70 h
Hence If' (0) I ~ 1 ]

(
In(tan~) changes the equation
2
dy
27 Showthatthesubstitution z= d ; +cotx +4ycosec2x=0
2 dx dx
to (d2y/dz2) + 4 Y = O.

Sol Since x=lntan(~)


dz dx .
-=cosecx or -=smx ...(1)
dx dz
dy dydz dy.
Now, - = -. - = cos ec x·-- [From (1)] ...(2)
dx dz dx dz

=~(dY)=~(cosecxdY)
2y
d
dx' dx dx dx dz

48

= cosec X!!......(dY) + dy (-cosec x cot x)

dx dz dz

= cosec x·- - Y ) ·--cosec


d (d dz dy
xcotx­
dz dz dx dz

d2 d
= cos ec 2 x - { - c o s ec x cotx 2 [From (1)] ....(3)
dz dz
2
d d
But given ----?+
dx
cotx 2+4ycosec
dx
2x
=0
2
cos ec'x --?-
d
dz
d
dz
d
cos ec x cot x -.2::. + cot x cos ec x -.2::. + 4Y cos ec' x = 0
dz
[From (2) and (3) ]

2
2 d y 2
cosec x-z+4ycosec x=O or
dz
xe" xs 0
28 Letj(x) = [ then prove that
2_X3
X+X x c-O
(a) / is continuous and differentiable for all x. (b) / 'is continuous and differentiable for all
x.
xex+ex =eX(x+l), x c O
[Sol. f' (x) = [

1+2x-3x2 x >0

Lim f'(x) = 1; Lim f'(x) = 1


x~o- x-+o+
hence f (x) is continuous hence/ is continuous and differentiable at x = 0

ex +(x+l)e X =e X(x+2),
x c O

Again f ''(x) =
[
2-6x x c- O
Lim /"(x) = Lim /"(x) = 2 f' (x) is also continuous and differentiable]
x--+o+ x-e-O"

a+x b+x c--x


29 Let f(x)= R+x m+x n--xr . Showthat f" (x) = 0 andthat f(x)=f(O)+kx where kdenotes the sum
( p+x q+x r+x
of all the co-factors of the elements in [(0).
a+x b+x c+x a+x b+x c--x
[Hint: f' (x) =
R+x m+x n+x + + R+x m-i-x n+x
p+x q--x r+x p+x q+x r+x
f" (x) = 0 (obviously - two identical rows)
[' (x) = k ::::> f(x) = kx + x, f(O) = c
f(x) = [(0) + kx. Note that f' (x) = k
I abc abc
l
f' (0) = k = R m n + II+Rmn
p q r p q r 11
+ C l2 + c 13) + (c 2 1 + c22 + c23) + (c 31 +
(C I I C32 + C33)
sum of co-factors of elements [(0) ]
49
30 IfY = sX and Z = tX, whereallthe letters denotes the functions ofx and suffixes denotes the differentiation

X1 YY1
w.r.t. x then prove that X ZZl __ X3 /S1 t11
X y Z S2 t2
2 2 2

Sol Since y = sX and Z = tX ...(1)


r; = sXI + XSI and ZI =tX I + Xtl ...(2)
~ r; =sX2 +Xs 2 +2s1XI and Z2 =tX2 +Xt2 +2t1X1 ...(3)

x sX tX
XI sXI + XS I tX I + Xtl [From (1),(2) and (3)]
X 2 sX2 + XS 2 + 2slXI tX 2 + Xt 2 +.2tIXI
(

Expand w.r.t. first row, then


XSI
-X
I
Xs2+2s IX I

50

EXERCISE 3

x
Let f(x) = -.-, x> 0 and g(x)=x+3, xcl
smx
=2-x, x s O =x 2-2x-2,lsx<2
=x-5. x:::::2
find LHL and RHL of g( f'(xj ] at x = 0 and hence find Lim g( f'(x) ].
x--+o ,

x>O
Sol. f(X)={Si:X
2-x xsO

X+ 3 x<O
g(x)= x 2-2x-2 1 SX < 2
{
x-5 x~2

/
f (X) + 3 / f(x) < 1
2(x)-2f(x)-2
g(f(x))= f lsf(x)<2
{
f(x)-5 . f(x) ~ 2

x x
--+3 x>On--<l
sinx sinx
2-x+3 xs 0 n 2x<1

= (r()
-
x
sinx
x
-2 - - 2
sinx
(2-x)2_2(2-x)-2
x>On ls--<2

xsOn1s2-x<2
x
sinx

x x
-.--5 x>On--:::::2
smx sin x
2-x-5 xsOn2-x:::::2
(
't.

$
$

(_ x )2 _2(_X)-2 x>Onls-.-<2
x
sinx sinx smx

_x__ x
5 x>On--~2
sinx sin x
2-x-5 xsO

1
-x-3 xsO
X X x
=> g(f(x)) = ( - )
2 -2 ( - -) 2
Is--<2nx>0
sin x sin x sinx
x x
--5 x > On--;::: 2
sinx sinx

'.' gf(O) = - 0 - 3
=-3

LHL = !~~ g(f(x))

= x->o
lim -(x + 3)

=-0-3=-3

RHL = !~ g(f(x))

,
= I1m x
- -)
2 -x- - 2
2- ( ) ]
[(
x->o+ sin x sin x

=1-2-2
=-3
': LHL =RHL=-3
:. lim
x-> og(f(x))=-3

2 Let Pn =a Pn- 1
-I, 'if n = 2, 3, and Let PI = aX - 1 where a E R+then evaluate Lim Pn .
x~o X
Sol. Given Pn = a Pn-1 -I; Pn-I = a Pn-2 -1
Let PI = aX - 1

lim~
x~o x

, a Po-1 -I p
=> lim x --!!.=l
x~o X Pn-l
2
=> lim Rna x Pn-l
X40 X

. ap -I p
lim Rnax n-l x n-2
=> X40 X Pn-2

=> lim(Rna/ x Pn-2


X40 X

=> lim(Rnat- 1 x£!


X40 X

=:> (Rnat

, I (I I
3 /'
l+ax)
If the ~~ x 3 .Jl + x - 1+ bx exists and has the value equal to I, then find the value of ;- -T2 + b3 .

Sol. r
x~~
1[I l+axJ
.Jl + x - 1+ bx

= lim_1 [1+bX-(I+ax)~]

x~o x' ~ (1+ bx)

, l+bx-(1+ax) (1+xyI2

- I1m

- HO x\l+xyI2(1+bx)

We know that

n(n -1) 2+ n(n -1) (n - 2)


3+
(l+xr=l+nx+ x x ....
2! 3!

where, n E Q & I x I < 1

b 1 22.
1-(1--1) 1-(l-l) (1--
2 22 2
2)
3
]
1+ x-(1+ax) 1+2"x+ 2! x + 3! x + .

= lim
HO
r
-'=-

x 3(1
+ XY/2 (1+ bx)
____='_

H~-]) (~-2)
3!
X3_ ... _axl(1+!x+H~-1)~2+H~-1)(~-2)x . .
2 2! 3!
3+ Jj
l i m - - - - - - - - - - - - - - - - : c - - -3- - - - : - ; - ; - - - - - - - - - - - - - - - - - - : . . .
x-->o x (l + X)1I2(l + bx)

3
1221.(1.- 1) 2222
1.(1. -1) (1.- 2) 3
l(122 1.(1. -1) 2222
1.(1. -1) (1. - 2) 3
~\
bx - 2"
X - 2! X - 3! X - ••. - ax 1+ 2" + 2!
X X + 3! X +.....

=lim
x->O x\1 + X)J/2(l + bx)

b-~- ~G-l) x- ~(~-1)G-X)X2 . . . -a(ll+~x+ ~(~-1) x2+~(~-1) G-2) x+.~J

2 2! 2 2!
3

3! 3!

=lim------------ ----,-------------­
.-.0 x 2 ( +xy/2(1+ bx)

( b- a - ~) -(~+
l ~(~-1)1J X -(t(~-1)
2
l2
(~-2) +a.t(t-1)lJX2-(
2! 3! 2!
)X3 _ ( ) X 4 _ •••••••

= lim---------------- -=--------------­
2.1.1
X-40 x

Limit exists finitaly if

(i)b-a-!=O
2
1(1 1.)

(ii)~+ 2 2-
2 2!
=0
1 1
a 22
=:>---=0
2 2
1
=:>a--=O
4
1
=:> a=­
4
1
.: b-a =­
2
1
=:>b=a+­
2
3
=
4
1 3
a=-&b=­
4 4

-l( 1(1 ) 1(1 )(1 )'


2" "2 ~ 1 a _"2 2 -1 , "2 - 2
2. 3.
j x2 _ ( )x 3 _ ( )x4 _ .

= lim-----------::---------­
x-->o

4
.
= lim
HO
l---J-(
[
( a !~l~
8
222
2.3 )x-()x -
2

]
.

11111 11 1

= ---.-.-.-=--+-=-­
32 2 2 2 2 16 32 32

=>
.
~~7
1 (1 1+ ax)1

..)1+ X -1+bx =- 32

. f = - ­
.. 32

123

---+­
.. a f b

123

=---+­
1 -1 3

- - -
4 32 4

=4 + 2.32 +4
=4+64+4
=72

4
Let {an}, {bn}, {cn} be sequences such that
(i) an + bn + cn = 2n + 1 ; (ii) anbn +bncn + cnan = 2n - 1; (iii) anbncn =- 1 ; (iv) an < bn < c
n
Then find the value of Lim na .
n-7 00 n

Sol. x 3
- (2x + 1)x2 + (2x - 1) x + 1 = 0 ...(1)
roots of equation an' bn, cn an < b n < cn
x = 1 is a root of equation (1)
( so (x- l)(x2 - 2nx - 1) = 0

x = n±~n2 +1

x = n±~n2 +1

x = n±~n2 +1

5
x 3 - (2x + I)x 2 + (2n -I)x + 1
(x -1)
x 3 _x 2

- + 1

- 2nx 2 + (2n -I)x + 1


-2nx 2 +2nx
+
-x+1

-x+I

so the three roots of cubic equation.

n +~n2 + I,n -Jn 2


+ 1,1

do ·
I im => hm
. ~ n + j;;2;l
" n-e-eo
na
n n~co
n(n-\ln~
+I)x ~
n+\ln 2 +1
(

=> ~~
.
( r:IJ
-n

n I+Vl+~
1
=-"2

5 If n E N and an = 22 + 42 + 62 + ....... + (2nf and bn = 12 + 32 + 52 + ..... + (2n - 1f. Find the value

Llm
. F:-.jb; .
n~co ~

2n(2n + I) (4n + I)
an + bn = 6

b = 2n(2n+l) (4n+l) _ 4(n(n+I))(2n+l)


n 6 6

n(2n + I)
bn = 6 [2(4n+I)-4(n+I)]

b = n(2n + I) [4n _ 2] = _n(..:....2_n_+---,I)---,(_2n_-_I-'-.)


n 6 3

Lim j;;: -.Jb:


n~oo J-;;

2nn(n + 1) (2n + 1) n(n + 1)(2n + 1)


6
Lim---'------~--­
2
n~oo J-;;

Lim
n~oo
~(J4n2+6n+2-J4n2-1)
,,3
by ratio radization

1 6

= )3"2+2

=)3
2

6 At the end points A, B ofthe fixed segment oflength L, lines are drawn meeting in C and making angles
8 and 28 respectively with the given segment. Let D be the foot of the altitude CD and letx represents

( the length ofAD" Find the value ofx as 8 tends to zero i.e. Lim x .
e~o

CD
Sol. tan8 = ­
x

CD
tan28= - ­
L-x

2tan8

xtan8 = (L - x) 1 28

-tan

2(L- x)

x-
- I-tan 28

..
II(
x(1- tan 28) = 2L - 2x

7
x(2 + I - tan 28) == 2L

2L
x 3 -tan 2 8 ' tan8 --t 0

8--tO

7 At the end-points and the midpoint of a circular arc AB tangent lines are drawn, and the points A and B
are joined with a chord. Prove that the ratio ofthe areas ofthe two triangles thus formed tends to 4 as the
arc AB decreases indefinitely.
Sol. Let radius of circle is r &
LAOB=20
LACB =1t-28
( '.' points A, C, B, 0 are concyclic) (
:. LAOP = LBOP = 8

1t
L ACP=LBCP= --8
2

In ~AOP,

AP AP
sin8=-=­
OA r

=>AP= r sin8

OP OP
cos8= -==­
OA r

=> OP == rcos8

AB == 2AP = 2r sin8

(
in~AOC,

r
=>cos8= ­
OC

r
=:>OC= - ­
cos8

pc=oc-OP

_r_ r cose
case

= r(_1 -case)
case

r
= - - (1 - cas 2e)
case

r . 2e
:. PC= --sm
case

1
:. ar(LiABC) = "2 AB. PC

1 r. 2e
= - 2r sine. --.sm
( 2 case

r
'.' OC= - - OR=r
case

RC=OC-OR

r
=---r
case

r
= --(I-case)
case

In LiDRC,

DR
~ cate = RC ~ DR = RC case

r
= --e (1 - case). cotl)
cas

r case r(1- case)


= --(1-case).-..­
case sine sine

DR+RE=DE
~DE=2DR

9
2r
= -.-(1- cos G)
sin G

1
are (.6.DEC) = - x DE x RC
2

_1 2r(l-cosO).r(1-cosO)
2 sin O cosf

r 2(I-cosS)(I-
cose)
= -'----'--'-----'­
sin O.cosf

sine.cosa

4r 2 sin" ~sin2 ~ 4r 2sin" ~


= __-=2_--=2 _ 2
sin e. cos e sin e cos e /' (
AB =2r sinO
IfAB ~ 0 i.e. 2sinO ~ 0 ::::::> S ~ 0

. ar(.6.ABC)
11m -----'---'-­
• 0 oar(.6.CDE)
AB....

= r 2 tan e.sin 2 _

lim _--::-- e
0....0 4r 2 sin 0 /2

sinO.cose

, taneosin30.cosO
= I1 m - - - - , - - - ­
0.... 0 4sin 4 S /2

tanS.sin 3SocosS (
e4
=lim--~--- 4
0---+0 4 sin 0/2

e 4

. Ca~e) -( si;er·cos o
= lim
0....0
4 (sine 12) ' ­1
4

0/2 24

1.1.1 =4

4.1.1/16

10
x
Sol. Put -1- = Y => Y ~ 1 as x ~ 00
+x

1 - _1_ = Y => 1 + x = _1_ => x = _1_ -1 => x = _y_


l+x l-y· l-y l-y

e Lim[COS2lty'-I{
y->I ~l-y
Lf = e Lim(-2Sin 2lty'{
y~1
1,
(I-y)
I)

Lim_2[Sin
y->I
2"(I-y')

,,2(I_y.)2
,,2(l_y·)2
(l_y)2

.l]

_2lt2l(l-y'I)2.y2=e-2,2.2 [.: Lim x -1 = n]

e l-y x-e-l x-I

AI t, : Let limit = eL

x
Let = - - = t
l+x

9 Lim (x -1 + cos x)"


x~ X

Sol. Lim
x--+o
(x -1+COSX)~
x
(

(x-l+l-~+
1/ X
4

x + .....J1
Lim
x-->o

l 21 4!
X

• x( X
3 \ I/x
I
c.;
~~l2+4r+
x
2
II
..... J~ =e
-1/2

Llmll--+- ) => e
.--.0 2 4!

·
L un (X-l+COSX-X) -::::)e
1 -112

AIt: e x->O X X

11
10

Sol.

Lima 1/ X
- 1 a llx - 1 a 1/ x _ 1
_1_ _ + _2_ _ + .... + _n_ _
eX-O<D
1Ix 1Ix 1Ix
(
e' £n., )+(£n·2)+···+(£n.")

11 Letf(x)= sin-1(l-{x}).cos-1(l-{x}) then find LiIl}. f(x)and Li~ f(x), where {x} denotesthe fractional
..j2{x} . (l-{x}) x-e-O x--*o
part function.

. sin-l(1-{x})cos-l(l-{X})
Sol. L 1m _--'--.=-~'----_"----'-:..L._
{x} ~ 0 for x ~ 0+
x-->O' .J2{x}.(1- {x})

L' sin-1(1- {x} )sin- I ~1- (1- {x} i


xl~ .J2{x}(1- {x}) [1 - 1 - {x}2 + 2{x}}

1(1-{x})sin-1
Lim sin- J{x}(2-{x}) =2:
x-->o' .J2{x}(1- {x}) 2

{x})) [{x}~ 1]
1 1(1-
Lim sin- (1- {x})(cos-
x-->O- .J2{x}(1- {x])

12

L' ae" -bcosx+ce- x


12 Find the values of a, b & c so that x~ • =2
X.SInX

. ae" - bcos x + ce- x

Sol. L 1m . =2
x->o xsmx
2 2
a[I+~+ ]-b[I-~~+ ]+C[I-~+
4

x + ..... x - ••••• x - .....]


. I! 21 2! 4! l! 2!
Lim .
x->o X2[SI:X J
(i) a - b + c = 0 = constant

coeff. ofx = (ii) a - c = 0 => a = c

2(a + c) = 4 => a - c = 2

ja=l=c!

!b=2!

2 2

13 Lim 1 2 2 [a
2 : x - 2Sin(a Sin(7t where a is an odd integer
x-oa (a -x ) 27t) 2XJ)

2+x2
Sol. Lim 2 1 2 2 [a 7t
+cos-(a+x)-cos (-(a-x)
7t)]

x-e-a (a - x ) ax 2 2

.
Lim 2 1 2 2 [(a-xl +cos-(a+x)+(l+l)--cos-(a-x)
7t 2ax 7t ]
x->a (a - x ) ax 2 ax 2

a7t 7t ) _ . (a7t
[ cos ( -+-x -SIn - -
((a+X)7t))~+ Lim (4sin2%(a-x)~
)2 2
4 4 2 4 x--+a a - x 7t 2
. . (a+ x)
16

13
2 3 2n)
14 If L = Lim (1- x)(1- X )(1- x ) •.•..• (1- x then show that L can be equal to
x~l [(1- x)(1- x 2)(1­ x') (1- xn)f

(a)
fIn+r lfI(4r-2)
(b)
r=l r n! r=1

(c) The sum of the coefficients of two middle terms in the expansion of (1 + x)2n-l.
(d) The coefficient of x" in the expansion of(l + x?n.

Sol.

l_x n +1 l_x n+2 l_x n +3 (1-x 2n )


• 2 • 3 + .
I-x I-x I-x (1-x n )

divide by (l - x) in N" and Dr both-»

Cn + 1) (n + 2) 2n = 2n! = 2nC
, I
1.2.3....n n!n! n
r (
2.6.10 .....(4n - 2) 2n[1.3.5 .....(2n -1)]
( a) (b) = ----"----'-------'-"­
n! n!

2n[1.3.5 .....2n -1] [2.4.6......2n] 2n!


=
n!.2 n.n! n!n!
( c) 2n-Icn + 2n- 1cn+l
(d) 2nc
. n

. l-x+lnx
15 Evaluate, LIm - - - ­
x-e-l 1+cosnx
2h2
L' 1- x + {;nx
II L'Im-----,--'--------'~-
h+£n(1-£h)/n
Sol. x~rp 1+ cos nx h-+O 1- cos nfu
n 2£h 2
put x = 1- h (
Lim _h_+_£_n--O.(I_-_£_h,,---)

fh-+O 1 + cos n(1- £h)

. h+£h(1-£h)
L 1m --"":"'---::--'-
fh-+O 2' 2 n £h
sm­
2

expanding log (l-£h) ---+

. £h+[-h- ~ - ~ . .J Lim ,-~f


Lim
eh-+O (n
.
2sm - -
£h) 2
= n . (nh)
-.2sm -
2 2
ih-+O 2
n 2

2
14
16
·
LIm
roo exp( xIn(l + a: )) - exp( xln(l + b:))
LIIDlt----------­
j
y~O <-400 Y

Sol.

• erJ::! - eby • [e(a-b)y -1]


LIm =Llm .eby
y-40 y y-40 Y
=a-b

17 L et "0= 2 cos6'
1t I
and x n = -V2+xn_I' n= 1, 2 , 3, , fiIIId Lim
n-oec
~
2(n+l).-i-r>« 0

~
8 Of 8=­
Sol. xl = 2+ 2cos- = 2cos­ I
1t
6 2 6

8 8
x 2 = 2 cos '4 ~= 2 cos Y

1t 1t
= 28 = 2x-=­
6 3

..

18
· [.en (1 + X)IH
LIm __ 1]
x~ x2 X

· .en(1 + x)!"
Sol. LIm --'-------'­
2
X-40 x X

15
(l+X)[~_~+x
3
Lim .•.•.. ]-1
HO x 1 2 3
x

1 1
=--+1 =­
2 2

19 Let L = IT
00

1- 2
4
; M = IT
00 [
n ­
3
-3-
1J
n=3
(
n )
n=2 n +1

then find the value ofL-I + M-1 + N-I.

Sol. L= (1- 3~)(1- :2)(1-;) .

(1-~) (1-1) (1-~) (1-~) (1+~) (1+1) -(1+~)

1 2 3 4

-x-x-x­
3 4 5 6 . (%)(~) (I)
= lx1=(~)

2 3 -1 33 -1 43-1

M = -3-·-3-·-3-········
2 +1 3 +1 4 +1
2
-­..­
(n -1)(n + n + 1) = (2 -1. 3 -1. 4 -1. 5 -1 .....)

(n+1)(n 2-n+1) 2+1 3+1 4+1 5+1

(2 2+2+132+3+11

r
l2 2 -2+ 32 -3+ 1)"····

· ~)-v
102 n 2+n+1 2 ,~~
=> . =
(n + l)n 3 3 (b _
i' -FM" ...('Nr~ tl // ¥
20 A circular arc of radius 1 subtends an angle o~ians, 0 < x <
the figure. The point C is the intersection of the two tangent lines at A & B. Let
T(x) be the area of triangle ABC & let Sex) be the area of the shaded region.
Compute:
(a) T(x) (b) Sex) &
%as shown in

(c) the limit of T(x) as x ~ o.


f)c
o 1 A
Sex)

16

2sin(7t
Sol. Act: ar (L\ABC) = ..!...e = x)
2

1 2 1 2 •
ar (L\ABC) = -r x--r smx
2 2

x
.e=rtan­
2

1 2 X •
T(x) -tan -smx
_=2 2
Sex) !(x-sinx)

1 1
(a) ar(L\ABC) = T(x) =2".AB.CD = 2" .2r sin 2"' rsec2"- rcos2"
x ( x X)

• 2 X • 3 X
sm - sm ­
= r2 sin ~ __2_ = r 2 _ _2_
2 x x
cos- cos­
2 2

1 2 X .
= -tan -smx ('.: r = 1)
2 2
1 1. =-
(b) Sex)= -x--smx 1(x-smx
. )
222
I I
(area of ar c = 2" r2el area of L\ = 2" b C sin A)

1 2X. 2 X •
-tan -.smx tan -.smx
(c) Lim 2 2 Lim 23
X-40 1 . HO ( x "1
-(x-smx) x-lx-3T+ .....)
( 2
x
tan 2
=> Lim -----.i. sin x = ~ =i
HO x2 1 4 2
4- x­
4 3!

17
n
21 Letf(x) = Lim ~)n-I sin ' ~ and g (x) = x - 4f(x). Evaluate Lim(l + g(x))cotx .
n-+co n=1 3n x--+o
3x
. 3 3sinx-4sin
Sol. sm x = 4 sin 3x = 3sin x - 4 sin 3 x

-1 ~
L..J (3 sm--smx
. x . ) + 3(3' x . 3 2"
sm2"-sm x) + 32(3' x . 3 3
x)
sm 3-sm
4 ne.l 3 3 3 3 3

-1 In 3n-1sm.--smx
' X •
4 n=l 3n ­ 1

1. XSin(~)
3 n


1

- LIm -smx

4 n....cc x

r 3n

..!..[x-sinx]

=g(x)=x-4 [~(X-SinX)]

g(x) = sin x \~

y .....) o

22 If fen, 8)= Ull-tan J, 2


; then compute ~l~ f(n,8)
Sol. f(n,O)=rr(1-tan2~) l( use tan 28 = _2_t_an---:8_)
r=1 2 I-tan 28

8 8 8

2tan- 2tan-2 2tan­3

_ 2 2 2
---.
tan 8
tan- tan­
8'
2
8
2

2
.
(

8tan­n

Lim 2
8

n....co {\ 8 tan 8

tanu-s­n
2

./cos 2x + (1 + 3x)1/3 _ 3 4cos 3 x -[n(1 + x)4

L= Lim V 2 4

x--+o X

If L = alb where 'a' and 'b' are relatively primes find (a + b).

18

cos2x+(1 +3xy/3 _ 3 4cos 3 x-fn(1 + xt


Sol. 2 4
Lim---'-----------"=----------'------'---­
x-->o x

2 t t
24 j
Lim [COSh (1t X))X where cosh t = e + e­
x~oo cos (1tjx) 2

Sol. e' =1+t+-+-


e t
3
.
2! 3!

1t
put -= t
x

L' ( e1+e-1\"1 'I 1z


1!.1J1l 2 cos t )
(
. (e1+e-t-'+'-2costV "')
Ltml
1-+0 2ees t t2)l­
e

2+2I' / 2!+2S -2COSlf '\


L· ~ "
I~[ 2~st t»
e

Lim(el+e-I-2+2(1-COSI»).'C
t--+O 2cost t2
(II) Alt: e
( ,,' . e'+e- I-2 4siIll/2
2~~-I'-+-I'­
e
e,,'/2(1+1) = e"

25 Through a point A on a circle, a chord AP is drawn & on the tangent at A a point T is taken such that
AT = AP. IfTP produced meet the diameter through A at Q, prove that the limiting value ofAQ when P
moves upto A is double the diameter ofthe circle.

when P~A AQ
Sol. T~A sin(1t -8)

1t-28 ~ 0

AQ = fsin8 ,.,(1)
cos8

19
AP = R= 2r cos (28 - ~) = 2r sin 28 when P--)A

2rsi028.si08 2

AQ =
cos
8 = 4r sin 8

Alt :- gemetrically:­

AQ=AS + SQ

in I1TAQ ~ LQ ="2

io I1PSQ--)

LQ=LP = Q

SQ=PS

as P --) A ~ SP = SA = 2r y-ISQ ~ AQ = AS + SQ = 2r + 2r = 4r
(
26 Using Sandwich theorem, evaluate

.
[ 1 1
~~~ j;;2 + ~02 +1 + ~02 +2 +
. 1 1
+ ~02 +20

]
111

Sol. - - 2< - -2< - ­ 2

1+0 -1+0 -1+0

2 2 2

--<2
- - 2< - ­ 2

0+n - 2+0 -1+0

3 3 3

--< --<-­
0+n 2 - 0+0 2 -1+0 2

n(n+l) S
---''------,- 0(0+1)
< 0 < ----'--"'­ (
2
2(0+0 ) - -2(1+0 2 )

1 1 1

(a) -J02 + 20 S J;;2 S R

20
---,==== <
S0 <20­
-J0 2+20
- - R
t = 2

20

27
(1
If L = Lim , -
1
r:-?"
x~o \In(1+x) In(x+-vI+x 2 ) ;
I
then find the value of
L+I53
L .

Sol. L-Lim[_1-
- HO en(1+ x)
1
en(x +.JI + x")
1
put x = - x (.: x ~ 0, we can put x = - x) i

I
t
L = L. [1

im
HO en(1-x)
1]
---,====­
en(.JI+x 2-x)
I'

I
f
Adding both:

1 1
I
2L= L Hfl
.1
HO en(1+ x)
+--­
1
en(1- x) en(x+.JI+x 2) - en(~(1+x2_x))
,.I
iI
i!
/

2) 2-x2)
. en(I-x { en(1+x }
2L = LIm - = 0 .. N is absolute zero
HO en(1+x)en(1-x) en(x+.JI+x 2)en(.JI+x2-x) (. r )

x 2)/en(1
= Lim en(1-2 + x). en(1- x) = +1
HO -x X (-x)

1
L=+ ­
2

1
+-+ 153 306+1
2 = =307
+­1 +1
2
(
ax 2+bx+c+e nx
28 A function f: R ~ R is defined as f(x) = lim nx where f is continuous on R. Find the
n--+oo 1+ c.e

value of a, band c.

. ax" + bx t c t e'"
Sol. f(x) = I1 m - - - - - ­
n--+oo 1 + c.e'"

ax 2 + bx + c + e"
lim ;x<O
n--+oo 1+ c.e'"
ax" + bx-r c-r e'"
lim
;x=O
n--+'" 1+ c.e"

ax? + bx + c + enx
lim .x c- O
n--+oo 1+ c.e"
21
ax" + bx+c+O nx
x < O(lim e = 0)
l+c.O n-><XJ

c+l
x=O
c+l

ax ' bx c

-nx
+-+-+1
. -=------=-----"'---
hm e e" e" x>O
l+c

(lim
h-><XJ
ehx = (0)

ax' + bx + C ; f <0
r
; " x =0
(
; x c- O
c

since f(x) is continuous function 'v' x E R

:. lim f(x) = lim f(x) = f(O)


X40+ x----:,.o­

=;> lim!=1 & Iimtaxi-r bx-r c) e J


x->o+ C x->o­

1
=;> -=1 =;>a-0+3.0+c= I
c

:. c = 1 =;>c=1
:. c = 1, a, b E R
(
14X- 5[ [xl for x> 1
29 Discuss the continuity offin [0,2] where f(x) = [[ ] ; where [x] is the greatest integer
COS1tX for x:::;; 1
not greater than x.
Sol. f(x) = cos 1tX

1 x=O
1
[cos nx] = 0 X <x:::;;- 0
2
1
-1 -<x:::;;1
2

22

5
(4x -5) l<x<­
4
14X- 5 1 ; 1<x <2 5
14x - 51 [x] = { 6 ; 4x-5 -:S:x<z
x=2 4
6 x=2
.
6
3
x =0
0 O<x:>:.!.­
2
1
-1 .!.- < x :>: 1

f(x) =
~

-(4x - 5) 1< x <


·· 5
4x - 5
5
-:>:x<2
4
2 ·i 4
4 ~
6 x =2 1'1

function dis at 0, o,.!-,


2
1,2

30 Iff(x) = x + {-x} + [x] , where [x] is the integral part & {x} is the fractional part ofx. Discuss the
continuity off in [ - 2, 2 ].
Sol. f(x) = x + {-x} + [x]
.: {x} =x-[x]
{-x} =-x- [-x]
f(x) = x + (- x - [- x] + [xj)

<
X- (- X) = 2X; X E I

f(x) = [x] - [-x] [x] _ (_ [xl - 1) = 1 - 2[x]; x ~ I

f(x) ={ 1_22~X] : :::

-4 x =-2

5 -2 < x <-1

-2 x = -1

3 -1<x<O

f(x) = 0 x=O

1 0< x < 1

2 x = 1

-1 1 < x < 2

4 x=2

23
so the function is discontinuous at all integers in [-2, 2].

31
is continuous at x = 1 but discontinuous at x == 2.
ax-b for
Find the locus of (a, b) for which the function f(x) = 3x lI
x~1
for 1 < x < 2
bx 2 - a for xz2

Sol. conti at x == 1
a-b==3 ...(1)
dis at x == 2
6:;t 4b - a
6:;t 4b -3 - b
6:;t 3b - 3
(a, b):;t (6, 3)
(x, y):;t (6,3) Ans

32 f (x) ==.
asj!lX -a tan X
.
tan X-SIn x

for x > ° (
__ In(1 + x + x 2 ) +In(1- x + x 2 )

for x < 0, if f is continuous at x == 0, find 'a'


secx-cosx

now if 9 (x) = In (2 - :) . cot (x - a) for x:;t a, a:;t 0, a> 0. If g is continuous at x = a then show that
g(e-I) =- e.
Sol. Since the function is conti at x == then °
V.F-Ix = 0 == RHLlx=o == LHLl x = 0 since the function is conti then

RHLlx=o = !~rr.f(x) f(O) == LHLlx=o == RHLlx=o

• a sinx _ a tanx
== h m - - - ­ -£na==l
x-->o· tan x - sin x

• a!llnx(asinx-tanx -1) (
== hm -----'------'­

x-->o· -l(sin x - tan x)

since g(x) conti at x = a

g(a) =limg(x)
x-->a

LHLI x--0 == x-->o


lim f(x) == lim£n(2-~)cot(x-a)
x-->a a

2
= lim £n(1 + x + x ) + £n(1- x + x 2 ) == lim_£n_(-'-2_-_-_:-'-)
X-->O- sec x - cos x x-->a tan(x - a)

24
2) 2)).cos
= lim £n((1+ x + x (1- X + x X
put x= a + h
x-->O- 1- cos 2x

putx= O-h

2 4)cosh 1
= lim £n(1 + h + h
g(a)=-­
h-tO sin" h a

= lim(~)2
smh
h-->O
(l+h2)cosh

33 Let f(x + y) = f(x) + fey) for all x , y & if the function f(x) is continuous at x = 0, then show that f(x) is
continuous at all x.
Sol. f(x + y) = f(x) + fey) V x, y E R
put x =y = 0
f(O + 0) = f(0) + f(0)
=> £to) + 2f(0)
=> f(O) = 0
since f(x) is continuous at x = 0

lim
x-->O
f(x) = f(0)

lim f(x) = lim f(x) = f(O)


=> x----.o- x-)oo+

we have to prove that f(x) is continuou V x E R

consider an orbitrary constant point x = a

so we have to prove

lim f(x) = f(a)


x-->a

LHS = limf(x) = lim f(x)


x---+a x-a40

= V~f(a+h) (putx-a=h)

= lim(f(a)+ f(h)) [',' f(x + y) = f(x) + feY)]


h-->O

25
= lim f(a) + limf(h)
h40 h40

= f(a)
\.
+ limf(h)
h~O

= f(a) + f(O)

= f(a) + 0

= f(a)

=RHS

34 Given f(x) =
f=1
± tan(~) 2f
sec(~1)
r: ; r, n E N

en (f(X) + tan -¥) - (f(X) + tan -¥f .[sin (tan~)]


g(x) = ~~~
1 + (f(X) + tan {of
n
.r' = k for x = 4" and the domain of g (x) is (0, nI2). (
where [ ] denotes the greatest integer function.

Find the value ofk, ifpossible, so that g(x) is continuous at x = 1t/4.Also state the points ofdiscontinuity

of g (x) in (0, 1(/4) , if any.

. x . x

smxcos--cosxsm- x
sinx/2 2 2 = tanx-tan-

Sol.
x x x 2

cos-Lcos x cos-s.cos x cos-v.cos x

2 2 2

x x

tan - sec x = tanx - tan­


2 2

x x x x

tan - 2 sec- = tan--tan­ 2

2 • 2 2 2
(
x x x x

tan-.sec-=tan-·
3 2 2
-tan­ 3

2 2 2 2

x x x x

tan-.sec-1 = tan--tan­
1
2 0
2 20
-2 0
-
0

f(x) = tan x -tan(; J

f(x) + tan(; ) = tan x ...(1)

using (1)

26

. €n(tan x) - (tan x)" [sin (tan %)] 1t


11m - - - - - - - = - - - - ' - - - - - - - ' ' - - = ­ x:;t:­
g(x) = n->oo 1+ (tan x t 4
1t
k x=­
4

1t
x;;t.­
4
g(x) =
1t
x=­
4

.~

f
'<..

lim
n-->oo x" =rr:IJ
x <1
x=1
x c- l
/'

1t
0 x<­
4
1t
limttan x)" = 1 x=­
n-->oo 4
1t
r:IJ x>­
4

35 Let f be continuous on the interval [0, 1] to R such that f(O) = f(1). Prove that there exists a point c in

[o,~] such that f (c) = f (c+i)


Sol. Consider a conti function

Now

g(O) = r(i) - f(O) => g(O) = r(~) - f(l)

g(~J g(l)- f(~J =>g(O)=f(1) - f(~)

since g is continuous and g(O) and g( ~) are of opposite sign hence the equation g(x) = 0 must hav~~t

least one root in[ 0, ~ J.

; xcO
36
; x c- O

where a> 0, find the value of 'a' & 'g(O)' so that the function g(x) is continuous at x = O.

LHLI x--0 = x-->o


lim gtx) RHLI = (.en2a)2
Sol. x=o 2

_r
_III!
[I-aX + xax.ena)
x2 since the function is conti (
x~ aa
put x= a-h g(O) = LHLlxo=() = RHLlxo=()
= lim[ 1- a-h- ha-h.ena) (.en(2a)/ = (.ena/
x-->o a-hh2 2 2
. [ah-I-h.ena).~~
I1m , lorm
=
x-->o 2h 0 (.en 2a + .ena) (.en 2a - .ena) = 0

= Iim[ah.ena-o-.ena).~ .en(2a2) . .en2 = 0


h~ 2h '0 Ans

.en2a2 = 0

1
LHL I = (.ena)2 2a2 = 1, a = ± ,,2
r;:;' a > 0
x=o 2 '

RHLI _ = lim g(x)


x-o x-->o+

(.en2a)2
:. g(O) = .
.2

put x = 0 + h 1(
="2 .en2·
1)2
h

28
. [(2a)h - h£n2- h£na -l),Q~ 1 t: 2
= 1h~O
1m
h
2 '
0
lorrn = -(£n,,2)
2

= Ii1m (2a)h £n2a - £na2 "-lOrrn


h-so 2h
0 c:
'0
="21(14 (£n2) 2)

= lim (2a)\£n2a)2 1 2
= -(£n2)
h~O 2 8

37 A function f:R~R satisfies the equation f(x+y)=f(x). fey) for all x,y in Rand

f(x) =1= 0 for any x in R. Let the function the differentiable at x = 0 and f'(O) = 2.
Show that fl(X) = 2f(x) for all x in R. Hence determine f(x).

Sol Given that f(x+y)=f(x).f(y) for all xER ... (1)

Putting x = y := 0 in (1) ,/we get


I.
"
" f (O){f (0) -I} := 0 => f (0) = 0 orf (0) = 1

If f(O):=O, then f(x).:=f(x+O) = f(x).f(O):= 0 for all xER


Which is not true (given f (x) =1= 0 )

So, f(O)=l

.. f'(x) = lim f(x+h)-f(x)


h~O h

= lim f(x)f(h)-f(x)
h~O h
f(h) -1
= f (x) lim-----'---''--­
h~O h

:= f (x) lim f (x) - f ( 0 ) (': f(O) =1)


h-+O h-O
(
= f(x)f'(O) = 2f(x) (': f'(O) = 2)
f'(x) := 2
f(x)
Integrating both sides w.r.t.x and taking limit 0 to x

If'(x)
-dx = .hrX2dx
f(x)

In f(x) -In f(O):= 2x Inf (x) -In 1 = 2x

In f(x) -0 = 2x f(x) = e 2x "

29
38 Letfbe a function such that f(x+f(y))=f(f(x))+f(y) v x.y e R and f(h)=h for

0< h < £ where £ > 0, then determine f'(x) and f(x).

Sol Given f(x+f(y)) = f(f(x)+f(y)) .... (1)


Putting x = y = 0 in (1), then
f(O + f(O)) = f(f(O)) + f(O) ~ f(f(O)) = f(f(O)) +f(O)

.. f(O)=O .... (2)

f(x + h) - f(x)
Now f'(x) = lim~-!...-~ (for 0 < h < E)
h-->O h

= lim f(h+x)-f(x)
h-->O h

= lim f(f(h))
(form (1))

h-->O h

= lim f(h) (-:f(h)=h) (

h-->O h

=lim~=l
h-->°h
(-:f(h)=h)

Integrating both sides with limites 0 to x then f(x) = x


f'(x)=1.

-2 , -3 $X $0
39 Let f(x)= { x-2 , 0<x$3 ,where g(x)=f(lxl)+lf(x)l· Test the differentiability of

g(x) in the interval (-3,3).


Sol From the given function
2+2 for -3 $X $0
f(lxl)={-X-2 for -3$x$0 and !f(X)I={_X for 0 < x s 2
x - 2 for 0 < x s 3 x- 2 for 2 < x s 3 (
g(x) = f(lxl)+lf(x)1

-x for -3 $x$O
- 0 for 0 <x s 2
{
2x-4 for 2<x$3
Check the differentiability
At

x =0: Lg,(0) = lim g (0 - h) - g ( 0)


h-->O -h

-(O-h)-O
= lim =-1
h-->O -h

30

Rg'(O) = limg(O + h) - g(O)


h--+O h

( 0 - 0)
= lim
h--+O h
=a

Lg'(O) * Rg'(O)
.. g (x) is not differentiable at x= a
Check at

x=2: Lg' (2) = limg ( 2 - h) - g ( 2)


h--+O -h
0-0
=lim-=O
h--+O -h

and Rg'(2) = lim g(2 + h) -g(2)


h--+O h

-
/ =lim 2(2+h)-4-0 =2
/'

~.
• h--+O h

Lg'(2) * Rg'(2)
Hence g (x)' is not differentiable at x = 2.
Graphical method:
-2 -3::; x s a
.. f (x) = {x_ 2 ; a < x::; 3
Graph of f(x) :

31
Graph of f(lx!) :

Graph of If(x)! :

y
~

(
1

-3 -2 -1

-1

Graph of g(x)=lf(x)l+f(lx!) :

-3 -2 -1

It is clear from the graph that g(x) is not differentiable at x = 0 and 2.

40 Let f:R~R is areal valued function v x.y e R such that If(x)-f(y)I~lx-yI3.

Prove that h(x) = Jf(x)dx is continuous function of x \;f x E R.

Sol Since If(x)-f(y)I~lx-yI3 x:;t:y

.. f(x)-f(y) ~lx_yI2
x-y
Taking lim as y ~ x, we get

32

lim f(x)-f(y) ~limlx-yI2


y-->x X - Y y-->x

lim f(x)-f(y) ~llim(x-y)21


y-->x X- Y y-->x

=> If'(x)\ ~ 0 => jf'(x)I=O (':If'(x)lzO)

f'(x) =0 => f (x) = c (constant)

h(x)= Jf(x)dx= JCdx=cx+d where d is constant of integration.

hex) is a linear function of x which is continuous for all x E R.

41 Let f(x;YJ= f(X);f(Y) for all real x and y.1f f'(O) exists and equals -1 and f(O)=l,

then find f(2).


,)

Sol
.
Since
'~(x+YJ_f(X)+f(Y)
-2­ - 2 .... (1)

.. f' (x) = limf (x + h) - f (x) = lim


f( 2x + 2h) f(2x +
2 - -2­
0)
~ h ~ h

f (2x) + f(2h) f(2x) + f (0)


= lim- - - - -2- ' ' ' ' - - - - - - -2= - - ­ [ from (1) ]
h-->O h

= lim f (2h) - f (0)


h-->O 2h - 0
= f'(O)
=-1 'if xER ( given)
Integrating, we get f(x)=-x+c
(
Putting x=O, then f(O)=O+c=l ( given)
.. c = 1 then f(x) = 1- x f(2) = 1- 2 =-1
Graphical method :
Suppose A(x,f(x)) and B(y,f(Y)) be any two points on the curve y = f(x).

33
B
y

X+ Y f(x) + f(Y)]
If M is the mid-point of AB then co-ordinates of Mare [ -2-' 2

y
x+
According to the graph, co-ordinates of Pare ( -2-,f (x+YJJ
-2­ and PL >ML .
(
~ f(x ;Y J > f(X);f(Y)

x + YJ f(x)+f(Y)
But given f ( -2­ = 2 which is possible when P ~ M

i.e. P lies on AB. Hence Y= f (x) must be a linear function.


Let f (x) = ax + b ~ f(O)=O+b=l (given)
and f'(x) =a ~ f'(O)=a=-l (given)
.. f(x)=-x+1 f(2) = -2+ 1 =-1.

42 Let f(x~YJ= f(X)~f(Y) V x,YER;n:;t:O,2 and if f'(O)=k (A finite quantity) then

prove that f(x) = kx V XE R. (

Sol Given f(X~YJ=f(X)~f(Y) ... (1)

Putting x = Y= 0, we get (n-2)f(0) = °


.. f(O)=O (.: n-2:;t:O)

f(nx + nh) _f(nx+O)


f' (x) = limf(x +-h)-'- f (x) = lim n n
~o h ~o h

34

f(nx)+f(nh) _ f(nx)+f(O)
n n
= lim--=-------------=-"-­ [ from (1) ]
h-->O h

= lim ---.of(_nh-"-)-_f~(0.. .:. .)


h-+O nh-O
=> I'{x] = k

On integrating we get f(x)=kx+c

Putting x = 0, then f(O) = 0+ C = 0 (: f(O) = 0)

.. c = 0 then f(x) = kx.

x + Y) 2+f(x)+f(y)
43 If f ( -3- = 3 for all real x and y and fl(2) = 2 then determine y = f(x).

5 o 1r
1 f(x+ y)= 2+f(X~+f(Y)
0 :

... (1)
Differentiating both sides w.r.t. x treating y as constant,

then f'(x;y)(±)= 2+f'~X)+0

Now replacing x by 0 and y by 3x, then

f'(x) = f'(O) = c ( say)

At x = 2, f'(2)=c=2 (given)

fl(X)=2

On integrating we get f (x) = 2x + d

Putting x=O, then f(0)=0+d=2 [ from (1) ]

., f(x)=2x+2

Hence y = 2x + 2.

X+ 2y ) _ f(x) + 2f (v)
44 If f ( - 3 - - 3 "i x,YER and f'(O)=l; prove that f(x) is continuous for

all x E R.

Sol o. f(X+ Y)=f(X)+32f(y)


32
Differentiating both sides w.r.t. x treating y as constant

f 1 ( x + 2y
3
).1.3 = fl(X)+3
0

and replacing x by 0 and y by 23x


then f'(x) = f'(O) = 1 ( given)
35
On integrating, we get
f(x)=x+d,d is constant of integration which is linear function in x and hence it is
always continuous function for all x.

45 If f(X)+f(Y)=f(X+Y) for all x,YER and xy v I and limf(x) =2, find


1- xy x--+o X
f(J3) and

f'( -2).

Sol Given f(X)+f(Y)=f(X+Y)


l-xy

Putting x = O,Y = 0, we get f(O) = 0 .. ,(1)

And putting y = -x, we get f(x)+ f(-x) = f(O) = 0

.. f (x) = -f ( -x) ... (2)

Now f'(x) = lim f(x+h)-f(x)


h--+O h
..r'.
= lim f(x+h)+f(-x) (
h--+O h

=2·-­ 2
1 .: lim f(x) = 2J
1+x [ x--+o X

2
=
1+x 2
.. f(x)=2tan-1x+c or f(0)=2tan-10+c=0
=> O=O+c c=O
then f(x) = 2 tan' x (

f( J3) = 2 tan" (J3) = 23 1t and

46 Let f(x+Y)=f(x)+f(y)+2xy-1 for all X,YER. If f(x) is differentiable and f'(O)=sin~


then prove that f(x»O V XER.
Sol Given f(x+y)=f(x)+f(y)+2xy-1 V x,YER ... (1)
Putting x = Y= 0 in (1), we get
f(O) = 1 ... (2)

f '(x) = lim f(x + h) - f(x)


h--+O h
36

= lim f (x) + f (h) + 2xh -1- f (x)


h->O h

=lim f(h)+2xh-l
h->O h

· f (h) -1 + I'1m ( ­ 2xh J


= Ih->O
1m
h h->O h

f(h)-f(O)
= lim + lim(2x)
h->O h h->O

= f'(O)+2x

= sin ] + 2x (: f(O) = sin ]


Integrating both sides w.r.t. x and taking limit 0 to x, then

J:f'(x)dx = J:(sin <j>+ 2x)dx

=> f(x)-f(O)=xsin<j>+x 2
2+xsin<j>+1
=> f(x)=x (-: f(O) =1)
Here coefficient of x 2 is 1 > 0 and Discriminant
o = sin 2 <j> - 4 <°.

Hence it is clear from graph f(x) > ° V XE R.

47 Let f be a one-one function such that f(x)f(y)+2=f(x)+f(Y)+f(xy) V x,YER-{O}

and f(O)=1,f'(1)=2 then prove that 3 Jf(x)dx-x(f(x)+2) is constant.

Sol We have f(x)f(Y)+ 2 = f(x)+f(Y)+f(xy) ... (1)


Putting x = 1 and Y= 1, we get

(f(1))2 + 2 = 3f(1)

( f(l) = 1,2 => f(1)=2 ... (2)


f(1):;t:1 (': f(O) = 1 and f is one-one function)
1
In (1), replacing Y by ­
x

f(x )f(~) + 2 = f(x) + f(~) + f(l)

f(X)f(~J =f(X)+f(~J (':f(1)=2)

f (x) = 1 ± x n (x E N)

=> f'(x) = ±nx n - 1 => f'(1)=±n=2


Taking positive sign => n=2 then f(x)=1+x 2
37
Now, 3 Jf(x)dx-x(f(x)+2)
2)dx-x(1+x 2
=3 J(1+x +2)

=3(x+ ~J+C-3X-X3
= C= constant.

48 If e-XYf(xy) = e-Xf(x) + e-Yf(y) V x,y E W, and f'(l) == e, determine f(x).

Sol Given e-XYf(xy) ==e-Xf(x)+e-Yf(y) (1)

Putting x==y==l in (1) we get f(l)==O (2)

Now, f'(x) =limf(x+h)-f(x)


h.....O h

= lim----,---f
( x_(1 +_~)--'-..-)-_f(x_.1) ,rl
h.....O h (

. ehf( x) + e X+h-I-~f( 1 + ~) - f(x) - eX-If (1)


= hm-----~------1.-----
h.....O h

hJ
e h.-!'.Xf ( 1 +-
" (eh
= f( x )11m - + e (x-I) n
- -lJ 1m h x (:f(l)=O)
h.....O h h.....O
x·­
X

fl(l)
= f(x).l + e X-I . ­
x
(
x-I
=f(X)+~ (':f'(1)==e)
x
eX

f'(x) = f(x) +­
X

=> ~(e-Xf(x)) =!
dx x
O~ in.tegrating we have e-Xf(x) = In x+c at x=l,c=O

49 Let f:R~R, such that f'(O)=l

and f (x + v) = f (x) + f (v) + e X+ Y(x + v)- xe" - ye Y+ 2xy V x, Y E R then determine f (x).

38

Sol Given f(x + Y) = f(x) + f(y) + e X+ Y (x + y) -xe x - ye Y + 2xy ... (1)

Putting x = y = 0, we get f(O) = 0 ... (2)

= lim--of(,--x+_h---"-).;f--,-(x---!.. .)
Now, f'(x)
h--70 h
h
= lim f(x)+f(h)+ex+h(x+h)-xe x -he +2xh-f(x)
h--70 h

f(h) (eh -1) }


=lim -+xe X +e x+h _e h +2x
h--70{ h h

= f I (0) + xe".1 + eX -1 + 2x
= 1 + xe" + e" + 2x -1
= xe" +e x +2x
Integrating both sides w.r.t. x with limit 0 to x
.. f(x)-f(0)=xe x-e X + e X + x 2
f (x) - 0 = xe" + x2
Hence f(x)=x 2+xe x
50 Let f(xy)=xf(y)+yf(x) for all x,YER+ and f(x) be differentiable in (0,00) then

determine f(x).

Sol Given f(xy) = xf(y)+yf(x)


Differentiating both sides w.r.t. x treating y as constant,
f'(xy).y = f(y) + yf'(x)

Putting y = x and x = 1, then f'(x ).x = f( x) + xf '[I]

xf'(x) -f(x) f '[I]


x2 =-x­
=> ~[f(X)J = f'(l)
dx x x

Integrating both sides w.r.t. x taking limit 1 to x,

f(x) _ f(l) =f'(1){lnx-In1}


x 1

f(x) -O=f'(l)lnx (:f(l)=O)


x

Hence, f(x) = f'(l)(x ln x).

51 Let f(xY)=f(x)f(y) \j x,YER and f is differentiable at x e I such that f'(l)=l also

f(l)"* 0 then show that f is differentiable for all x"* O. Hence, determine f(x).
Sol Given f(xy) = f(x)f(y)
39
Putting x = Y= 1 then we get f(l) = 1.
Differentiating both sides w.r. t. x treating Y as constant,
f'(xy).y = f'(x)f(Y)
Replacing Y by x and x by 1, then
f'(x).x = f'(l)f(x)

~ f'(x) = f(x)f'(l) = f(x) (-: f'(l) = 1)


x x

f'(x) 1
~ f(x) =;­
Integrating both sides w.r.t. x and taking limit 1 to x, then

r
1
f'(x) dx =
f(x)
r
1 X
.!.dx

~ In f (x) -In f (1) = In x -In 1 (.: f (1) = 1)


~ In f (x) ­ 0 = In x - 0 .. f (x) = x. }-'
(
52 If 2f(x) = f(XY)+f(~) for all X,Y E W,f(l) = 0 and f'(I) = 1, then find f(~) and f'(2).

Sol Given 2f(x) = f(xy)+t(~) ... (1)

Replacing x by y and y by x in (1), then

2f(y) = f(xy) +t(~) ... (2)

Subtract (2) from (1), we get

2{f(X)-f(Y)}=f(~J-f(~J ... (3)

Putting x e I in (1) then 2f(I)=f(Y)+f(~)=O (-: f(I)=O)


(
.. f(y)=-t(~) .. f(~J=-f(~) ... (4)

Now from (3) and (4), we get

2{f(x)-f(y)} = 2f(~)

or f(x)-f(y) = t(~ J ... (5)

Now, f'(x) =limf(x+h)-f(x)


h.....O h

40
[ From (5) ]

= lim
( h)
f 1+- 1 1
x =-f '(l)=- {-: fl(l) = 1}
h-->O h x x
-·x
x

f l (2) = .!
2
and f (x) = In x + In c for x = 1, and f (1) = In 1 + In c
=> O=O+Inc .. ln c e O
then f(x) = In x f(e)=Ine=l.

x 1- 1
53 Suppose p(x)=aO+alx+a2x2+ ... +anxn. If \p(x)!::;le - 1 for all x z O, prove that
la1 + 2a 2 + ... + nanl ::; 1.
(
2
Sol Given p(x) = a o +a1x +a 2x + ... + a.x"

=> p'[I] = a 1 + 2a 2 + ... + nan ...(1)


11
Now, Ip(l)\::; le - -11

= leO -11 = 11-11 = 0

=> Ip(l)I::;O => p(l)=O (·:/p(l)1 ~ 0)


As Ip(x)I::;lex-1-11

we get Ip(1+h)I::;leh-11 Vh>-l,h;t:O


h
=> Ip(l + h) -p(l)l::; le -11 (-: p(l) = 0)

=> P(I+h~-P(l) ::;le ; 11


h

Taking limit as h ~ 0, then

=> lim p(l+h)-p(l) ::;liml


h-->O h h-e-O
eh
h
-11
h-
· p(l+h)-p(l)
Iim ::;
ti1m-­
' e 1 1

h-->O h h-->O h

[ from (1) ]

41
54 Let f(~)= f(X);(Y) for all real x and y. If f(l)=f'(l), show that f(x)+f(l-x)=

constant, for all non-zero real x.

. f(XY) _ f(x)f(Y)
Sol Given 2 - 2
Replacing x by 2x and Y by 1, we get
2f(x) = f(2x )f(l) ... (1)
and,

... (2)

now,

f (x)
I = lim f (x + h)- f (x)
h....O h l'
(

f(2X)f(1+~)
2 x -f(x) [ from (2) ]
=lim--=----­
h....O h

. f(2X)f(1 - 2f(x) +~)


== hm--~------<---
h....O 2h

. f(2x )f(l +~) - f(2x)f(1) [ from (1) ]


=hm--~~----
h....O 2h

f(2x).
= --hm----"---~-
t(l+~)-f(l)
2 h....O h
x·­
x

= f(2x) .fl(l)
2x

= 2f(x) 'f'(l) = f(x)


f(1).2x X (': f '[I] = f(l))

fl(X) 1
==f(x)==~
Integrating both sides w.r.t. x, we get

42

Inf (x) = In x + In c
f(x)=cx (c is constant> 0)
f(x) +f(l-x) = cx+ c(l-x) = ex + c -cx = c = constant.

3 2
.
55 Let f(x) = x _x +x+ 1 and g(x) =tax{f(t): 0 s t s x}....D s xs 1Y-x,1 < x s 2.
€::..

Sol

( f(x) is strictly increasing in (0,2)


maximum value of f(t) in 0 s t s x is f(x)

g(x)={f(X) ,Osxs1
3-x , 1<xs2
3 2
= {x - x + X+1 ,Os x s 1
3-x 1<xs2

Graph of g(x) :

Clearly, 9 (x) is continuous for all x E (0,2) and differentiable at all points in this
interval except x = 1.

3 2 () {min f (t) : 0 s t s x ,Os x s 6


56 Let f (x) = x - 9x +15x + 6, and 9 x = x -18 , x > 6 ,then draw the

graph of 9 (x) and discuss the continuity and differentiability of 9 (x) .


Sol .. f(x) = x 3 -9x 2 +15x +6,
43
.. f'(x) = 3x 2 -18x + 15 = 3(x2 - 6x + 5) = 3(x -l)(x -5)

If f'(x»O then xE(-00,l)u(5,00)


---,

and if f'(x)<O then xE(1,5)

Hence f(x) is increasing in


\'-----_------'1
(-00,1)u(5,00) and decreasing in (1,5).
Now, f(x)=6 ~ x3-9x2+15x+6=6

x(x 2 -9x+15) = 0

x=O
,
9±m
2

x=O
,
9-m

2 (
X :;f: 9+m
2'
'; 9-m
2
>6J
,/
(
6 o:s;x< 9-m
2
9-ffi
g(x)= x3-9x 2+15x+6 ---:s;x:s;6
2
x-18 x>6

Graph of g(x) :

Clearly g(x) is continuous in [0,(0) and differentiable at all points in this interval

other than
9-J2i and 6.
2

44

.
b sm _l(X+C)
--
1
--<x<O
2 ' 2
1 x =0 , If f (x) is differentiable at x = O. Find the
57 Let f{x) =
2
eax / 2 -1 1
O<x<­
x 2

value of a also prove that 64b 2 = 4 - c 2 •


ah
e 2 -1 1
Sol f{O+h)-f{O) ---­
Rf'{O) = lim = lim h 2
h-->O h h-->O h

.r'
(
a 1
at h~O numerator must be = 0, then 2.1 - "2 = 0

a=l

... (1)
=>

2(e ~-l)+h
Replacing h by -h then P = lim--'----.".-"'--- .,. (2)
h-->O 2h 2
h h h
e2 + e 2 - 2 e h - 2e2 + 1
Adding (1) and (2) then 2P = lim
h-->O h2 = lim
h-->O 2 ­
h

h e2

P=! => Rf'(O) =! ... (3)


8 8

45

Lf'(O)=limf(O-h)-f(O)=lim
b .
Sin
-l(-h+cJ
-2- -2
1

h-e-O -h h~O -h
Now, at h ~ 0 numerator must be = 0

..
. -1
b Sin (c2J-2=
1 0

then,

. _l(C-h) . -l(C)
U'(O) = blim Sin 2 -Sin 2
h~O -h

{(_C;
= blim----"-sm-'
h~O
h)JP:}_(1-
:_ - ~)(_1-
(c ;-------<-h
-h

n}

-%~1-(
"blrrn sin-'{( c;h )JP:} c;h ))}
(

,~O (C;h)JP:)_% [1-( c;r'JJ


{U~)R-%~}
-h

. {(T)R-%R~}{(T)R+%~}
~-b~ h{(C;h)R-%~}
{

. (Tn1-~)-~{1-(C~rJ

~-bl~ h{(C;h)f~~}%~}

=-blim (2c-h)(-h)
,~ 4h{(C;h)fl}% {1-(<C;hJ}}

46

2bc b

+A}-zJP:} ... (5)

From (3) and (5),

S-ZA
1 b

58 Let a E R. Prove that a function f: R ~ R is differentiable at x:= a if and only if there


is a function g:R~R which is continuous at a and satisfies f(x)-f(a):=g(x)(x-a)
for all a E R.
Sol Let f: R ~ R be differentiable at x = a E R, then

. f(x)-f(a) ,

hm ( x-a ):= f (a) exists and finite


.
( ~a

i.e. U'(a)=Rf'(a):=f'(a)

lim f(x)-f(a) = lim f(x)-f(a) :=f'(a)


x-->a- (x - a) x-->a+ ( X- a)

!~_ 9 (x):= !~~+ 9(x) = f' (a) {'.'f (x) - f(a ) := 9 (x) (x - a)} ...(1)
1 •f(x)-f(a)
Again f (a) := x-->a
hm ( X - a )

:= lim 9 (x) = 9 (a )
x-->a

From (1) and (2), we get !~~_g(x):= !~~+g(x):=g(a)

L.H.L=R.H.L=V.F.

9 (x) is continuous function at x = a E R.

(
59 Let 9 (x) := 0 if -e s x < 1

:={1+~sin(lnx2n)} if Ls x s e,

where {} denotes the fractional part function and

f(x):= xg(x) for g(x):= l+~sin(ln x


2n)

:= x(g(x) + 1) otherwise
Discuss the continuity and differentiability of f( x) over its domain.

Sol Given g(x):= {l+~sin(ln x 2n)} for 1:s; x s; e

47
= 0 for - e :s:: x < 1

g(x) = l+~sin(ln X21t)-[1+~sin(ln x


21t)]

i.e.,

= ~sin(ln x [~sin(ln x 21t)],1:S:: x :s:: e


21t)
-

= O,-e:s:: x <1
where [.] denotes the greatest integer function.
consider: 1 :s:: x :s:: e

=> O:S::ln(x 21t):S::27t

Case I: If O:S::ln(x
21t):S::7t i.e. l:s::x:s::~ then O:s::sin(1n(x 21t)):S::1

=> o, ~sin(ln(x21t)):s:: ~ .. [ ~ sin(In(x21t ))] = 0


r'
.. g(x) = ~sin(lnx21t) for l:s::x:s::~
(

Case II :

- ~ :s:: ~ sin (In(x21t )) < 0

.. g(x)=l+ ~sin(ln(x21t)) for ~ <x <e

Case III: If In(x 21t)=27t => x=e => g(x)={l}=O


Combining all cases, we get

f (x) = x (1 + ~ sin(In(x21t )) J for~ <x <e

= x(1+ ~ sin(ln(x21t))J for1:s::x:s::~


(
=x(1+0) for-e:S:: x < 1

=x(1+0) for x = e

f (x) = x (1 + ~ sin(In(x )))


21t
for 1:s:: x s; e

=x
.. f is differentiable in (-e, 1) and (1,e)
Check the differentiable of f(x) at x = 1.

Lf'(l) = lim f(l-h) -f(l)


h--.O -h

= lim (1- h) -1 = 1

h--.O -h

48

and Rf'(l) = limf(l+h)-f(l)


h-tO h

. (l+h){l+~sin(ln(l+ht'))-l
= lim-----O...---------'--­
h-tO h

(1 + h) (In(l+h) 2,,)
h+--sin
= lim 3
h-tO h

= lim[1 + _(1+_h) _sin--,-{In_(l_+_h)2--,-"}]


. h-tO 3 h

. (l+h). sin(ln(1+h)2")
= 1 + lim lim --'------'­
h-tO 3 h-tO h

. (l+h). sin{21tln(1+h)} 21tln(1+h)


=l+hm lim .-~------!....
( h-tO 3 h-tO 21t In(1 + h) h

= 1 + (_1;_0}1.21t.1

21t
=1+-·
3
Thus f is not differentiable at x = 1.
Hence f is continuous and differentiable for all x E domain of except not
differentiable at x = 1.

60 Suppose that f and g are non-constant differentiable real valued functions on R.


If for every x, YE R,f(x + Y) = f(x )f(Y) - g(x)g(y), g(x + v) = g(x)f(Y) +f(x)g(y) and

f'(O) = 0 then prove that {f(X)}2 +{g(X)}2 = 1 V XE R.

'() I' f(x + h) - f(x) \. f(x + h) - f(x + 0)


Sol We have f x = rm = im ---'---<----'--------'­
( h-tO h h-tO h

= lim{f(x)f(h)-g(x)g(h)} -{f(x)f(O)-g(x)g(O)}
h-tO h

. f(x)(f(h)-f(O)) . g(x)(g(h)-g(O))
= lim - lim-----'--~--=-----.:...---,----'--'-'-
h-tO (h-O) h-tO (h-O)

= f(x)f'(O) - g(x)g'(O)

= O-g(x)g'(O) (': f'(O) = 0)


f'(x)=-g(x)g'(O) .... (1)

and g '()
x = I'im g (x + h) - g(x) = I'rm ------0.-_--'------'-_--'­
g (x + h) - g (x + 0)
h-tO h h-tO h

49
= lim {g (x )f (h) + f(x )g(h)} ­ {g(x)f(O) + f(x )g(O)}
h~O h

= g(x ) lim f(h) ­ f(O) + f(x ) lim g(h) ­ g(O)


h~O h- 0 h~O h- 0
= g(x)f'(O) + f(x)g'(O)

=O+f(x)g'(O) (-: f'(O) =0)


= f(x)g'(O) .... (2)
Multiplying (l) by f(x) and (2) by g(x) and adding we get
f(x)f'(x) +g(x)g'(x) = 0

or 2f(x)f'(x) + 2g(x)g'(x) = 0 on integrating we get

..... (3)
Putting x = 0, y = 0 in the given equation then
-I
f(O) = {f(O)}2 _{g(O)}2 and g(O) = 2f(0)g(0) F
(
1
or g(O){2f(O) -I} = 0 or g(O) = 0 or f(O) ="2

If g(O) = 0, then f(O) = (£(0))2 - 0 orf(O) = 1

1 1 (1)2
and for f(O) ="2'"2= "2 _(g(O))2

~ (g(0))2=_~ (Impossible)

Hence f(O) = 1 and g(O) = 0 from (3), {f(O)t + {g(0)}2 =C


~ 1+0=c .. c=1
Hence {f(X)}2 +{g(X)}2 =1.

61 Let f(x) be a real valued function not identically zero such that (
f(x+yn)=f(x)+{f(y)r;V x.v e R{where n is odd natural number> 1) and f'(O)~O.

Find out the values of f'(10) and f(5).

Sol Given that f(x+yn)=f(x)+(f(y)f

Putting x = y = 0 ~ f(O) = 0

f '(0) = lim f (0 + h) - f (0 )
h~O h

=limf(h)-O
h~O h

50

= lim f (h) = A(say) ... (1)


h....O h

Also,

f 0) = lim f (0 + h) - f ( a)

I (

h....O h

. f(O+(hllnf)-f(O)
= lim-----'----------'---­
h....O h

=A n [from (1)]
From (1) and (2), A= An ,r'
(
A = -1,0,1 (': n is odd and A E R)

f'(O) ~ a (':A;i;-l)

f'(O) = 0,1

f(x + h) - f(x)
Agai n f I ( x) = lim ----'--~'-----~
h....O h

= lim f(X+(h1/nf)-f(x)
h....O h

. f(x)+(f(hlln)f -f(x)
= lim --'----'---------'-'----­
h....O h

For A = O,f'(x) = a

On intergrating we get f(x) = c

At x = 0, f (0) = c = a (:f(O)=O)

f(X) = a

which is impossible as f(x) is not identically zero, i.e., f(X);i; a


and for A=1 f'(x)=l
On intergrating w.r.t. x and taking limit a to x,

then f: f'(x)dx = f:l. dx

51
=> f(x)-f(O)=x => f(x)-(O)=X (-:f(O)=O)
Hencef(x) = x and f'(x) = 1 .. f'(lO) = 1 and f(5) = 5.

r..; t f:W~R satisfies the functions equation f(xy)=eXY-X-Y{eYf(x)+eXf(y)} 'if X,yEW.

If f'{l) = e determine f(x).

iven f(xy) = eXY-x-y {eYf(x)+eXf(y)}

utting x=y=l then f(1)=e- 1(e1f(1)+e1f(1))


=> f(l) = 2f(1) .. f(l) = 0 ... (1)

f I ( x)
o

= 11m
f(x+h)-f(x) . f(X(l+~)J-f(X'l)
= hm--'-----'-------'--'---­
h....O h h....O h

X(l+~)-X-(l+~) (l+~)f(X) X.f(l+~) x-x-l{ef(x)+eXf(ll}


. e
= Ih....O
lm-------------­
e +e -e r' (0
h

h( (h)J
h-l-j{ el+!!xf(x)+exf l+j{
e -f(x)-O
= lim
h....O h
(0: f(l) = 0)

h I ' ex+h-l.-!'.xf ( 1 +­ hJ
= limf(x) (e -.) + lim x
h....O h h....O h

h X+h-l-~{t(l+~ )-f(ll}
= f(x)lim(~)
h + lim e
h....O h....O
(h)
x· ­ (
x

eX-l.f'(l) () ex-1·e
) 1+
= f( x· =f x + - ­ (-:f'(l)=e)
x X

eX eX
=f(x)+­ f'(x) - f(x) =­
x x

f'(x) - f(x) = 1. eXf X) - eXf (x) _ 1


I (

eX x e 2x --;

52

on integrating we get f (x) = In x + c ... (2)


eX

Putting x = 1 then f(l) = 0 +c = 0


e

.. c == 0

from (2), f(x) = eX In x.

63 Let f: W ~ R be a differentiable function with f(l) = 3 and satisfying:

f f (t) dt = y r
f (t) dt + x f f (t) dt; '<j x, yEW

then find f (x).

Sol We have ff(t)dt=yff(t)dt+xff(t)dt

Differentiating both sides w.r.t. x treating y as constant; we get

f(xy).y==yf(x)+ S:f(t)dt
.,1"
(
Putting x == 1, we get yf(y) == Yf(l) + S: f(t)dt
~ yf(y)==3y+ J:f(t)dt (-:f(1)=3)

Again differentiating both sides w.r.t. y, we get

yf'(y) + f(y).l == 3 + f(y)

~ f'(y) == ~
y
Integrating both sides w.r.t. y with limit 1 to x then
yf'(l) = 31nx - 3ln1

f (x) - f (1) == 3ln x - 3ln1

~ f(x) - 3 = 3ln x - 0 (-: f(l) == 3)

( ~ f(x)==3+3Inx

== 3lne + 3lnx == 31n( ex)

Hence f(x) = 31n( ex).

64 Let f(xrnyn)==mf(x)+nf(y) '<j x.y e R" and '<j m.n s R If f'(x) exists and has the value

~,then find lim f(l+x) .


x_ x-->o X

Sol .. f(xrnyn) = mf(x)+nf(y) .... (1)

Putting x=y=m=n=l, then f(l)=f(l)+f(l)


~ f(l)=O

53
. f(x+h)-f(x) . f(
f X) == lim
1( == lim----"--'--------'--'-------­
X(l+~))-f(X'l)
h-->O h h-->O h

.+vmr {(l+;JT}-+vmf((l)""r)
== hm-----"----------'-------­
h-->O h

. ml(X'lm) + nl{ + ~r} ml(xV m ) - nl(l)


== hm------"--------<------­
(1 -
h-->O h

r'
('
(Putting y == 1 in (1) then f( x m ) == mf(x))

f(l+~) ( h)
f 1+-
~ == ..!lim x lim x == e

x x h-->O (~) h-vO (~)

f (1 + x)
Hence lim == e
h-->O X

65 Let f be a continuous and differentiable function in (X1,X2). If f(x).f'(x) ~ x~l-(f(x)t

1
and lim. (f(X))2 == 1 and lim_ (f(X))2 == 2 for x E (X1,X2), then prove that
X4~ X~X2
x~ -x~ ~ -3
1t

(assume that ~~f(g(x)) == f(~i~g(x)) holds everywhere).

Sol Given f(x), f'(x) ~ x~l-(f(x)r

~ f(x)f'(x) -x ~ a or 2f (x)f' (x) _ 2x ~ a


J1-(f(x)r J1-(f(x)r

or d]lsin-1(f(x))2}
dx _x 2 ~O

~ F(x)==sin-1(f(x))2 _x 2 is a non decreasing function.

~ lim. F (x) :s; lim F (x)

X4Xl X4X2

54
1t 2 1t 2 2 2 1t
--X <--X X -X >-.
2 1 - 6 2 1 2 - 3

66 Are there any non-constant differentiable functioef: R ---+ R such that


f(f(f(x))) =f(x) 2: 0 Iv' xER? ~
Sol Given f(f(f(x))) = f(x) .... (1)
Applying f to both sides of the equation (1), then
f(f(f(x))) = f{f(x)} .... (2)

If g(x) = f(f(x)) Iv' X E R then equation (2) can be written as g(g(x)) = g(x); g is also
a differentiable function on Rand g(x) 2: 0 Iv' X E R.
Then the range T=g(R) of g is an interval in [0,00). Let a be the infimum of T.
Since g(t) = t for all t E T and g is continuous.
=? g(a)=a
Assume T has more than one element. Choose 8 > 0 such that (a, a + 8 ~ T).
Then X E (a - 8,a)

g(x)-g(a) ~ 0
g(x) 2: g(a) = a
x-a

Lg (a) = lim g ( x) - g (a)


I ~0
x~a- x-a

=limg(a-h)-g(a) ~O
.... (3)
h~O -.:h

g(x)-g(a) =1
For x E (a,a+8) we have
x-a
f Hence Rg'(a) = lim g(x)-g(a) = 1 .... (4)
x-a
x~a+

As g is differentiable at a, therefore (3) and (4) are contradictory. This concludes that
T is a single point i.e., g is a constant function,
g(x)=c Iv' xER, (c is constant)
from (1), f( c) = f(x) Iv' x E R
This shows that f is a constant function. Thus there is no non-constant differentiable
function satisfying .(1).

67

55
g (x) = {max {f(t) : x +1 :s; t :s; x + 2,-3 :s; x < O}

I-x, for x ~ 0

Test continuity of g(x) for xE[-3,1].


Sol Since f(x)=x 3-3x2+6
~ fl(X) = 3x 2 -6x
= 3x(x -2)
for maximum and minima f'(x) = 0
x=0,2

f"(x) = 6x-6

f"(O) = -6 < 0 (local maxima at x = 0 )

f"(2) = 6 > 0 (local minima at x = 2 )

Cut off x-axis x3 _ 3x 2 + 6 = 0 has maximum 2 positive and 1 negative real roots.

Cut off y-axis. F(O) = 6.

Now graph of f(x) is :

Clearly f(x) is increasing in (-oo,0)u(2,oo) and decreasing in (0,2)


~ x+2 <0 ~ x < -2 ~ -3:s; x < -2

~ -2 :s; x + 1 < -1 and -1 :s; x + 2 < 0

in both cases f(x) increases (maximum) of g(x)=f(x+2)

.. g(x)=f(x+2);-3:s;x<-2 ... (1)

and if x-i-Lc O and 0:s;x+2<2 ~ -2:s;x<-1

then g(x) = f(O)

Now for x + 1 ~ 0 and x + 2 < 2 -1 s x < 0,g (x) = f (x + 1)

56

f(x+2) -3 ~ x <-2
f(O) -2~x<-1
g(x) ­
Hence - f(x+l) -1 ~ x <-0
I-x x:2:0

Hence g(x) is continuous in the interval [-3,1].


[2rx]
n
68 If f(x) = lim I-2-'
n
n-seo r=l
discuss the continuity of f(x) where [.] denotes the greatest

integer function.
Sol Since x =[x]+[x]
=> [x] = x - {x}, where {x} is the fractional part of x.
.. [2rx] = 2rx - {2rx}

Now, 0 ~ {2rx} < 1 for all r


n n n

I
r=l
0 ~ I {2rx} < II
r=l r=l

{2rx} 1
lim 0 ~ lim
n-4oo
I-2-
n

n-4oo r=l n
< lim­
n n-seo

From Sandwitch theorem :

.... (1)

f(x) = lim I[2rx]


n-4oo r=l
-
n

n 2

= lim f_2rx_ - 7-{2_rx~}


n-4OO r=l n2

n 2rx n {2rx}
= lim , , - - lim , , -2­
n-4oo
2
f::t n n-4oo f::t n

. -1 ( 1 + -1 J -
= 2x lim lim . I--
{2rx}
n
2
n-4oo 2 n n
n-4oo r=l

1- .
= 2x. (1+0)-0 { from (1) }
Z
=X \;j XER.

Hence f(x) = x \;j X E R,which is continuous everywhere.

57
I - ax+xaXlna

i
x 2 x<O
function f(x) = a .x
2xax- x In 2 - x Ina -1
2 x>O
x
where a> .

Without u ing C Hospital's rule or series expansion, find the value of 'a' and 'f(0)'

so that f( ) is continuous at x = O.

For x < ,

.... (1)

P lim a 21 -1- 2t In a (
1-.0 4e

_1I'
- -
41--+0
1m [ -
t
1 2I'
a 1 --)2 + -
4
1m [ -
1--+0
1e
-
a 1--- ; ctin
--­
a)

1 ( Ina )2 +-P
=- 1 [ from (1) ]
4 2

P=.!.(lna)2 .... (2)


2
For x> 0,
RH.L. = lim f(x) = limf(O + h)
x->O+ h->O
(
. 2 h.ah - h In 2 - h In a-I
= hm---------:~---
h->O h2

= lim[a h
h->O
-1)[2 h-1) + lim[a
h
h

h-.O
h - h In a
h2
-1) + lim[2
h-.O
h
- h In
h2
2-1)
1 - I'
= (In a)(ln 2) + -(In a)2 + -(In 2/ [ from (1) ]
2 2
But f(x) is continuous at x = 0
LH.L. = RH.L.

58

..

!(ina)2 = (In a) (in 2)+!(Ina)2 +!(In2)2


2 2 2

0= (In a Hin 2) + ~(in 2)2

1
=> 0= In a +2"In 2 (': In 27:-0)

=> In a = _! In 2 = In (2r~ a=2 2


2

and f(0)=L.H.L.=~(Ina)2

1(1
= 2" -2"In 2
1
)2 = 8(ln 2) 2.

X\I-X)Sin( ~) if Oc x s j '
70 I: [0, I] -)0 R is defined as I(x) = x , then prove that
[
° if x = °
(a) / is differentiable in [0, I] (b) / is bounded in [0, I] (c) /' is bounded in [0, 1]

X\I-X)Sin(:2) if Oc x s l
[Sol. I(x) =
[
° if x =0

f t (0+) = Lim
h \1- h) sin ~ -
h
°
°
=
h~O h

f' (1-) = Lim


(1- h)\+h)sin 1 2
(1- h)
° = Lim - (1- h)3 sin 1 2 = - sin 1

h-e-O -h h~O (I-h)

Hence f is derivable in [0, I], obviously f is continuous in [0, I] hence f is bounded

(x3 -X4)Cos(~J(-4)+sin~(3X2 -4x 3) x 7:- °


hence f' (x) = x x x
[
° if x =°

Lim = (0)+sinl(3-4) , hence f' is also bounded]

x~l-

·
L im (xx-J)
71 Find the value of x~O+ x . [Ans. I]

[Sol. I = Lim x(xX-I) (00 form)


x~O+

(ex1nx_I)

Inl=Lim (xX-I).lnx = Limit~ Limit xlnx .lnx

x~o x-e-O x In x x-e-O

59
=Limit x(ln x)z (as x ~ 0 x lnx ~ 0)

x~o

. . (lnx)z .. 2lnx z

=Llmlt - ­ = Limit ---.X (use Lopital's rule)


x~o I/x x~o x
= Limit -2Inx.x = 0
x~o

. I-cosx·cos2x·cos3x........ cosnx
72 If L im has the value equal to 253, find the value of n
x~o xZ
(where n EN). [Ans. 11]

[Sol. L= 'L>z =253' n(n+l)(2n+l) =506' n == 11 satisfies this. Ans. ]


2 ' 6 '
73 Let a l > az > ll:3 an> 1; PI > Pz> P3"..· > Pn > 0 ; such that PI + Pz + P3 + + Pn = 1

Also F (x) = ~la~ +pza~ + ....... +Pna~rx. Compute

(a) Lim F(x) (b) LimF(x) (c) Lim F(x)


//. x~o+ x~oo x~-oo.
[Sol. (
(1) x -­ L'rm ~ la lx +pza zx + ....... +P a x JlX
L'im F() (100 form)

x~o+ x~o+ n n
x x x 1
L = e1 where I == Lim PIal +pzaz +· ...... +Pnan­
1 x~o x
using L'HospitaI's Rule

I= ~~tIJ- (PI In a) a~ + pzlna z a~ +:...... + P n In an a~)


= PI In a) + pzln az + + Pn In an
== In raPI Pz Pn)
~ 1 'a Z .....a n

L ) = e1 = a )Pj ·a Z··
Pz a Pn
.. ·n Ans•

(2) Lim Ftxj>' L, =


x~oo
Lim(p)a~
x~oo
+pza~ + ....... +P naXn r (00 0 form)
[only when a) az etc. > 1]
. In(p)a~ +pza; + ....... +P aX)

In L = Lim n n
Z x~oo x (
using L'Hospital's Rule

L = Lim ~) Ina) a~ +pz/na z a~ +· +P n In an a~)


....(1)
z x~co p)a~ +pza~ + +Pna~

dividing by a~ and taking limit, we get

.
Lim ,
az
-
x [
, -
a3 ]X , etc all vanishes as x ~ 00
X~OO [ a) ] az .. .

p) Inal
= In a)
Pj
hence In L z = In a l => L z = a) Ans,
(3) Lim F(x) = L3 (say)
x~-co

60
dividing by (an? and taking x~~, anI ]X, [aa2n JX etc vanishes

[a

InL = Pn/nan

3
Pn
Lim 1-cos3x·cos9x·cos27x cos3°x 10 fi h I f
74 If ------------=-------,-----------,---- = 3 , ind t eva ue 0 n. [Ans.4]
x~o 1 1 1 1
1- cos - x .cos - x .cos - x cos - x

3 9 27 3°

2x 3x
· 1-cos3x 'cos3 ·cos3 cos J" x
[Sol. Let U = L l f f i - - - - - - - - -2
= - - - - - - - ­
n x~o x

x x x x

1-cos- ·coS 2·coS 3 cos n

and
V = Lim 3 3 3 3

.n x~o x 2

· -D(cos3x·cos32x·cos33X cos3°x)
U = L lffi-----O..-----------------'­
° x~o 2x

now let Y= cos 3x . cos 3 2x· cos'x cos 3 nx

In y = In cos 3x + In cos 32x + + In cos 3nx

1 dy

Ydx = - [3 tan 3x + 32 tan 32x + + 3 n tan 3 nx]

dy
dx
= _ IT
r~l

cos S' x[3tan3x+3 2 tan3 2x+ +3° tan 3° x]

2 2x+ n nx 2 2)2 3)2


U = Lim 3tan3x+3 tan3 +3 tan3 = 3 +(3 +(3 + +(3 n)2
n X~o 2x 2

32[32n -1]
U o = (32-1)'2 ....(1)

1
lilly replacing 3r by y:- we get

....(2)

2n +2 = 10 ~ In = 41 Ans.]

61

Das könnte Ihnen auch gefallen